You are on page 1of 324

This course material is aligned to the IASSC Body of Knowledge for Black belt.

It helps you to
prepare for the IASSC certified Lean Six Sigma Black Belt Examination. It helps you to develop
skills to manage projects using Lean Six Sigma DMAIC methodology

MODULE 1.0 Define Phase


1.1 Basics of Six Sigma

Six Sigma is a process based or project-based approach for improving the quality of the products and
services. In other words, it enables businesses to provide products or services which are reliable and
consistent.

Six sigma is a methodology, which aims at improving process and increasing customer satisfaction (both
internal & external). The concept behind this approach is to reduce the variation in processes. This
reduction leads to consistent and desired outcomes from processes. Hence, Continuous process
improvement with low defects is the goal of this method. We can infer, that Six Sigma Methodology is a
customer focused continuous improvement strategy that minimizes defects and variation towards an
achievement of in product design, production or administrative process.

The word Sigma is a statistical term that measures how far a given process deviates from perfection. A
six sigma process is one in which 99.9997% of all opportunities to produce some feature of a part are
statistically expected to be free of defects (3.4 defective features per million opportunities). The
central idea behind Six Sigma: If you can measure how many "defects" you have in a process, you can
systematically figure out how to eliminate them and get as close to "zero defects" as possible and
specifically it means a failure rate of 3.4 parts per million or 99.9997% perfect.

WHY we need six sigma methodology

Variability in a process is inevitable

In all our processes, variation in the inputs and the process conditions lead to variation in the
output or the desired results

Sometimes the variation in the process is too large that we get the outputs not in the acceptable
level, i.e. we produce defectives
As the variation increase over time, the defectives also increase
Types of variations in the process:

1. Special cause variation / Assignable cause Variation: It is a shift in output caused by a


specific factor such as environmental conditions or process input parameters. It can be
accounted for directly and potentially removed and is a measure of process control for exp,
for recording customer contact information, an untrained operator new to the job makes
numerous data-entry errors.

2. Common cause variability is a source of variation caused by unknown factors that result
in a steady but random distribution of output around the average of the data. For exp, for
recording customer contact information, an experienced operator makes an occasional
error.
Common cause variation is a measure of the process's potential, or how well the process can
perform when special cause variation is removed.
Six Sigma Methodology helps you to identify the root causes of the common cause
variations by using advanced Statistical tools. Six Sigma Methodology does not help in
solving problems caused by Special cause variations

KEY FEATURES OF SIX SIGMA

 6 SIGMA is a structured, data driven, statistical methodology that can be used to improve
the quality and performance of the service to the highest levels by reducing variability in the
process
 At 6 SIGMA level of quality, the Number of Defect is 3.4 Per Million Opportunities (DPMO)
 This methodology was Initially implemented in Motorola, GE, Allied Signals
 It was not a new invention, it was just a consolidation of quality concepts and the tools
prevalent in the world
 It is a collection of tools to solve processproblems
 It is a robust but not rigidmethodology

Reasons why Six Sigma works:

• Bottom line results: It is reduction in cost, and eliminating revenue leakage, etc.

• Senior management is involved: Six Sigma is both a top down and a bottom’s up approach where
support of senior management is also a must.

• A disciplined approach is used (DMAIC): Six Sigma is the only methodology available which is a
documented step-by-step methodology for resolving business problems.
• Short project completion times (3 to 6 months): Due to the rigor in the approach of Six Sigma, it
takes approximately 3 to 6 months to complete the six sigma project.

• Clearly defined measures of success: Measures of success need to be defined in the measure
phase and need to be articulated very well in order to process to later phases of Six Sigma.
• Infrastructure of trained individuals (black belts, green belts): Six Sigma also focuses on building
the team’s skills and competencies by providing an infrastructure to train individuals as green
belts, black belts, master black belts, champions, etc.

• Customers and processes are the focus: Six Sigma always starts with the customer and focuses on
creating robust processes.

• A sound statistical approach is used: Statistics helps in taking data based decisions which are
mostly correct. Six Sigma focuses on creating a sound statistical approach.
Organizations that follow a six sigma improvement process for several years find that some operations
achieve greater than six sigma quality. When operations reach six sigma quality, defects become so rare
that when they do occur, they receive the full attention necessary to determine and correct the root
cause. As a result, key operations frequently end up realizing better than six sigma quality.

Sigma Reject(PPM) %Yield Cost of Poor


Quality
6.0 3.4 99.99966 Less than
10% of sales
5.5 32 99.99968
5.0 233 99.9767 10-15% of
Sales
4.5 1,350 99.865
4.0 6,210 99.379 15-20% of
Sales
3.5 22,700 97.73
3.0 66,807 93.3193 20-30% of
Sales
2.5 158,650 84.135
2.0 308,537 69.1463 30-40% of
Sales
1.5 5000,000 50.00 More than
40% of sales
3.4 out of a million transactions delivered late 3.4 out of a million respondents were dissatisfied from
conducted survey 3.4 out of a million hiring requirements were not met
For a process performing at 6 sigma level, the process would have only 3.4 Defects per Million
Opportunities. Similarly, for a 5 sigma level process, the DPMO will be 233, for 4 sigma, 6210 and likewise
for 1 sigma, the DPMO is 697,672.
Table lists the sigma level against Defects Per Million Opportunities. In the table, you would see that at 6
sigma level the DPMO is 3.4 only, for 5 sigma the DPMO is 233, for 4 sigma it is 6210, 3 sigma it is 66,810,
for 2 sigma it is 308,770 and for 1 sigma it is 697,672. The principle of Six Sigma is to deliver near perfect
products and services by improving the process and eliminating defects. The end objective is to delight
the customers.

For example, if a product must have a thickness between 10.32 and 10.38 inches to meet customer
requirements, then the process mean should be around 10.35, with a standard deviation less than 0.005
(10.38 would be 6 standard deviations away from 10.35).

The two most widely used Six Sigma methodologies are DMAIC and DMADV. Both methods are designed
so a business process will be more efficient and effective. While both of these methodologies share some
important characteristics, they are not interchangeable and were developed for use in differing business
processes. Before comparing these two approaches in more detail, let’s review what the acronyms stand
for.

• DMAIC: Define, Measure, Analyze, Improve, Control

• DMADV: Define, Measure, Analyze, Design, Verify

In general, DMADV is associated with new services and product designs; it may not always work with
existing products and processes.
When there is no existing product, DMADV can be implemented to design the product or process.
DMAIC is used on a product or process that already exists but is no longer meeting customer needs
and/or specifications.
DMAIC:
Define: Define the problem statement, create a project charter and map the as-is process

Measure: Collect data from the process, baseline current quality level and perform measurement
system analysis
Analyze: Study the business process and the data generated to understand the root causes of the
problem

Improve: Identify possible improvement actions, prioritize them, test the improvements, finalize
the improvement action plan

Control: Full scale implementation of improvement action plan. Set up controls to monitor the
system so that gains are sustained

The business successes that result from a six-sigma initiative include:

• Cost reductions: Reduction of cost is critical for running a company successfully.

• Productivity improvements: Productivity improvement implies maximizing performance of

resources to give optimum result.

• Market - share growth: In this fast growing world, competition is becoming stiffer at every
moment. Market-share growth can be achieved through going beyond boundaries of our location
and globalizing our organization.

• Customer relations improvements: If customers are not happy, the organization is destined to
doom. Hence, customer relations is the most critical aspect.

• Defect reductions: Defects is rework and each reworked transaction is additional cost. • Product
and service improvements: Products or service need to change with time to acclimatize to the
recent changes in the world. A classic example is that of i-Phones, we have seen several improved
versions of i-phones in last few years.

• Culture changes: Culture changes need to be accommodated in order to thrive in the current
fierce and competitive world.

• Cycle - time reductions: From order delivery to receipt of cash, if the time taken to process all of
this is high, we are not in a great position in the organization. Hence, cycle-time reduction is a
critical aspect.

1.1.2 General History of Six sigma and Continuous improvement


Six Sigma (6σ) was introduced by engineer Bill Smith while working at Motorola in 1986. The term Six
Sigma originated from terminology associated with statistical modeling of manufacturing processes.
The maturity of a manufacturing process can be described by a sigma rating indicating its yield or the
percentage of defect-free products it creates. Motorola set a goal of "six sigma" for all of its
manufacturing.

Other early adopters of Six Sigma include Honeywell and General Electric, where Jack
Welch introduced the method. By the late 1990s, about two-thirds of the Fortune 500organizations
had begun Six Sigma initiatives with the aim of reducing costs and improving quality.

We can see many such examples across the globe where Six Sigma has changed the way organizations are
working. Six Sigma is not only a process improvement methodology, but also a tool to have bottom line
savings.

Here are some more Six Sigma benefits:

• Motorola® credits the six sigma initiative for savings of $940 million over three years.

• AlliedSignal® (now Honeywell®) reported an estimated $1.5 billion in savings in 1997.

• GE® has invested a billion dollars with a return of $1.75 billion in 1998 and an accumulated
saving of.$2.5 billion for 1999.

Organization-wide Planning and Deployment of Six Sigma

1.1.3 Deliverables of Lean Six Sigma Project

Embarking on a Lean Six Sigma initiative begins with a management decision to embrace a change that
says “There’s a better way to run our organization.” The readiness assessment includes a review of the
following areas:

Assess the outlook and future path of the business:


• Is the strategy course clear for the company? • Can we meet our financial and growth

goals?

• Does our organization respond effectively to new circumstances?


Evaluate the current organizational performance:

• What are our current overall business results?

• How effectively do we meet customer requirements?

• How effectively are we operating?

Review the capacity for system’s change and improvement:

• How effective are we in managing system changes?

• How well are our cross functional processes managed?

• Are our current efforts in conflict with six sigma?

The above assessment will go a long way towards deciding if current efforts are sufficient or whether
the timing is appropriate to undertake a six sigma effort. Lean six sigma can be applied as a targeted
approach. A number of so-called lean six sigma companies have improvement techniques and teams in
place and only assign black belt assistance as needed.

A decision on six sigma might be negative if the following conditions exist:

• The company already has an effective improvement effort inplace

• Current changes are already overwhelming the company's resources

• The potential gains aren’t sufficient to finance the necessary investments

1.1.4 The Problem Solving Strategy Y=f(X)

In this equation X represents the input of the process and Y the output of the process .Y is a function
of X.

Y is the dependent output variable of a process. It is used to monitor a process to see if it is out of
control, or if symptoms are developing within a process. It is a function of the X’s that contribute to
the process.
Y is the output measure, such as process cycle time or customer satisfaction. f is the letter
representing “function” (what the value(s) of X(s) does/do for Y (the output). X(s) is/are any process
input(s) (variables) having assigned or inherent values(s) that is/are involved in producing the output.
To get results, should we focus on the Y or X? Y is equal to the Effect and X is equal to Causes.
Simply put, the Y = f(x) means that the process outcome is a result of the different process inputs. The
ultimate goal of DMAIC is to be able to identify which are the few processes and input variables that
have more influence on the process outcome measures. So, you can simply see that each one of the
DMAIC phases can be described as how it contributes to the goal:
Define: Understanding the project Y and how you can measure it.
Most projects usually start with an undefined business or process problem and knowing where to
start can be difficult. So, your main goal here needs to be to start by clearly stating the problem.

Measure: Prioritizing the potential x's and measuring both the x's and the Y.

This stage usually begins with a detailed process mapping, which will help you identify the all the
potential causes (the x's) as well as the ones that can have a greater influence on the Y. Since you
haven't got any facts or data yet, you need to remember that you're basing your decisions only on
your perceptions. Only after this is that your team needs to collect the data not only for measuring
the x's and the Y as well as to identify the cause-and-effect relationships. Then, the team needs to
identify any patterns in the data collected by using different charts. Sometimes, you may need to
use Pareto analysis to help you during the process.

Analyze: Testing the different relationships between the x and the Y and quantify or verify the most
important x's.

If you're afraid because there are so many different tools to perform the analyses, you don't have
to. They only exist because there are different types of data (discrete or continuous). This is the
phase where you'll need to analyze all the statistical tools like the regression analysis, hypothesis
tests, among others, as well as all graphical tools like scatter plots, pie charts, stratified frequency
plots, among others.
Improve: Implementing the solutions that allow you to improve the Y and also address the most
important x's.

This is the phase where you're looking for solutions to the problems you've identified. When you find
yourself with many different solutions, you need to use one single criterion and select the best
solutions according to that criterion only. Despite there are many different criteria, there are two that
are the most used: how much the solution addresses the specific x's and how much the solution
contributes to improving the Y.
Control: Monitoring, over time, the most important x's and the Y. In this phase, you'll need to make
sure that the new performance of the Y can be maintained over time. So, you'll need to create a
process management chart that will show the critical checkpoints that occur during the process, the
new process flow, as well as any recommended actions in case it's not working as it should.

Here is an example for a pizza delivery company that desires to meet customer expectations of on
time delivery.

Measure = On time pizza deliveries

"Y" would be the percent of on time deliveries

"f(x)" would be the x's or reasons or things that heavily influence on time deliveries
x1: traffic x2: # of

delivery's en route x3:

directions x4: reliability of

delivery vehicle

The statement Y=f(x) in this example will refer to the proven x's determined through the steps of a
Six Sigma project.

1.1.5 Voice of the Customer, Business and Employee

Customers can constitute

Lost Prospective
Current,happy
Customer Customer
customer

Competitor’s
Current, customer
unhappy
customer

Voice of the Customer

Six sigma quality is built around the customer. Everything starts and ends with customers. They
define quality and set expectations. They rightfully expect performance, reliability, competitive
prices, on-time delivery, service, and clear and accurate transaction processing.
At times, the customer of the project may not be as evident as initially thought. The receiver of the
next operation, an internal department, could be thought of as a customer. The external customer
of a process could be the purchaser. But yet, if the purchaser is a distributor, then they may not really
be the true customer.
Green Belts and Black Belts must consider how both internal and external customers can be identified,
and what their requirements might be in order to understand and improve the business process. The
relationship that management can develop with either basic customer type will affect the company's
ability to be effective in delivering customer satisfaction.
Internal Customers

An internal customer can be defined as anyone in the company who is affected by the product or service
as it is being generated. An internal customer is a customer who is directly connected to an organization,
and is usually (but not necessarily) internal to the organization. Internal customersare usually
stakeholders, employees, or shareholders, but the definition also encompasses creditors and external
regulators.

For example, in many cases, an information technology department serves primarily internal customers.
IT departments may provide a service desk that represents a internal customer service function for
technical services such as software support.

Internal employee communications for customer satisfaction can be improved through the following
options:

• Company newsletters

• Story boards

• Team meetings

• Posting customer letters

• Staff meetings

• Display of goals

• Quality awards

External Customers
External customers are not part of the organization but are impacted by it. End users, Intermediate
customers, and impacted parties are described in more detail below. Generally, external customers play
a critical role by providing the major portion of company revenues.

Voice of External Customer (VOC) Methods:


Surveys: Surveys are a designed set of questionnaire which is sent out to potential or existing customers.
Surveys are cost effective, however, have very low response rate.

Interviews: Interviews are individual meetings with potential or existing customers where a set of
questions are asked and answers are discussed to understand customer voices. Interviews can tackle
Complex Issues, however, requires Trained Resources.
Focus Group: A group of people are called together in one conference room and a discussion is held on
specific topics that need to be addressed. These focus groups are excellent for identifying the CTQ (Critical
to Quality), however, are difficult to generalize.
Suggestions: Client/Customer/Employee feedback is received and treated as suggestion to improve
product or service. Suggestions provide good improvement opportunities, however, does not look at
overall process.

Observations: During the course of the process, individuals can have observations and can provide
feedback to the process which does act as a Voice of Customer instruments to gather Data.
Market research: Market research is the systematic gathering and interpretation of information about
individuals or organizations using statistical and analytical methods and techniques of the applied social
sciences to gain insight or support decision making.

The information gathered should allow the organization to identify customer requirements and to spot
upcoming trends. The trends will be new ways for the company to gain or retain customers. Clearly, the
voice of the customer is critical to business success

Voice of the Business

The voice of the business is derived from financial information and data. By dissecting the financials,
analysts can identify market weakness, utilization of investment capital, research and development
status, and process complexity. Based on organizational strategy and direction, understanding the voice
of the business assists in identifying potential projects to aid in moving the organization closer to its goals
and objectives.

The relevance of this evaluation is that it allows analysts to pinpoint the financial outcomes of projects
that have been chosen and deployed, and it reveals their ultimate value. By identifying value levers
(strategic, process, customer and financial) and prioritizing them, analysts can translate the information
into opportunity areas, project level of effort and value, and apply a valuation criteria that leads to project
selection.

Thus, the impact of the voice of the business is that it provides clarity in terms of identifying revenue
growth areas, economic value added and market value.

Voice of the Employee

The voice of the employee is crucial to carrying out the message and mission of the company. The
employee voice brings about personal ownership and a sense of responsibility that employees are
directly contributing to the business’s success. Through their voice, employees provide a dialogue with
management that sheds light on how they will apply the company vision to their daily tasks and how
they will engage within the project selection process with department champions, project sponsors and
Belts.
An overarching impact that is sometimes overlooked is the influence employees have on cultural change.
By understanding the vision and articulating it to fellow employees, they take on an advocacy role that
encourages synergy and forward movement together.

The voice of the employee is also linked to the voice of the process in that problem areas can be identified
along with the voice of the customer and voice of the business to identify potential opportunities.

Employees also have an impact on financial results. By participating in project selection, they can share
information related to resources, costs, talent, processes and solutions that provide strategic direction.
Through the project selection phase, they continually manage project portfolios for their specific
departments and help identify potential financial outcomes.
1.1.6 Six Sigma Roles and Responsibilities

Six Sigma Roles:

Many organizations have implemented the following roles in their six sigma programs.

• Executive Sponsors

• Champions

• Master Black Belts

• Black Belts

• Green Belts

• Process Owners

Six Sigma Roles and Responsibilities – Executive Sponsors

Executive sponsorship is a key element in an effective black belt program.Executive leadership sets the
direction and priorities for the organization. Team is comprised of leaders that communicate, lead, and
direct company’s overall objectives. Typically receive training that includes a six sigma program
overview. Sponsors are key members in any Project as they determine the success of the Project.

Six Sigma Roles and Responsibilities – Champions

Six sigma champions are typically upper level managers that control and allocate resources to promote
process improvements and black belt development. Champions are trained in the core concepts of six
sigma and deployment strategies used by their organization. With this training, six sigma champions lead
the implementation of the six sigma program. Champions also work with black belts to ensure that senior
management is aware of the status of six sigma deployment. Champions ensure that resources are
available for training and project completion.
Six Sigma Roles and Responsibilities -Process Owners

Key processes should have a process owner. A process owner coordinates process improvement activities
and monitors progress on a regular basis. Process owners work with black belts to improve the processes
for which they are responsible. Process owners should have basic training in the core statistical tools but
will typically only gain proficiency with those techniques used to improve their individual processes. In
some organizations, process owners may be six sigma champions.
Six Sigma Roles and Responsibilities – Black Belts

Six sigma black belts are most effective in full-time process improvement positions. The term black belt is borrowed
from the martial arts, where the black belt is the expert who coaches and trains others as well as demonstrates a mastery
of the art. In a similar way, six sigma black belts are individuals who have studied and demonstrated skill in
implementation of the principles, practices, and techniques of six sigma for maximum cost reduction and profit
improvement.

Black belts typically demonstrate their skills through significant financial improvement and customer benefits on
multiple projects. Black belts may be utilized as team leaders responsible for measuring, analyzing, improving, and
controlling key processes that influence customer satisfaction and/or productivity growth. Black belts may also operate
as internal consultants, working with a number of teams, at once. They may also be utilized as instructors for problem
solving and statistics classes. Black belts are encouraged to mentor green belt and black belt candidates.

Potential black belts often undertake four weeks of instruction over a three or four month period. A set of software
packages can be used to aid in the presentation of projects, including Excel or MINITABTM for the statistical portions.
Specific elements will differ, but all stress and understanding of variation reduction, training, and project management.
Black belts often receive coaching from a master black belt to guide them through projects. Black belts have the
following duties in their company:

• Mentor: Have a network of six sigma individuals in the company

• Teacher: Train local personnel

• Coach: Provide support to personnel on local projects

• Identifier: Discover opportunities for improvement

• Influencer: Be an advocate of six sigma tools and strategy

Six Sigma Roles and Responsibilities – Green Belt

Green belts are not usually in a full time process improvement positions. Green belt are Individuals who have mastered
the basic skills, but have less experience than black belt. They demonstrate proficiency with statistical tools by using
them for positive financial impact and customer benefits. Green belt operate under the supervision and guidance of a
black belt or master black belt. Green Belts receive training in the basics of Six Sigma so that they can lead smallscale projects
within their own business areas. They are expected to devote a substantial amount of their time to learning and
implementing Six Sigma, essentially as part of their core role. Unlike Black Belts, Green Belts typically conduct projects
that are contained within one functional area of an organization, and may or may not be involved in selecting the project.
Often Green Belts are the leaders of a specific business area, such as a department supervisor or manager, although
individuals in other positions such as analysts may also take on the Green Belt role. Although the required knowledge
and skill base for Green Belts is not nearly as extensive as that for Black Belts, Green Belts are expected to develop a
solid understanding of descriptive statistics, data and process variation, and effective project management.
1.2 The Fundamentals of Six Sigma

1.2.1 Defining a Process

The six sigma philosophy maintains that reducing “variation” will help solve process and
business problems. The fundamental principle on which six sigma methodology is built is the
Process Approach.
Every organization is made up of a series of interacting processes. A process is a set of
activities that uses resources (people, machines, etc.) to transform inputs into outputs. The
process approach considers the interaction between these processes, and the inputs and
outputs that tie these processes together. The output of one process becomes the input of
another. A process is a set of activities done in order to convert the inputs into outputs.
Process can be subdivided into sub processes. All the process problems (manufacturing /
services / software development) can be solved by analyzing problems in the inputs and the
process conditions.
Process Analysis
Processes are rarely designed intentionally and they all evolve overtime. Throughout this
evolution, inefficiency, redundancy and waste creep in. It is critical to avoid finding fault with
a specific process: recognize that people and departments make changes in an effort to get
the job done, often without knowledge of how those changes might impact upstream and
downstream processes. The performance of individuals is only as good as the process will
allow it to be. Processes, especially cross-functional business practices, are usually not
documented, not standardized, not measured, not systematically and continually improved
and not managed by the micro-process doer or owner.

Most businesses, at some time or another, need to stop and examine their processes to identify and
eliminate waste and inefficiencies.
By using a set of statistical tools to understand the fluctuation of a process, management can
begin to predict the expected outcome of that process. If the outcome is not satisfactory,
associated tools can be used to further understand the elements influencing the process.
These efforts seek to eliminate the causes of variation in processes while leaving the basic
process intact. In six sigma terms, process improvement teams find the critical “Xs”(causes)
that create unwanted “Ys”(defects).

A good definition of a process is the most important step towards an efficient process structure.
Here are some more specific basics that can help when defining processes.
Eliminate waste—unnecessary steps, too many reviews, create from scratch when not necessary, etc.
Insure all steps add value—if it doesn’t add value, why are you doing it? (This value could be for
reporting data, compliance or other required steps.)
Define tasks simply with one verb and one noun—“Throw the ball.” Often we create
compound tasks that should really be explicit in the process definition. Recognize milestones
that take the form of “Concept Complete”which is not a task, but a result.

Be explicit at intake—It’s important that the complexity of the process match the job. We often
define processes based on doing new creative and forget that 80-90% of our work is modifying
existing pieces.
Be explicit at intake (Part 2)—Make sure all the information needed to move forward is defined
up front before starting work. Often, lots of time is wasted in going back to the client to gather
additional information or to clarify points that should have been clear at intake.
Identify the value added for each task—This is usually easy, such as, ‘an image is resized’ or
‘web page is updated with new content.’ It can even be something like, ‘enter time in tracking
system.’ This last one may not add value to the project, but it does add value to the
organization.
Be sure each task has an owner—Particularly when there are collaborative steps like kick-off
meetings or ‘review with management,’ it’s easy to identify a process step without explicitly
identifying the owner. If there is no ownership, there is no accountability.
Be clear when collaboration is needed—Sometimes processes have parallel activities such as
‘write copy’ and ‘develop design.’ If these activities require collaboration between the
participants, call it out. You may avoid having the copy and design come together later and
finding out that the design assumed headlines that aren’t there, or the text is too long for the
allotted space. Then you re back to rework that could have been easily avoided.
Identify appropriate task durations—It’s important to develop task durations that are specific
to the deliverable. ‘Create Concept’ will take longer for a brochure than for an email. Capture
this in your processes and you will be able to estimate delivery times much more accurately
(and deliver on time more frequently).
Identify the right reviewers (and the wrong ones)—Look carefully at why each reviewer is
included for each type of project. It may be possible to eliminate unneeded reviews by
empowering staff to a greater degree and also by recognizing levels of strategic importance.
In other words, maybe the VP of marketing doesn’t need to sign off on everything going
through the department.

PROCESS MAPPING LEVELS


Level 1: The Macro Process Map, sometimes called a Management level or viewpoint.

Level 2: The process map,sometimes called the worker level or viewpoint. This example is from the
Process Modelling levels

perspective of the pizza chef.


Pizza dough
No

Take order Place in Observe Check Remove if


Add 1
frequent if done done
from cashier ingredients oven
Yes

Start new
pizza

scrap
No

Pizza Put on delivery


Place in box Tape order on rack
1 correct
box
Yes

• Level one: this very high level map outlines the operational levels of an organization
and are rarely, if ever, actually drawn. Examples include: customer processes,
administrative processes.
• Level two: shows end-to-end processes across the above operational areas. For
example a level two process for purchasing capital equipment would cross several
operational areas: the requesting department, purchasing, accounts payable, asset
management, receiving and biomedical/maintenance. Also called top down or high-
level process maps. They are quick and easy to draw, but may not provide the detail
necessary to build understanding or realize improvements.
• Level three: shows the roles, inputs, outputs and steps required to complete a specific
process within an operational area. For example, the purchasing process (request,
sourcing, cut PO) might be depicted as a process map.Also called cross- functional or
deployment diagrams. Usually contain enough information for improvement efforts,
but often miss inefficient details and don’t function well for training or as operational
documentation.
• Level four: is the documentation of systems, instructions and procedures required to
complete steps in the level three processes and shows inputs, outputs, associated
steps and decision points. For example, specific steps necessary to cut a PO in the
enterprise application would require a level four process map. The procedures and
system instructions can be represented as text, an algorithm or detailed process map.
Because of the level of detail, they can be resource-intensive to create, but offer the
greatest improvement potential. Since they illustrate decisions and subsequent
actions, they are excellent training and reference
materials. But if your organization is new to process mapping, use these sparingly. The
time and effort may turn stakeholders off before they’ve had a chance to experience
the benefits of the work.
Critical to Process (CTP) are the key process input variables. These are the process parameters
which influences other critical approaches – Critical to Quality (CTQ), Critical to Delivery (CTD)
and / or Critical to Cost (CTC).
To illustrate CTP in terms of process, we must mention it as the critical x ’s in the function -
> y = f(x). It is essential to know what the x’s are – for example, temperature, pressure,
humidity, but not essentially their definite setting or level. Every organization and business
processes has constraints. There are different forms of constraints. When a product or
service or its characteristics are not sufficient to fulfil the requirements of customers, the
projects of improvement should be implemented to correct the fault or lacuna identified in
the processes. They represent the necessities of quality but have deficiency in terms of
measuring the specificity.

▪ Critical to Delivery (CTD) : Critical to Delivery (CTD) characteristics are services to


manufactured goods, and/or transactional characteristics that significantly influence one
or more CTSs in terms of Delivery. It represents the customers who have stated needs in
relation to delivery. It is of late delivery. However, early delivery might create problem
for some of the customers as it may invite excess account requiring payment before the
need for such inventory arises. CTDs are transformed into critical requirements of the
customer through the quantification of these impact areas.

Critical to Cost (CTC):Critical to Cost (CTC) analysis is the input to the Quality Function
Deployment activity designed for the satisfaction of customers’ requirements. This is the
process of analyzing the inputs and outputs and find out the path that influence the cost
of process outputs. CTC are similar to CTQ but the former deals entirely with the impact
of cost on the customer. This is the way to compute numbers and an overall project.
1.2.2 Critical to Quality Characteristics (CTQ’s)

CTQ trees (critical-to-quality trees) are the key measurable characteristics of a product or
process whose performance standards or specification limits must be met in order to
satisfy the customer.
In other words, CTQs represent the product or service characteristics as defined by the customer/user
also known as Voice of Customer.
Voice of Customer (VOC ) to Critical to Quality (CTX )Conversion
Customer requirement / VoC is the data collected from customers that gives information
about what they need from the process. Customer requirements are often high-level, vague,
and not specific. Customer requirements when translated into Critical Process Requirements
that are specific and measurable are called Critical To X (CTX) factors.

A fully developed CTX has four elements:

• Output characteristic (CTX Name)


• Y metric (CTX Measure)
• Target Specification/tolerance limits
As discussed previously, Voice of Customer is the customer’s voice, expectations,
preferences, comments, of a product or service in discussion. It is the statement made by
the customer on a particular product or service.
Customer Needs and Requirements: Need is a desire or an expectation of a customer from
a given product or service. Customers may have many stated needs which are often vague
and generally are “wants” from a product/service. For example Customer requires an Air-
Conditioner for its use in his/her bed-room. The real need of the customer is cool
temperature in the bedroom.
Similarly, the real want from the Air-Conditioner is it has to be quite, cost-effective and
maintenance free. When the customer states his/her requirements, we will often get to hear
that the need is “Cool Temperature, Less Noise (quite), Cost Effective and Free of
Maintenance ”. However, “Cool Temperature” is the need and the rest are just wants. It is
important for the project team to understand the stated needs of the customer and separa
te them from wants.
The main reason that we separate needs and wants are: Needs are important critical features
and Wants are expectations of the product or service over and beyond the needs. If the
product/service is not able to fulfill “Wants” of the customer, the customer may only be
highly displeased / dissatisfied.
However, if the product/service is not able to fulfill “Needs” of the customer, he/she will not
use the product/service and there is a high likelihood that he/she will switch to competitor s
product/service. Organizations reputation may also be at stake if “Needs” are not met.
Requirement is an attribute of the product or service which fulfils the needs of a
customer. Customer defines these requirements and are the “musts” of a product or service.
For example in the above example of an air conditioner, customer’s requirement is “Cool
Temperature” and the rest are “Good to Have” features. Customer will not buy the Air-
Conditioner if all of “Good to Have” features are present but the “Requirement(s)” are not
fulfilled.
However, customer may buy the product/service if the “Requirement” is fulfilled and “Good to
Have” features are present or not present.
Translating Voices to Needs: Voice of Customer (VOC) methodology can be used to capture
the customer needs – both current (stated needs) and latent (unstated needs). VOC
methodology helps capture the needs of customer through stated verbatim comments
(customer voices). It helps translate verbatim comments (customer voices) into customer
needs to product/service output characteristics (customer requirements).
Translating Voices to Needs: Voice of Customer (VOC) methodology can be used to capture
the customer needs – both current (stated needs) and latent (unstated needs). VOC
methodology helps capture the needs of customer through stated verbatim comments
(customer voices). It helps translate verbatim comments (customer voices) into customer
needs to product/service output characteristics (customer requirements).
Example :
VoC: 'I have to wait for 48 hours to get a reply to a single
email‘ CTX Name: Responsiveness

CTX Measure: TAT (HH:MM)


CTX Specification: Less than 24 hours from the time an email drops into the mail box.
Defect: Number of emails the TAT has been violated for (E.g. 200 emails every day) Unit:
Emails (e.g. 300 emails) Opportunity: 1 Per Email

Graphical presentation of conversion :

Example of Translation of VOC to CTQ

1.2.3 Cost of Poor Quality (COPQ)

Cost of poor quality (COPQ) or poor quality costs(PQC), are costs that would disappear if
systems, processes, and products were perfect. It quantifies the negative outcomes due to
waste, inefficiencies and defects in a process. It is a methodology that allows an organization
to determine the extent to which its resources are used for activities that prevent poor
quality, that appraise the quality of the organization's products or services, and that result
from internal and external failures. COPQ was popularized by IBM quality expert H. James
Harrington in his 1987 book ‘Poor Quality Costs’.

Joseph Juran separated these Cost of poor quality (COPQ) costs into 3 categories:
1. Prevention Costs: costs incurred to prevent errors / failures Including quality planning, training,
preventive maintenance, housekeeping etc.

2. Appraisal Costs: costs incurred to determine the degree of conformance to quality requirements
Including testing, inspection, audits, reviews and surveys, etc.

3. Failure Costs:

➢ Internal Failures: associated with defects found before the customer receives the product
or serviceIncluding scrap, rework, expediting, equipment downtime, injuries, etc.

➢ External Failures: Including product recalls ,returned products, complaint handling, lost
sales, lost reputation etc.
Cost of Poor Quality can be calculated as the sum of all the above costs.
For example, let’s considern inventory handling. If you have a lot of inventory (in lean,
inventory is considered waste), then you will have material handlers that are needed to
move the inventory along.
For example, sales people can enter the order wrong, can contribute to lost sales with a
faulty sales process, or spend too much time with the wrong customers.
COPQ is often represented as a dollar amount (or as a percentage of sales) and is
composed of the categories listed above. The goal of any organization is to reduce COPQ to
zero.
1.2.4 Pareto Analysis (80:20 rule)

Pareto Analysis is a statistical technique in decision-making used for the selection of a limited
number of tasks that produce significant overall effect.
The Pareto principle (also known as the 80/20 rule, the law of the vital few, or the principle of
factor sparsity) states that, for many events,the principle dictates that 80% of the failures are
coming from 20% of the causes.
As a tool in Six Sigma, Pareto is part of the quality control tools that are derived from historical
data in order to come up with efficient and most appropriate actions to address most common
and impacting failures. With the use of Pareto, scarce resources are efficiently allocated. Pareto
analysis only applies to historical data and will not provide forecast analysis. Another limitation
of the Pareto is that it relies only on the data which it is based on.
Pareto diagrams are very specialized forms of column graphs. They are used to prioritize
problems (or opportunities) so that the major problems (or opportunities) can be identified.
Pareto diagrams can help six sigma teams get a clear picture of where the greatest contribution
can be made.

Pareto diagrams are used to:


• Analyze a problem from a new perspective

• Focus attention on problems in priority order

• Compare data changes during different time periods

• Provide a basis for the construction of a cumulative line

Pareto Chart - Example - Issue Management in a software company

Why a high number of tickets are raised in the issue management

tool? The process improvement team chose to study

the data for 90 days .


Data is summarized in terms of the % of the total for each category of cause and their cumulative %.

S.No. Causes / defects Frequency

1 Mouse issues 272

2 Keyboard issues 186

3 Unable to install software 127


4 Virus issues 99

5 Intranet connectivity 19

6 Internet Connectivity 12

7 Software gets corrupted 3

Total 718

Below is an example of a Pareto chart:


To further illustrate, the Pareto chart has been designed as a visual representation of the vital few
against the trivial many. With the help of the chart, it is easy to identify the causes of most of the
problems. Below is an example of a Pareto chart:

Let’s take the food service setting as an example to illustrate the Pareto principle. We will base
our study on data gathered from customers’ comments on the survey sheets provided to them.
We will use the information below:
Pareto Exercise Chart Food Serving Survey

The survey yielded 100 counts of dissatisfied customers and their reason for the dissatisfaction.
The first step in using the Pareto Tool is to rank the failures according to their Count (eg. The
frequency of occurrence) from highest to lowest:

Pareto Exercise Chart Food Serving Survey Step 1 Rank Failures

The second step is to compute the percent distribution of each failure. We do this by dividing
the count of each failure by the total number of failures, then multiplying by a factor of 100. For
example our biggest failure in this imaginary survey is “Food Presentation” where a total of 25
failures were accumulated. We’ll divide this by the total number of failures, which is 100, then
we multiply this by 100 to get the actual percentage – so in the end the corresponding
contribution of “Food Presentation” to the overall failures is 25%:
Pareto Exercise Chart Food Serving Survey Step 2 Calculate Percentile

In the third step, we’ll compute the cumulative contributions of the failures in order of their
rank. To do this just add the contribution of the preceding failure to the next (in percentage)
like this:

Pareto Exercise Chart Food Serving Survey Step 3 Calculate Cumulative Contributions

Now you are ready to construct your Pareto Chart. You will need the failures, count and
cumulative contribution. Highlight these variables, then using Excel format go to the “Insert”
tab and click on Column chart. It should look like this:
Pareto Exercise Chart Food Serving Survey Excel Example

Click and then right-click on “Cumulative Contribution”, under “Format Data Series” choose secondary
axis. Your table should appear as below:

Pareto Exercise Chart Food Serving Survey Excel Example Sorted By Cumulative Contributions

Click on the “Cumulative Contribution” again to change chart type from bar to line chart to complete
the Pareto chart:
Pareto Exercise Chart Food Serving Survey Example Sorted By Cumulative Contributions Line Chart

Pareto Analysis

Using the data above, we noticed that the 80% contribution lies somewhere in between “Food
Presentation” and “Overall Taste” or “Store Ambience” with the first 2-3 factors having the
biggest weight. This is the Pareto principle of unequal distribution of causes and effects – 80%
of the problems are caused by 50% of the factors.

Of course in reality it doesn’t have to be a perfect 80% / 20% distribution, and don’t get caught
in the 80/20 stigma of thinking that it must always equal 100%. In reality you may see number
such as 50% of market share being controlled by 1% of the market players, or 90% of the
problems being contributed by 30% of the factors.The conclusion from this example is that the
store can focus its scarce resources in addressing the top four reasons for dissatisfaction which
are:

“Food Presentation” – 25%

“Quality of Service” – 24%

“Cleanliness” – 15%, and “Overall

Taste” – 12%

and effectively resolves 80% of the problem of dissatisfied customers.

Without the Pareto, the business entity might be spreading out their resources in trying to
address all of the reasons of dissatisfaction. Or they might be concentrating on minor issues like
“Serving Size” (5%) and “Promptness” (5%) which don’t really have a strong impact compared
with “Food Presentation.”
From the customers’ point of view, based on the data, they don’t mind waiting a bit, getting
smaller portions and paying more. But, they want their food nicely presented – which might call
for better plates and platters, waiters and service crew to be nicer and a spic and span
surroundings.

1.2.5 Basic Six Sigma Metrics

Primary Project Metrics: Metrics are needed to ensure that the requirements for the project
are measurable and therefore controlled during the project. Primary Metrics are developed
in the Define phase of the project, but are not finalized until the Measure phase.

The primary metrics for consideration in the project come from various sources like

➢ Suppliers

➢ Internal Processes ➢ Customers

➢ Examples of the Primary metrics for the project are

➢ Quality

➢ Cycle Time

➢ Cost

➢ Value

➢ Labor

Examples of the Primary metrics for the project are :

➢ Quality

➢ Cycle Time

➢ Cost

➢ Value ➢ Labor

Secondary Project Metrics: The secondary metrics for the project are derived from the primary
metrics. These are usually numerical representations of the primary metrics.

Some examples of secondary metrics include

➢ Defects per Unit (DPU)

➢ Defect per Million Opportunities (DPMO)

➢ Average Duration of Receivables


➢ Lines of Error Free Software Code
➢ Scrap %

Before discussing various metrics, it is important to understand unit and opportunity.

➢ Unit: The event/transaction produced or processed.

➢ Opportunity: Any inputs to event/transaction that can be measured to assess the


probability of it not meeting a Customer Requirement. In other words, it is the
expression of the probability of the occurrence of an error.

Defects per Unit (DPU)


One way to understand how well a process performs is to identify the resulting Defects per Unit
(i.e. the number of defects per manufactured unit).
For exp: The chart identifies 10 total units produced. Of them, 7 are defective and there are 14 total
defects. Note that almost all units have multiple defects.
It is important to distinguish between these two concepts. A defect is an occurrence of non-
conformance to customer requirements. A defective unit is a product with one or more defects
or non-conformant features. It is possible for a single product to have multiple defects.

The formula for Defects per Unit is:


It means, 17/10, there are 1.4 Defects per Unit in this process .

Defects per Opportunity (DPO)


Defects per Opportunity describe the average defects (opportunities for defects) in each unit
produced. This is not a hard count of the number of defects, but a measure of the probability of
defects. DPO may be expressed by three separate formulae using similar data. They each depend
on the data collected from the chart above:

D = # defects (14) TOP = total # of failure opportunities

(40) O = # defect opportunities (4) DPU = defects per unit (1.4)

U = # units (10)

DPU = 0.35
DPO =
4
D
DPO =

10 4
DPO = UO 14 = 0.35
=
D 40
TOP
Either formula, using different data points, gives the same result for this process: the average
number of defects, or the probability of defects, in each produced unit is 0.35 DPO. Defects
per Million Opportunities (DPMO)

Defects per million opportunities (DPMO) is also a probability measure. It calculates the
probability that a given process will produce X defect opportunities per million units produced.
One million units is used in the formula as a convenient constant, regardless of the actual
number of units a process produces. The formula is:

DPMO = DPO x 106


DPMO = 0.35 x 106 = 350,000

This formula uses the data from the previous metric, DPO. It differs from a calculation of
(defective) Parts per Million (PPM) (will be discussed below) in that it accounts for the fact that
there are multiple failure opportunities in a single manufactured unit. One should not assume that
those opportunities automatically become actual failures. For the metric, DPMO, using the data
from the chart above, there is a probability that there are 350,000 defect opportunities.

Parts per Million (PPM)


PPM counts the quantity of defective parts per million parts produced. As noted above, it does
not account for the fact that multiple defects may affect a single part. One defective part, even
with multiple defects, counts as a single defective among other defectives in the population. As
with DPMO, it uses 1,000,000 as a constant, regardless of the actual number of parts produced.

The formula is:

PPM = Defectives percentage of a population x 106


PPM = 0.70 x 1,000,000 = 70,000
In our process example, 7 defective units (count the number of defectives, not the number of defects)
of 10 units produced yields a PPM of 70,000.

First Pass Yield

• First Time Yield(Yft) - units passed/ Units input for First


time

Rolled throughput Yield

• Rolled Throughput Yield (Ytp) - Yield1*Yield2*Yield3*Yield4

Cycle Time

• Cycle Time = 1/Throughput Rate


First Pass Yield :First pass yield (FPY), also known as throughput yield (TPY), is defined as the
number of units coming out of a process divided by the number of units going into that process
over a specified period of time. Only good units with no rework or scrap are counted as coming
out of an individual process.
First Pass Yield and Throughput Yield are synonymous terms that define the number of units
successfully produced in a process step, i.e., without defects, divided by the total number of
units entering that process. For example, of 10 units entering the stamping process to yield
correct dimensions, 6 were defect-free products, or 6/10. To count as FPY/TPY units, they must
pass completely through the process without rework or scrap. The formula is simple:

The First Pass Yield or Throughput Yield of the stamping process is 0.60, or 60 percent.

FPY / TPY = UNITS DEFECT−FREE UNITS


ENTERED PROCESS

FPY/TPY = 6/10

= 0.60

First Time Yield (FTY) is a calculation used to determine when the total number of
acceptable product leaves the process when rework/scrap are counted, but only if
the rework performed corrects the defect(s). Let’s assume that of the 5 treatment
(anodizing) defects found, 4 are able to be reworked.
This increases the number of acceptable units through that process by 4 units, or 7
acceptable units total. The formula is:

It is important to include only the successfully reworked product in the formula.


The rework resulted in a First Time Yield of 0.70, or 70 percent.

Rolled Throughput Yield (RTY) :Rolled throughput yield (RTY) is the probability that
a process with more than one step will produce a defect free unit. It is the product of
yields for each process step of the entire process.is an accumulated total, by
multiplication, of the results of all FPY/FTY-measured processes. For any process, it is
ideal for that process to produce its product without defects and without rework.

For example, we have four processes that produce a finished part: material,
stamping, anodizing and properties.
In the materials (verification) process, there are 2 failures in 10 units
(FPY = 80 percent). In the stamping process, there are 4 failures in 10
units (FPY = 60 percent).
In the anodizing process, there are 5 failures in 10 units (FPY = 50 percent).
In the properties (verification) process, there are 3 failures in 10 units (FPY
= 70 percent). The formula is:

Our example indicates that the Rolled Throughput Yield (RTY) for all processes is only 16.8 percent.

Cycle Time, Manufacturing Lead Time, and TAKT Time


These three metrics are also critical to lean six sigma.

Cycle Time is simply the time from beginning to completion of a process step. In our example, it
might be the elapsed time in the stamping process from the retrieval of a raw material aluminum
bar from an input bin, loading it into the press, cycling the press, removing the stamped part, and
placing it into an output bin. Each of these segments can be measured separately, or the
cumulative of all segments together, but they are timed by a stopwatch and recorded accordingly.
Several measurements should be taken to arrive at an average cycle time for the process.

Manufacturing Lead Time is the measure of the total time to manufacture a part through its
complete value stream, or entire manufacturing process. In our example, this would include both
value-added and non-value-added time in process, calculated from the point of retrieval of raw
material from the supplier delivery dock to the point of loading finished goods into a shipping
truck. The stamping process noted above is just one segment of the value stream, which could
even include the transport time of a single, or batch, of components from one internal process to
the next, as well as delay time in storage between active process steps.

It is important to separate value-added activity (that is, any process activity that adds value to the
in- process part) and non-value added activity (that is, any process activity that does not add
value, such as transport and storage). For a more complete value stream, the order process, and
delivery to the customer’s dock may be included.

TAKT Time (takt is German, meaning “pace,” or “rhythm”) is the calculated rate required to
produce product within the time demanded by a customer order.5 If our L-bracket customer has
ordered 10,000 brackets to be delivered in daily increments of 500, and we know we have 8
hour(480 minutes) to produce
each daily batch, the formula to
use is:
We have less than one minute of takt time to meet the customer demand. If it seems impossible
to fully produce one bracket per minute (let alone less), we must recall that a series of processes
like this example can produce multiple units in a batch, so the takt time is achievable.

1.2 The Fundamentals of Six Sigma


1.2.1Defining a Process

The six sigma philosophy maintains that reducing “variation” will help solve process and
business problems. The fundamental principle on which six sigma methodology is built is the
Process Approach.
Every organization is made up of a series of interacting processes. A process is a set of
activities that uses resources (people, machines, etc.) to transform inputs into outputs. The
process approach considers the interaction between these processes, and the inputs and
outputs that tie these processes together. The output of one process becomes the input of
another. A process is a set of activities done in order to convert the inputs into outputs.
Process can be subdivided into sub processes. All the process problems (manufacturing /
services / software development) can be solved by analyzing problems in the inputs and the
process conditions.
Process Analysis
Processes are rarely designed intentionally and they all evolve overtime. Throughout this
evolution, inefficiency, redundancy and waste creep in. It is critical to avoid finding fault with
a specific process: recognize that people and departments make changes in an effort to get
the job done, often without knowledge of how those changes might impact upstream and
downstream processes. The performance of individuals is only as good as the process will
allow it to be. Processes, especially cross-functional business practices, are usually not
documented, not standardized, not measured, not systematically and continually improved
and not managed by the micro-process doer or owner.

Most businesses, at some time or another, need to stop and examine their processes to identify and
eliminate waste and inefficiencies.
By using a set of statistical tools to understand the fluctuation of a process, management can
begin to predict the expected outcome of that process. If the outcome is not satisfactory,
associated tools can be used to further understand the elements influencing the process.
These efforts seek to eliminate the causes of variation in processes while leaving the basic
process intact. In six sigma terms, process improvement teams find the critical “Xs”(causes)
that create unwanted “Ys”(defects).

A good definition of a process is the most important step towards an efficient process structure.
Here are some more specific basics that can help when defining processes.
Eliminate waste—unnecessary steps, too many reviews, create from scratch when not necessary, etc.
Insure all steps add value—if it doesn’t add value, why are you doing it? (This value could be for
reporting data, compliance or other required steps.)
Define tasks simply with one verb and one noun—“Throw the ball.” Often we create
compound tasks that should really be explicit in the process definition. Recognize milestones
that take the form of “Concept Complete”which is not a task, but a result.
Be explicit at intake—It’s important that the complexity of the process match the job. We often
define processes based on doing new creative and forget that 80-90% of our work is modifying
existing pieces.
Be explicit at intake (Part 2)—Make sure all the information needed to move forward is defined
up front before starting work. Often, lots of time is wasted in going back to the client to gather
additional information or to clarify points that should have been clear at intake.
Identify the value added for each task—This is usually easy, such as, ‘an image is resized’ or
‘web page is updated with new content.’ It can even be something like, ‘enter time in tracking
system.’ This last one may not add value to the project, but it does add value to the
organization.
Be sure each task has an owner—Particularly when there are collaborative steps like kick-off
meetings or ‘review with management,’ it’s easy to identify a process step without explicitly
identifying the owner. If there is no ownership, there is no accountability.
Be clear when collaboration is needed—Sometimes processes have parallel activities such as
‘write copy’ and ‘develop design.’ If these activities require collaboration between the
participants, call it out. You may avoid having the copy and design come together later and
finding out that the design assumed headlines that aren’t there, or the text is too long for the
allotted space. Then you re back to rework that could have been easily avoided.
Identify appropriate task durations—It’s important to develop task durations that are
specific to the deliverable. ‘Create Concept’ will take longer for a brochure than for an email.
Capture this in your processes and you will be able to estimate delivery times much more
accurately (and deliver on time more frequently).
Identify the right reviewers (and the wrong ones)—Look carefully at why each reviewer is
included for each type of project. It may be possible to eliminate unneeded reviews by
empowering staff to a greater degree and also by recognizing levels of strategic importance.
In other words, maybe the VP of marketing doesn’t need to sign off on everything going
through the department.

PROCESS MAPPING LEVELS


Level 1: The Macro Process Map, sometimes called a Management level or viewpoint.
Level 2: The process map,sometimes called the worker level or viewpoint. This example is from the perspective
of the pizza chef.

Process Modelling levels


• Level one: this very high level map outlines the operational levels of an organization

Pizza dough
No

Take order Place in Observe Check Remove if


Add 1
frequent if done done
from cashier ingredients oven
Yes

Start new
pizza

scrap
No

Pizza Put on delivery


Place in box Tape order on rack
1 correct
box
Yes

and are rarely, if ever, actually drawn. Examples include: customer processes,
administrative processes.
• Level two: shows end-to-end processes across the above operational areas. For
example a level two process for purchasing capital equipment would cross several
operational areas: the requesting department, purchasing, accounts payable, asset
management, receiving and biomedical/maintenance. Also called top down or high-
level process maps. They are quick and easy to draw, but may not provide the detail
necessary to build understanding or realize improvements.
• Level three: shows the roles, inputs, outputs and steps required to complete a specific
process within an operational area. For example, the purchasing process (request,
sourcing, cut PO) might be depicted as a process map.Also called cross- functional or
deployment diagrams. Usually contain enough information for improvement efforts,
but often miss inefficient details and don’t function well for training or as operational
documentation.
• Level four: is the documentation of systems, instructions and procedures required to
complete steps in the level three processes and shows inputs, outputs, associated
steps and decision points. For example, specific steps necessary to cut a PO in the
enterprise application would require a level four process map. The procedures and
system instructions can be represented as text, an algorithm or detailed process map.
Because of the level of detail, they can be resource-intensive to create, but offer the
greatest improvement potential. Since they illustrate decisions and subsequent
actions, they are excellent training and reference materials. But if your organization is
new to process mapping, use these sparingly.
The time and effort may turn stakeholders off before they’ve had a chance to
experience the benefits of the work.
Critical to Process (CTP) are the key process input variables. These are the process parameters
which influences other critical approaches – Critical to Quality (CTQ), Critical to Delivery (CTD)
and / or Critical to Cost (CTC).
To illustrate CTP in terms of process, we must mention it as the critical x ’s in the function -
> y = f(x). It is essential to know what the x’s are – for example, temperature, pressure,
humidity, but not essentially their definite setting or level. Every organization and business
processes has constraints. There are different forms of constraints. When a product or
service or its characteristics are not sufficient to fulfil the requirements of customers, the
projects of improvement should be implemented to correct the fault or lacuna identified in
the processes. They represent the necessities of quality but have deficiency in terms of
measuring the specificity.

▪ Critical to Delivery (CTD) : Critical to Delivery (CTD) characteristics are services to


manufactured goods, and/or transactional characteristics that significantly influence one
or more CTSs in terms of Delivery. It represents the customers who have stated needs in
relation to delivery. It is of late delivery. However, early delivery might create problem
for some of the customers as it may invite excess account requiring payment before the
need for such inventory arises. CTDs are transformed into critical requirements of the
customer through the quantification of these impact areas.

Critical to Cost (CTC):Critical to Cost (CTC) analysis is the input to the Quality Function
Deployment activity designed for the satisfaction of customers’ requirements. This is the
process of analyzing the inputs and outputs and find out the path that influence the cost
of process outputs. CTC are similar to CTQ but the former deals entirely with the impact
of cost on the customer. This is the way to compute numbers and an overall project.
1.2.2 Critical to Quality Characteristics (CTQ’s)

CTQ trees (critical-to-quality trees) are the key measurable characteristics of a product or
process whose performance standards or specification limits must be met in order to
satisfy the customer.
In other words, CTQs represent the product or service characteristics as defined by the customer/user
also known as Voice of Customer.
Voice of Customer (VOC ) to Critical to Quality (CTX )Conversion
Customer requirement / VoC is the data collected from customers that gives information
about what they need from the process. Customer requirements are often high-level, vague,
and not specific. Customer requirements when translated into Critical Process Requirements
that are specific and measurable are called Critical To X (CTX) factors.

A fully developed CTX has four elements:

• Output characteristic (CTX Name)


• Y metric (CTX Measure)
• Target Specification/tolerance limits
As discussed previously, Voice of Customer is the customer’s voice, expectations,
preferences, comments, of a product or service in discussion. It is the statement made by
the customer on a particular product or service.
Customer Needs and Requirements: Need is a desire or an expectation of a customer from
a given product or service. Customers may have many stated needs which are often vague
and generally are “wants” from a product/service. For example Customer requires an Air-
Conditioner for its use in his/her bed-room. The real need of the customer is cool
temperature in the bedroom.
Similarly, the real want from the Air-Conditioner is it has to be quite, cost-effective and
maintenance free. When the customer states his/her requirements, we will often get to hear
that the need is “Cool Temperature, Less Noise (quite), Cost Effective and Free of
Maintenance ”. However, “Cool Temperature” is the need and the rest are just wants. It is
important for the project team to understand the stated needs of the customer and separa
te them from wants.
The main reason that we separate needs and wants are: Needs are important critical features
and Wants are expectations of the product or service over and beyond the needs. If the
product/service is not able to fulfill “Wants” of the customer, the customer may only be
highly displeased / dissatisfied.
However, if the product/service is not able to fulfill “Needs” of the customer, he/she will not
use the product/service and there is a high likelihood that he/she will switch to competitor s
product/service. Organizations reputation may also be at stake if “Needs” are not met.
Requirement is an attribute of the product or service which fulfils the needs of a
customer. Customer defines these requirements and are the “musts” of a product or service.
For example in the above example of an air conditioner, customer’s requirement is “Cool
Temperature” and the rest are “Good to Have” features. Customer will not buy the Air-
Conditioner if all of “Good to Have” features are present but the “Requirement(s)” are not
fulfilled.
However, customer may buy the product/service if the “Requirement” is fulfilled and “Good to
Have” features are present or not present.
Translating Voices to Needs: Voice of Customer (VOC) methodology can be used to capture
the customer needs – both current (stated needs) and latent (unstated needs). VOC
methodology helps capture the needs of customer through stated verbatim comments
(customer voices). It helps translate verbatim comments (customer voices) into customer
needs to product/service output characteristics (customer requirements).
Translating Voices to Needs: Voice of Customer (VOC) methodology can be used to capture
the customer needs – both current (stated needs) and latent (unstated needs). VOC
methodology helps capture the needs of customer through stated verbatim comments
(customer voices). It helps translate verbatim comments (customer voices) into customer
needs to product/service output characteristics (customer requirements).
Example :
VoC: 'I have to wait for 48 hours to get a reply to a single

email‘ CTX Name: Responsiveness


CTX Measure: TAT (HH:MM)
CTX Specification: Less than 24 hours from the time an email drops into the mail box. Defect:
Number of emails the TAT has been violated for (E.g. 200 emails every day) Unit:
Emails (e.g. 300 emails) Opportunity: 1 Per Email
Graphical presentation of conversion :

Example of Translation of VOC to CTQ

1.2.3 Cost of Poor Quality (COPQ)

Cost of poor quality (COPQ) or poor quality costs(PQC), are costs that would disappear if
systems, processes, and products were perfect. It quantifies the negative outcomes due to
waste, inefficiencies and defects in a process. It is a methodology that allows an organization
to determine the extent to which its resources are used for activities that prevent poor
quality, that appraise the quality of the organization's products or services, and that result
from internal and external failures. COPQ was popularized by IBM quality expert H. James
Harrington in his 1987 book ‘Poor Quality Costs’.

Joseph Juran separated these Cost of poor quality (COPQ) costs into 3 categories:
1. Prevention Costs: costs incurred to prevent errors / failures Including quality planning, training,
preventive maintenance, housekeeping etc.

2. Appraisal Costs: costs incurred to determine the degree of conformance to quality requirements
Including testing, inspection, audits, reviews and surveys, etc.

3. Failure Costs:

➢ Internal Failures: associated with defects found before the customer receives the product
or serviceIncluding scrap, rework, expediting, equipment downtime, injuries, etc.

➢ External Failures: Including product recalls ,returned products, complaint handling, lost
sales, lost reputation etc.
Cost of Poor Quality can be calculated as the sum of all the above costs.
For example, let’s considern inventory handling. If you have a lot of inventory (in lean,
inventory is considered waste), then you will have material handlers that are needed to
move the inventory along.
For example, sales people can enter the order wrong, can contribute to lost sales with a
faulty sales process, or spend too much time with the wrong customers.
COPQ is often represented as a dollar amount (or as a percentage of sales) and is
composed of the categories listed above. The goal of any organization is to reduce COPQ
to zero.
1.2.4 Pareto Analysis (80:20 rule)

Pareto Analysis is a statistical technique in decision-making used for the selection of a limited
number of tasks that produce significant overall effect.
The Pareto principle (also known as the 80/20 rule, the law of the vital few, or the principle of
factor sparsity) states that, for many events,the principle dictates that 80% of the failures are
coming from 20% of the causes.
As a tool in Six Sigma, Pareto is part of the quality control tools that are derived from historical
data in order to come up with efficient and most appropriate actions to address most common
and impacting failures. With the use of Pareto, scarce resources are efficiently allocated. Pareto
analysis only applies to historical data and will not provide forecast analysis. Another limitation
of the Pareto is that it relies only on the data which it is based on.
Pareto diagrams are very specialized forms of column graphs. They are used to prioritize
problems (or opportunities) so that the major problems (or opportunities) can be identified.
Pareto diagrams can help six sigma teams get a clear picture of where the greatest contribution
can be made.

Pareto diagrams are used to:


• Analyze a problem from a new perspective

• Focus attention on problems in priority order

• Compare data changes during different time periods

• Provide a basis for the construction of a cumulative line

Pareto Chart - Example - Issue Management in a software company

Why a high number of tickets are raised in the issue management

tool? The process improvement team chose to study

the data for 90 days .

Data is summarized in terms of the % of the total for each category of cause and their cumulative %.

S.No. Causes / defects Frequency

1 Mouse issues 272

2 Keyboard issues 186

3 Unable to install software 127

4 Virus issues 99
5 Intranet connectivity 19

6 Internet Connectivity 12

7 Software gets corrupted 3

Total 718

Below is an example of a Pareto chart:


To further illustrate, the Pareto chart has been designed as a visual representation of the vital
few against the trivial many. With the help of the chart, it is easy to identify the causes of most
of the problems. Below is an example of a Pareto chart:

Let’s take the food service setting as an example to illustrate the Pareto principle. We will base
our study on data gathered from customers’ comments on the survey sheets provided to them.
We will use the information below:
Pareto Exercise Chart Food Serving Survey

The survey yielded 100 counts of dissatisfied customers and their reason for the dissatisfaction.
The first step in using the Pareto Tool is to rank the failures according to their Count (eg. The
frequency of occurrence) from highest to lowest:

Pareto Exercise Chart Food Serving Survey Step 1 Rank Failures

The second step is to compute the percent distribution of each failure. We do this by dividing
the count of each failure by the total number of failures, then multiplying by a factor of 100. For
example our biggest failure in this imaginary survey is “Food Presentation” where a total of 25
failures were accumulated. We’ll divide this by the total number of failures, which is 100, then
we multiply this by 100 to get the actual percentage – so in the end the corresponding
contribution of “Food Presentation” to the overall failures is 25%:
Pareto Exercise Chart Food Serving Survey Step 2 Calculate Percentile

In the third step, we’ll compute the cumulative contributions of the failures in order of their
rank. To do this just add the contribution of the preceding failure to the next (in percentage) like
this:

Pareto Exercise Chart Food Serving Survey Step 3 Calculate Cumulative Contributions

Now you are ready to construct your Pareto Chart. You will need the failures, count and
cumulative contribution. Highlight these variables, then using Excel format go to the “Insert” tab
and click on Column chart. It should look like this:
Pareto Exercise Chart Food Serving Survey Excel Example

Click and then right-click on “Cumulative Contribution”, under “Format Data Series” choose secondary
axis. Your table should appear as below:

Pareto Exercise Chart Food Serving Survey Excel Example Sorted By Cumulative Contributions

Click on the “Cumulative Contribution” again to change chart type from bar to line chart to complete
the Pareto chart:
Pareto Exercise Chart Food Serving Survey Example Sorted By Cumulative Contributions Line

Chart Pareto Analysis

Using the data above, we noticed that the 80% contribution lies somewhere in between “Food
Presentation” and “Overall Taste” or “Store Ambience” with the first 2-3 factors having the
biggest weight. This is the Pareto principle of unequal distribution of causes and effects – 80% of
the problems are caused by 50% of the factors.

Of course in reality it doesn’t have to be a perfect 80% / 20% distribution, and don’t get caught
in the 80/20 stigma of thinking that it must always equal 100%. In reality you may see number
such as 50% of market share being controlled by 1% of the market players, or 90% of the
problems being contributed by 30% of the factors.The conclusion from this example is that the
store can focus its scarce resources in addressing the top four reasons for dissatisfaction which
are:

“Food Presentation” – 25%

“Quality of Service” – 24%

“Cleanliness” – 15%, and “Overall

Taste” – 12%

and effectively resolves 80% of the problem of dissatisfied customers.

Without the Pareto, the business entity might be spreading out their resources in trying to
address all of the reasons of dissatisfaction. Or they might be concentrating on minor issues like
“Serving Size” (5%) and “Promptness” (5%) which don’t really have a strong impact compared
with “Food Presentation.”
From the customers’ point of view, based on the data, they don’t mind waiting a bit, getting
smaller portions and paying more. But, they want their food nicely presented – which might call
for better plates and platters, waiters and service crew to be nicer and a spic and span
surroundings.

1.2.5 Basic Six Sigma Metrics

Primary Project Metrics: Metrics are needed to ensure that the requirements for the project
are measurable and therefore controlled during the project. Primary Metrics are developed in
the Define phase of the project, but are not finalized until the Measure phase.

The primary metrics for consideration in the project come from various sources like
➢ Suppliers

➢ Internal Processes ➢ Customers

➢ Examples of the Primary metrics for the project are

➢ Quality

➢ Cycle Time

➢ Cost

➢ Value ➢ Labor

Examples of the Primary metrics for the project are :

➢ Quality

➢ Cycle Time

➢ Cost

➢ Value ➢ Labor

Secondary Project Metrics: The secondary metrics for the project are derived from the primary
metrics. These are usually numerical representations of the primary metrics.

Some examples of secondary metrics include

➢ Defects per Unit (DPU)

➢ Defect per Million Opportunities (DPMO)

➢ Average Duration of Receivables

➢ Lines of Error Free Software Code


➢ Scrap %
Before discussing various metrics, it is important to understand unit and opportunity.

➢ Unit: The event/transaction produced or processed.

➢ Opportunity: Any inputs to event/transaction that can be measured to assess the


probability of it not meeting a Customer Requirement. In other words, it is the
expression of the probability of the occurrence of an error.

Defects per Unit (DPU)


One way to understand how well a process performs is to identify the resulting Defects per Unit
(i.e. the number of defects per manufactured unit).
For exp: The chart identifies 10 total units produced. Of them, 7 are defective and there are 14 total
defects. Note that almost all units have multiple defects.
It is important to distinguish between these two concepts. A defect is an occurrence of non-
conformance to customer requirements. A defective unit is a product with one or more defects
or non-conformant features. It is possible for a single product to have multiple defects.

The formula for Defects per Unit is:


𝑇𝑇𝑇𝑇𝑇𝑇𝑇𝑇𝑇𝑇𝑇𝑇𝑇𝑇𝑇𝑇𝑇𝑇𝑇𝑇 #𝐷𝐷𝐷𝐷𝐷𝐷𝐷𝐷𝐷𝐷𝐷𝐷𝐷𝐷𝐷𝐷𝐷𝐷𝐷𝐷𝑇𝑇𝑇𝑇𝐷𝐷𝐷𝐷 𝐷𝐷𝐷𝐷
=
DPU =
𝑇𝑇𝑇𝑇𝑇𝑇𝑇𝑇𝑇𝑇𝑇𝑇𝑇𝑇𝑇𝑇𝑇𝑇𝑇𝑇 #𝑈𝑈𝑈𝑈𝑈𝑈𝑈𝑈𝑈𝑈𝑈𝑈𝑇𝑇𝑇𝑇𝐷𝐷𝐷𝐷 𝑈𝑈𝑈𝑈
= 14
= 1.4
10
There are 1.4 Defects per Unit in this process

Defects per Opportunity (DPO)


Defects per Opportunity describe the average defects (opportunities for defects) in each unit
produced. This is not a hard count of the number of defects, but a measure of the probability of
defects. DPO may be expressed by three separate formulae using similar data. They each depend
on the data collected from the chart above:

D = # defects (14) TOP = total # of failure opportunities

(40) O = # defect opportunities (4) DPU = defects per unit (1.4)

U = # units (10)

DPU = 0.35
DPO = = 1.4
O
4
DPO = D
10 4
DPO = UO 14 = 0.35
=
D 40
TOP

Either formula, using different data points, gives the same result for this process: the average number
of defects, or the probability of defects, in each produced unit is 0.35 DPO.
Defects per Million Opportunities (DPMO)
Defects per million opportunities (DPMO) is also a probability measure. It calculates the
probability that a given process will produce X defect opportunities per million units
produced. One million units is used in the formula as a convenient constant, regardless of the
actual number of units a process produces.
The formula is:

6
DPMO = DPO x 10

DPMO = 0.35 x 106 = 350,000

This formula uses the data from the previous metric, DPO. It differs from a calculation of
(defective) Parts per Million (PPM) (will be discussed below) in that it accounts for the fact that
there are multiple failure opportunities in a single manufactured unit. One should not assume
that those opportunities automatically become actual failures. For the metric, DPMO, using the
data from the chart above, there is a probability that there are 350,000 defect opportunities.

Parts per Million (PPM)


PPM counts the quantity of defective parts per million parts produced. As noted above, it
does not account for the fact that multiple defects may affect a single part. One defective
part, even with multiple defects, counts as a single defective among other defectives in the
population. As with DPMO, it uses 1,000,000 as a constant, regardless of the actual number of
parts produced.
The formula is: PPM = Defectives percentage of a
population x 106 PPM = 0.70 x 1,000,000 = 70,000

In our process example, 7 defective units (count the number of defectives, not the number of defects)
of 10 units produced yields a PPM of 70,000.

First Pass Yield

• First Time Yield(Yft) - units passed/ Units input for First


time

Rolled throughput Yield

• Rolled Throughput Yield (Ytp) - Yield1*Yield2*Yield3*Yield4

Cycle Time

• Cycle Time = 1/Throughput Rate

First Pass Yield :First pass yield (FPY), also known as throughput yield (TPY), is defined as the
number of units coming out of a process divided by the number of units going into that process
over a specified period of time. Only good units with no rework or scrap are counted as coming
out of an individual process.

First Pass Yield and Throughput Yield are synonymous terms that define the number of units
successfully produced in a process step, i.e., without defects, divided by the total number of
units entering that process. For example, of 10 units entering the stamping process to yield
correct dimensions, 6 were defect-free products, or 6/10. To count as FPY/TPY units, they must
pass completely through the process without rework or scrap.
The formula is simple:
The First Pass Yield or Throughput Yield of the stamping process is 0.60, or 60 percent.

FPY / TPY = UNITS DEFECT−FREE UNITS


ENTERED PROCESS

FPY/TPY = 6/10

= 0.60

First Time Yield (FTY) is a calculation used to determine when the total number of acceptable

product leaves the process when rework/scrap are counted, but only if the rework performed

corrects the defect(s). Let’s assume that of the 5 treatment (anodizing) defects found, 4 are able

to be reworked.

This increases the number of acceptable units through that process by 4 units, or 7 acceptable units
total. The formula is:

It is important to include only the successfully reworked product in the formula. The rework resulted
in a First Time Yield of 0.70, or 70 percent.

Rolled Throughput Yield (RTY) :Rolled throughput yield (RTY) is the probability that a process
with more than one step will produce a defect free unit. It is the product of yields for each
process step of the entire process.is an accumulated total, by multiplication, of the results of all
FPY/FTY-measured processes. For any process, it is ideal for that process to produce its product
without defects and without rework.

For example, we have four processes that produce a finished part: material, stamping, anodizing
and properties.

In the materials (verification) process, there are 2 failures in 10 units (FPY = 80

percent). In the stamping process, there are 4 failures in 10 units (FPY = 60 percent).

In the anodizing process, there are 5 failures in 10 units (FPY = 50 percent).


In the properties (verification) process, there are 3 failures in 10 units (FPY = 70

percent). The formula is:

Our example indicates that the Rolled Throughput Yield (RTY) for all processes is only 16.8 percent.

Cycle Time, Manufacturing Lead Time, and TAKT Time These


three metrics are also critical to lean six sigma.
Cycle Time is simply the time from beginning to completion of a process step. In our example,
it might be the elapsed time in the stamping process from the retrieval of a raw material
aluminum bar from an input bin, loading it into the press, cycling the press, removing the
stamped part, and placing it into an output bin. Each of these segments can be measured
separately, or the cumulative of all segments together, but they are timed by a stopwatch and
recorded accordingly. Several measurements should be taken to arrive at an average cycle time
for the process.

Manufacturing Lead Time is the measure of the total time to manufacture a part through its
complete value stream, or entire manufacturing process. In our example, this would include
both value-added and non-value-added time in process, calculated from the point of retrieval of
raw material from the supplier delivery dock to the point of loading finished goods into a
shipping truck. The stamping process noted above is just one segment of the value stream,
which could even include the transport time of a single, or batch, of components from one
internal process to the next, as well as delay time in storage between active process steps.

It is important to separate value-added activity (that is, any process activity that adds value to
the in- process part) and non-value added activity (that is, any process activity that does not add
value, such as transport and storage). For a more complete value stream, the order process, and
delivery to the customer’s dock may be included.

TAKT Time (takt is German, meaning “pace,” or “rhythm”) is the calculated rate required to
produce product within the time demanded by a customer order.5 If our L-bracket customer
has ordered 10,000 brackets to be delivered in daily increments of 500, and we know we have 8
hours (480 minutes) to produce each daily batch, the formula to use is:

#𝐩𝐩𝐩𝐩𝐩𝐩𝐩𝐩𝐩𝐩𝐩𝐩𝐩𝐩𝐩𝐩𝐩𝐩𝐩𝐩𝐩𝐩𝐩𝐩𝐩𝐩𝐩𝐩𝐩𝐩𝐩𝐩𝐩𝐩𝐩𝐩𝐩𝐩 𝐩𝐩
𝐦𝐦𝐦𝐦𝐩𝐩𝐩𝐩𝐩𝐩𝐩𝐩𝐩𝐩𝐩𝐩𝐩𝐩𝐩𝐩𝐦𝐦𝐦𝐦𝐦𝐦 𝐦𝐦
𝐚𝐚𝐚𝐚𝐚𝐚𝐚𝐚𝐚𝐚𝐚𝐚𝐩𝐩𝐩𝐩𝐚𝐚𝐚𝐚𝐚𝐚𝐚𝐚𝐚𝐚𝐚𝐚𝐚𝐚𝐚𝐚𝐦𝐦 𝐦𝐦
Takt time =
#𝐩𝐩𝐩𝐩𝐩𝐩𝐩𝐩𝐩𝐩𝐩𝐩𝐩𝐩𝐩𝐩𝐩𝐩𝐩𝐩𝐩𝐩𝐩𝐩𝐩𝐩𝐩𝐩𝐦𝐦𝐦𝐦
𝐩𝐩𝐩𝐩𝐩𝐩𝐩𝐩𝐩𝐩𝐩𝐩𝐦𝐦𝐦𝐦𝐩𝐩𝐩𝐩𝐦𝐦𝐦𝐦𝐩𝐩𝐩𝐩
𝟒𝟒𝟒𝟒𝟒𝟒𝟒𝟒𝟒𝟒𝟒𝟒
Takt time = = 0.96 minutes
𝟓𝟓𝟓𝟓𝟒𝟒𝟒𝟒𝟒𝟒𝟒𝟒

We have less than one minute of takt time to meet the customer demand. If it seems impossible
to fully produce one bracket per minute (let alone less), we must recall that a series of
processes like this example can produce multiple units in a batch, so the takt time is achievable.
1.3 Selecting Lean Six Sigma Projects

1.3.1 Building a Business Case & Project Charter

The Project Charter is the first step in a Lean Six Sigma project, and therefore takes
place in the Define phase of DMAIC.A charter is a written document that defines the
team’s mission, scope of operation, objectives, time frames, and consequences for the
lean six sigma project. It is a living document that outlines a process improvement
project for both the team as well as leadership.
The Project Charter is periodically reviewed and refined throughout the project. A
critical element in the establishment of an improvement team is the development and
acceptance of a charter.

Charters can be developed by top management and presented to teams or teams can
create their own charters and present them to top management. Either way, top
management’s endorsement of a team’s charter is a critical factor in giving the team
the direction and support it needs to succeed. The charter should also identify the
objectives the team is expected to achieve.
Objectives should always be stated in measurable terms.

The charter should also define the operating scope. This is an opportunity to identify
the organizational or operational boundaries within which the team is expected and
permitted
to operate. Defining boundaries is crucial to avoid energy draining and time delaying turf
wars.
Teams need to know what top management expects of them. The team has the
authority, permission, and blessing from the necessary levels of management to
operate, conduct research, consider and implement any changes needed to achieve
the expected project results.
The charter begins with a purpose statement. This is a one or two line statement
explaining why the team is being formed. The purpose statement should align with,
and support, the organizations vision and mission statements.
A charter provides the following advantages
A Project charter answers the following questions about the project:

▪ What are the benefits of this project?

▪ Who will be impacted by the project?

▪ How will we know the project is successful?


Team project charter should contain the following key points:

Business case (financial impact): The business case is a short summary of the strategic
reasons for the project.
The business case is a short summary of the strategic reasons for the project. The
general rationale for a business case would normally involve quality, cost, for delivery
of a product with a financial justification. There are four basic activities:

• Design of a new product

• Redesign of an existing product

• Design of a new process

• Redesign of an existing process


A common problem for many projects is the lack of a company impact measurement.
For example, if the existing quality defective rate is at 5,000 defectives per million
opportunities; the possible justification is a reduction to 250 defectives per million
opportunities with a cost savings of $1,000,000.
Another example would be reduction of product cycle time from 6 weeks to 5
days for an additional 10,000 units of production, resulting in an additional
$1,000,000 of revenues.
A project improvement team should follow typical financial department justification
guidelines. The advantages and disadvantages of a project should be researched.
Other individuals or departments should be involved, if necessary, to examine the
key costs and resources for a successful project. Projects which do not show a
significant financial impact to the company should be stopped or eliminated as soon
as possible.

Problem statement:
▪ A problem statement details the issue that the team wants to improve.

▪ The problem statement should be crafted to be as descriptive as possible. That is,


how long has the problem existed, what measurable item is affected, what is the
business impact, and what is the performance gap.

▪ The problem statement should be neutral, to avoid jumping to conclusions. A

sample problem statement might be:

“The ABC Company, in 2007, has experienced a 25% drop in sales, with a 40% drop
in net profits.”
The problem statement should include a reference to a baseline measure for
guidance. A baseline measure is the level of performance of the particular metric at
the initial start of a project.

Project scope (boundaries): The project scope refers to the boundaries of the project.
The project scope refers to the boundaries of the project. It is an attempt to outline
the range of the team’s activities.
For example, in the area of product development, the team may decide to limit itself
to the launching of a new product at a single manufacturing site. In this case issues
or problems regarding market research, prototype development, or financial
investments would be outside the scope of the team activities.
The team champion, the team leader, and the team -all need to be involved in this process.
Two main types of scoping:

▪ The Longitudinal Scope

▪ The Lateral Scope

Longitudinal scope refers to clear articulation of “Start” and “End” of the process. It
describes the Length of the process. For example, let’s say we have to work on
reducing the Average Handle Time of a call (transaction) for a given call center. The
longitudinal scope will be referred to:
Start: The time when the call center agent receives the
call. End: The time when the call is closed.
Similarly, the lateral scope refers to the width of the process.
For example, the Average Handle Time Reduction project for the call center is
only applicable for Pittsburgh, Virginia and New York locations in United States
and is not applicable in any other location across the country.

Goal statement: The goal statement is created and agreed to by the team and
team champion. The goal statement needs to be created and agreed to by the
team and team champion.
Hopefully, the goals will be achievable within a 120 to 160 day period.

A typical “rule of thumb” for six sigma goals is a requirement of a 50% reduction in
some initial metric (or improvement of 50%).
For example, reduce the collectibles from 120 days to 60 days; reduce the scrap from
5% to 2.5%.
Another example could be, “Goal is to reduce the average handle time from 7.16
minutes to less than 6 minutes in the next 3 months.”

• Role of team members: Description of various six sigma roles are included in the
description.

• Resources required: Calculation of resource requirement is done at the start of the


project.
The resources required for a project must be detailed. Typical resources required
for a project could include:

• Qualified people

• Machine time

• Equipment

• Phones and faxes

• Machinery

• Computer equipment

• Lab or office space

• Utilities, etc.
• Milestones/ deliverables (end products of the project): A set of milestones are
used to keep the project on track and to help bring a project to completion.

Identifying the above details, in written form, will provide a constant and consistent
target for the team.
Milestones/Deliverables

▪ For any well managed project, a set of stages or milestones are used to keep the
project on track and to help bring a project to completion.
▪ Initial team projects should be at the 120 day length.

▪ Only half of the project would be allocated to the define and measure stages.

▪ Assigning teams an initial project with lengths of more than 160 days will lower
the anticipated success rate.
In the sample milestones described in the diagram, the define phase gets over by
10th January, the Measure phase gets over by 30th January, the Analyze Phase gets
over by 15th March and Improve and Control phases each take a month to complete.
(Please note: The milestones could vary from project to project. This is just a sample
example).
Resources Required

The resources required for a project must be detailed. Typical resources required
for a project could include:
▪ Qualified people

▪ Machine time

▪ Equipment

▪ Phones and faxes

▪ Machinery

▪ Computer equipment
▪ Lab or office space

▪ Utilities, etc.

1.3.2 Developing Project Metrics

Key project measurements are also difficult to determine until the project selection
and charter processes are complete.
Careful selection of project measures ensure the overall success of the six sigma
project. Since most six sigma projects deal with time and money issues, most project
measures will also be about time and money or will be closely related to them. Project
measures provide the data and information needed to analyze, evaluate and improve
the business process as well as manage and evaluate, the impact of the six sigma
project. Project measures for both cost and revenue factors must be included.
After a detailed list of project activities has been created (along with the resource
requirements, and activity durations) the project budget is determined. During the
project, actual costs are collected and used as inputs to the revised estimated project
costs. The team leader or project manager compares the revised estimated costs
with the budgeted costs to monitor progress.
▪ The project budget must be reasonable, attainable, and based on estimates of
the tasks to be accomplished.
▪ Revenue factors included in the budget and analyses are:

▪ Income from additional sales generated as a result of improved product cost,


quality, features, availability to the customer
▪ Reduced losses for scrap, customer returns, warranty claims, cost of poor
quality (COPQ), low throughput, poor time to market

▪ Cost factors included in the budget are:


▪ Manpower and labor costs

▪ Materials

▪ Equipment costs, rentals, leases, etc.

▪ Subcontracted work or fees

▪ Overhead or consulting charges

▪ Reserve or contingency funds

Business level measures will likely be identified and provided by the executive
steering committee as part of the project selection process. Detailed operations
and process level measures should be chosen to support the business level
measures used.
Tollgate review

A Tollgate Review, as the name indicates, is like a checkpoint in a Six Sigma project at
which the various team members meet and determine whether the work has been
performed as indicated in the project plan and whether the objectives mentioned
have been achieved.Tollgate Reviews are normally carried out at the end of every
step in the DMAIC Process. That is, they will be carried out at the end of each of the
Define, Measure, Analyze, Improve and Control stages.
Tollgate Reviews help determine whether all the goals within each stage have been
achieved successfully and whether the project can progress to the next stage.
Who Performs Them?

The Tollgate Review in a Six Sigma Project is normally performed by a Master Black
Belt or by a Quality Council. However the Tollgate Review should also be attended
by Green Belts, Black Belts, the Project Champion or Sponsor and other members as
appropriate.
Tools Used

A variety of tools can be utilized in order to perform a Tollgate Review. Some of them
are listed below.

▪ Check Sheets

▪ Project Deliverables Document

▪ List of Milestones
Preparation Required

▪ Sometimes, the most conscientious project managers with the best of intentions
fail to successfully execute their Six Sigma projects because they are incapable of
conducting suitable Tollgate Reviews. This highlights the importance of pre- review
groundwork for the success of the project. In order to successfuly prepare for the
review make sure you:
▪ Ensure all required attendees are informed of the review.

▪ Block time in everyone’s work-day for the duration of the review.

▪ Prepare a suitable presentation consisting of check sheets, milestone lists,


etc. for review.

▪ Create a structured agenda for the presentation.

Project Charter – Assessment of Risks

In addition to a benefit cost analysis for a project, a decision to proceed must also
include an evaluation of the risks associated with the project. To manage project risks,
one should first identify and assess all potential risk areas. Risk areas include:
Business Risk:
▪ Technology changes

▪ Competitor actions

▪ Material shortages

▪ Health and safety issues

▪ Environmental issues

▪ Insurable Risks

▪ Property damage

▪ Indirect consequential loss

▪ Legal liabilities
▪ Personnel injury

Assessment of Risks

After the risk areas are identified, each is assigned a probability of occurrence and the
consequence of risk. The project risk factor is then the sum of the products of the
probability of occurrence and the consequence of risk.
Project Risk Factor = Summation of {(probability of occurrence) * (consequence of risk)}

Risk factors for several projects can be compared, if alternative projects are being
considered. Projects with lower risk factors are chosen in preference to projects with
higher risk factors.

1.3.3 Financial Evaluation & Benefits Capture

Six sigma is about making money. It is about profitability, although improved


quality and efficiency are immediate by products. The financial benefits of six
sigma projects are the measurements that create a link between philosophy and
action. Financial benefits and associated risks are the factors used to evaluate,
prioritize, select, and track all six sigma projects.
Cost-Benefit Analysis

Project cost-benefit analysis is a comparison to determine if a project will be (or was)


worthwhile. The analysis is normally performed prior to implementation of project
plans and is based on time-weighted estimates of costs and predicted value of
benefits. The cost-benefit analysis is used as a management tool to determine if
approval should be given for the project go-ahead. The actual data is analyzed from
an accounting perspective after the project is completed to quantify the financial
impact of the project.
The sequence for performing a cost-benefit analysis is:
▪ Identify the project benefits.

▪ Express the benefits in dollar amounts, timing, and duration.

▪ ldentify the project cost factors including materials, labor, and resources.

▪ Estimate the cost factors in terms of dollar amounts and expenditure period

▪ Calculate the net project gain (loss)

▪ Decide if the project should be implemented (prior to starting), or if the


project was beneficial (after completion)
▪ If the project is not beneficial using this analysis, but it is management’s desire
to implement the project, what changes in benefits and costs are possible to
improve the cost-benefit calculation?
Return On Assets (ROA) Return on Assets is equal to Net Income divided by Total Assets

Where the net income for a project is the expected earnings and total assets is the
value of the assets applied to the project. Additionally, a calculation of the return on
investment is widely used:

𝐍𝐍𝐍𝐍𝐍𝐍𝐍𝐍𝐍𝐍𝐍𝐍 𝐢𝐢𝐢𝐢𝐢𝐢𝐢𝐢𝐢𝐢𝐢𝐢𝐢𝐢𝐢𝐢𝐢𝐢𝐢𝐢𝐍𝐍𝐍𝐍
ROA =
𝐀𝐀𝐀𝐀𝐀𝐀𝐀𝐀𝐍𝐍𝐍𝐍𝐀𝐀𝐀𝐀𝐀𝐀𝐀𝐀𝐀𝐀𝐀𝐀𝐍𝐍𝐍𝐍 𝐓𝐓𝐓𝐓𝐢𝐢𝐢𝐢𝐍𝐍𝐍𝐍𝐀𝐀𝐀𝐀𝐓𝐓𝐓𝐓
𝐀𝐀𝐀𝐀𝐀𝐀𝐀𝐀𝐀𝐀𝐀𝐀𝐍𝐍𝐍𝐍𝐍𝐍𝐍𝐍𝐀𝐀𝐀𝐀

Example: For instance, if a company has $10,000 in total assets and generates $2,000
in net income, its ROA would be $2,000 / $10,000 = 0.2 or 20%.
In case we want to compare ROA of two companies.

Return on Investment: Where net income for a project is the expected earnings
and investment is the value of the investment in the project

Net Income Total


ROI= Assets

For example, if you invested $100 in a share of stock and its value rises to $110 by the
end of the fiscal year, the return on the investment is a healthy 10%, assuming no
dividends were paid.
Net Present Value: Net present value (NPV) of a project is the potential change in an
investor's wealth caused by that project while time value of money is being accounted
for. It equals the present value of net cash inflows generated by a project less the
initial investment on the project. It is one of the most reliable measures used in capital
budgeting because it accounts for time value of money by using discounted cash flows
in the calculation. NPV' is used since determining the value of a project is challenging
because there are different ways to measure the value of future cash flows. Because
of the time value of money, money in the present is worth more than the same
amount in the future.

Net Present Value =


Where n is the number of periods, t is the time period, r is the per period cost of
capital for the organization (also denoted as if annual interest rate is used) and CF,
is the cash flow in time period t. Note that CF, cash flow in period zero is also denoted
as the initial investment.
For Example : Calculate the net present value of a project which requires an initial
investment of $243,000 and it is expected to generate a cash inflow of $50,000 each
month for 12 months. Assume that the salvage value of the project is zero. The target
rate of return is 12% per annum.
Solution: We have,
Initial Investment = $243,000
Net Cash Inflow per Period =
$50,000 Number of Periods = 12
Discount Rate per Period = 12% ÷ 12 = 1%

Net Present Value


= $50,000 × (1 − (1 + 1%)^-12) ÷ 1% − $243,000
= $50,000 × (1 − 1.01^-12) ÷ 0.01 − $243,000
≈ $50,000 × (1 − 0.887449) ÷ 0.01 − $243,000
≈ $50,000 × 0.112551 ÷ 0.01 − $243,000
≈ $50,000 × 11.2551 − $243,000 ≈
$562,754 − $243,000
≈ $319,754

Payback Period

Payback period is the time in which the initial cash outflow of an investment is
expected to be recovered from the cash inflows generated by the investment. It is
one of the simplest investment appraisal techniques.
Formula: The formula to calculate payback period of a project depends on whether
the cash flow per period from the project is even or uneven. In case they are even,
the formula to calculate payback period is:

Payback Period =
Initial(8Z Incremental Inestment) Annual(or
Monthly)Cash Flow

Example: Company C is planning to undertake a project requiring initial investment


of $105 million. The project is expected to generate $25 million per year for 7 years.
Calculate the payback period of the project.
Solution
Payback Period = Initial Investment / Annual Cash Flow = $105M / $25M = 4.2 years
1.4 The Lean Enterprise
1.4.1 Understanding Lean

LEAN is a principle driven, tool based philosophy that focuses on eliminating waste so
that all activities/steps add value from the customer’s perspective.
The essence of Lean is to concentrate effort on removing variation from a process out-
put but not while lowering cost. The essence of Lean is to concentrate effort on
removing waste while improving process flow to achieve speed and agility at lower
cost.

• Lean removes many forms of waste so Six Sigma can focus on reducing variability
• Lean considers customer inputs and conducts a “war on waste.”

• In a layman’s language, Lean is “Elimination of Waste”.

• Lean focuses on the sequence of activities and work required to produce a product
or a service. This flow is called a Value Stream.

The cornerstones of a lean strategy are tools such as:

➢ Value stream mapping (VSM): Value stream map identifies Value


Added, Non-value added and Essential Non-Value Added activities
in a process. The non-value added activities are eliminated.

➢ Workplace organization (5S): 5S is a structured way o f


workplace organization. 5S helps in keeping the workplace spic
and span andthus helps in error free production.

➢ Total productive maintenance (TPM): Total Productive


Maintenance helps in ensuring the proactive maintenance of the
plant and machinery.

➢ Kanban/pull systems: Kanban is nothing but signaling cards.


Kanban system helps in using cards for raw material
replenishment.

➢ Kaizen: Kaizen is an small improvement in a process that helps in


improving the process or the function.
➢ Setup reduction: Suppose two different processes are using the
same equipment, in this case, when process one is complete, the
equipment should take least time to set up for the next process.
➢ Teamwork: Teamwork is the common and most important wayof
establishing and executing a project.

➢ Error-proofing: Error proofing is also known as Mistake- proofing


or Poka-Yoke. It helps in reducing inadvertent errors from
occurring.

➢ Problem solving: Problem solving is a tool commonly used to solve


minor problems with the help of process experts.

➢ Cellular manufacturing: Cellular manufacturing helps in ensuring


that the production is done in cells and is neatly tied for achieving
speed and agility.

➢ One-piece flow: One-piece flow is primarily used to avoid


significant wait time in the given process. One-piece flow helps in
removing unnecessary bottle-necks of waiting within the process.
Lean enterprise looks at the complete production system as outlined below:
A lean manufacturing cannot reach its full potential without taking into consideration
the complete system in its planning.
As given in the diagram, the complete production system starts Supply Chain and
parallelly requires Raw Materials, this gets processed and followed by that, gets
assembled. Post assembly, the products move to sales outlets and then finally to the
customer.
A true lean system is dependent on demand of its customers and reliability of

suppliers. Basically, Lean is nothing but systematic identification of waste and its

elimination.

It’s benefits include:


Lower production costs
Fewer Staff (personnel)
Quicker product delivery (lesser cycle time)
Higher quality
Higher profitability

Greater system flexibility

Cost
• Lean Producer: An Organization that uses Lean concepts
• 20th Centurty was known for Mass Production
Quality
Morale • One should use Lean terms effectively while training

Safety Delivery
Lean concepts

FIVE Areas that Drive Lean Producer


Mass production was recognized in 20th century. Similarly, Lean production is
recognized in 21st century.
Japanese terms are used in defining Lean principles to convey broad concepts such
as Poka Yoke, Jidoka and Kanban. However, one’s they are properly explained, these
terms can be more than just small mistake-proofing steps (Poka Yoke), automations
with human interventions (Jidoka) and those little cards which help control product
moves (Kanban). If these terms are not used effectively, it may have a negative effect
for different cultures across the globe. Hence, one should carefully choose the
training methods (and terms) for conveying lean tools and methods.
Use of Lean Techniques in Service

Manufacturing /

Production Sector:

Waste can be identified in manufacturing and production industry sectors and


Lean techniques are applicable in these sectors.
Service Sector:
Are Lean techniques applicable in a service oriented industry or office environment?
Every system contains waste. Whether a product is being produced or a material is
being processed or a service is being provided, we will always find elements which
could be termed as WASTE. The techniques for analyzing systems, identifying and
reducing waste, and focusing on the customer are applicable in any system, and in any
industry.
The business improvement methodology known as Lean Six Sigma is rooted in the
manufacturing industry, where it developed over the past few decades, reaching
widespread adoption worldwide.
However, according to the World Economic Outlook Database, published in April
2011, by the International Monetary Fund (IMF, 2011), the distribution of PPP
(Purchase Power Parity) GDP, in 2010, among various industry sectors in the main
worldwide economies, reflected a decline in the industrial sector, with the service
sector now representing three-quarters of the US economy and more than half of the
European economies.

Lean Six Sigma is now accepted widely as a business strategy to improve business
profitability and achieve service excellence, and its use in service organizations is
growing quickly. However, there are a number of barriers to the implementation of
Lean Six Sigma in services, such as the innate characteristics of services, as well as the
manufacturing origins of Lean Six Sigma that have conditioned service managers to
consider them as physical products only. Overall, the applications so far have showed
the benefits (such as lowering operational costs, improving processes quality,
increasing efficiency) to outweigh the costs associated with its implementation.
1.4.2 History of Lean – Lean Pioneers
The first person who integrated the concept of lean in the manufacturing system was
Henry Ford.

Further, in 1799, Eli Whitney came with the concept of interchangeable parts.
Further, in 1913, Henry Ford propounded the flow of production by experimenting
with interchanging and movement of different parts so as to achieve standardization
of work. However, there was a limitation to the Ford’s system that it lacked variety
and was applied to only one specification.

It was only after the World War II in 1930s, Toyota inspired from the Ford’s flow of
production concept and invented the Toyota Production System. The premise of this
new system is to change the focus from the use and utilization of individual machines
to the work flow from the total process. The Toyota Product system aims at reducing
the cost of production, enhancing the quality of products and increasing the
throughput times so that the dynamic customer needs are met. Some of the steps that
the system incorporated are the sight- sizing of the machines keeping in consideration
the needed volume of production, self- regulating features of machine so that the
quality of the manufactured products is enhanced, sequencing the machines as per the
process, develop quick steps so that the manufacturing of multiple parts in
comparative small volume becomes possible and keeping a strong communication of
the requirements of the parts between the steps of the process.

Chief Architects of Lean

The concept of lean management was continuously revived with the changing
times and needs of the industry. With this dynamism in the industrial
environment, a number of proponents of lean manufacturing made significant
contributions in the field of lean management.

Frederick Taylor

Early in the 1890s, the father of scientific management, Frederick Taylor investigated
closely in the work methods and workers at the factory level. After his supervision, he
propounded the concepts like standardization of work, time studies and motion
studies, in order to achieve efficiency in the work methods, processes and operations.
However, he ignored the behavioral aspect of the work, which invited many criticisms
against him.

•The Principles of Scientific Management: Outlines the foundation for modern


organization and decision theory. He does this through describing the
dilemma: workers harbor fears that higher individual productivity will
eventually lead to fewer jobs. Taylor’s suggestion is to deride this fear by
providing incentives for workers and re-framing the consumer, shareholder,
and worker relationship. A great and informative read for anyone interested in
efficient management practices.
Henry Ford

Starting in 1910, Henry Ford pioneered the famous manufacturing strategy, in which
all the resources used at the manufacturing site- people, machines, equipment, tools
and products were arranged in such a manner that a continuous flow of production
is facilitated. He has attained immense success by this process in manufacturing the
Model T automobile, and even became the richest man in the world. However, as the
world began to change, Ford could not change the work methods and failed when the
demand of the market was to add new models, colors and varieties to the products.
Finally, during 1920s the labor unions and the product proliferation ate away the
success of Ford, and it was by mid 1930s that General Motors dominated the
automobile market.

Sakichi Toyoda

Sakichi Toyoda established the Toyoda spinning and weaving company in the year
1918. Sakichi Toyoda was one of the initial contributors of success towards the famous
Toyota Production system that aims at eliminating all the wastes, by propounding the
Jikoda concept. Jikoda- ‘automation with a human touch’ means to facilitate quality at
source. He invented the automatic loom in 1896 that not only substituted the manual
work but also installed the capability to make judgments into the machine itself. The
system enhanced the work effectiveness and efficiency by mitigating the products
defects and associated wasteful work practices. The principle was jikoda leads to an
early detection of an abnormality, easy stopping of the machine or process on the
detection of the issue, immediate fixation of the abnormality and even helped to
investigate the root cause of the issue.

Kiichiro Toyota

Kiichiro Toyota was the founder and second president of Toyota Motor Corporation.
He was the son of Sakichi Toyoda and later in 1937, he founded the Toyota Motor
Corporation. He took forward his father’s concept of Jikoda and developed his
philosophy about just-in-time (JIT) concept in manufacturing. He paid to visit to the
Ford’s plant in Michigan to understand the flow of assembly line concept and then
proposed the Toyota Production system. The new system aimed at right sizing the
machines with respect to the actual volume needed and introduced mistake- proofing
so that quality is ensured and proper sequencing of work processes are done.

Taichi Ohno

One of the biggest achievements of Taichi Ohno was to integrate the Just-in-Time
system with the Jikoda System. After his visit to America to study the Ford’s methods
in 1953, he got highly inspired and understood the future needs of the consumers that
they will select the needed products from the shelves and how the products
replenished. This has inspired him to build the successful kanban system. He even
practiced the Dr. Edwards Deming method to incorporate quality at each step of the
process from design to aftersales services to the consumers. This was practiced and
brought at the floor level by Ohno who integrated this philosophy with the Kiirocho’s
just-in-time concept and principle of Kaizen. He is considered as the true architect of
the ‘Toyota Production System’.
The Toyota Production system relies on a number of concepts, that are pull system,
elimination of waste, Quick Die changes (SMED), non- value added work, U-shaped
cells and one piece flow. The pull systemdefines the flow the material in between
different processes as determined by the needs of the customers. The company
practices the pull system by using a Kanban system, which provides a signal to the
customer that the tools are available to be moved to the next process in the
sequence.
The Toyota Production System also identifies the waste, termed as Muda, and
recognizes that waste is anything that does not add value to the customers. Wastes
are of seven types; that are over-production, inventory waste, defects, waiting,
motion, over-processing and transportation and handling. The system aims at
identifying and eliminating these wastes so as to foster efficiency and effectiveness
in the production system.

Another method adopted by the company is the Quick die changes (Single Minute
Exchange of Dies) which aims at improving the flow (Mura) of production. The concept
is based on the premise that the tools and changeovers should take less than one
minute (single digit) at the maximum. During 1950s and 1960s, the company suffered
from the presence of bottlenecks at the car body molding presses. The root cause was
identified to be the high changeover times that increased the lot size of the production
process and drives up the production cost. Toyota implemented the SMED by placing
the precision measurement devices for the transfer of heavy weight dies on large
transfer stamping machines used to produce the body of the vehicles.

Further, the long production lines at Toyota were wrapped using U-shaped cells
layout that facilitates lean manufacturing. This increases the efficiency of the workers
to operate at multiple machines at a time. The Toyota Production System (TPS)
practices the one – piece flow whereby the need of the subsequent process is
determined by the previous operation. In other words, one piece is produced at a
time opposed to the mass production. Toyota places a single piece between different
workstations with the advantage of least variance in cycle time and minimum waiting
time. This would help to facilitate optimum balance between different operations and
mitigate over- production- which is considered one of the major wastes.

• The Toyota Way: In factories around the world, Toyota consistently makes the
highest-quality cars with the fewest defects of any competing manufacturer,
while using fewer man-hours, less on-hand inventory, and half the floor space
of its competitors. The Toyota Way is the first book for a general audience that
explains the management principles and business philosophy behind Toyota’s
worldwide reputation for quality and reliability.

Shigeo Shingo

Dr. Shigeo Shingo was an industrialist engineer, and a major consultant at Toyota,
that successfully helped the company achieve lean manufacturing. He mastered the
Kaizen concept. He understood the success of lean manufacturing by integrating the
people with effective and efficient processes. In 1960, he developed the SMED
system with the aim to achieve zero quality defects.
1.4.3 Lean and Six Sigma
There is an ongoing debate in some organizations regarding the difference between
lean and six sigma, and whether they are mutually exclusive. Toyota in particular is
credited with making lean a well-known approach as embodied in the Toyota
Production System (TPS).
Lean is about eliminating wastes, taking time out of processes, and creating better flow.
Asked about the essence of lean (TPS), Taiichi Ohno summarized it as “All we’re trying
to do is shorten the timeline from order receipt to collecting the cash for the goods or
services
provided.”

Six sigma has been defined in a variety of ways. One definition states, “Six sigma is a
business strategy and philosophy built around the concept that companies can gain a
competitive edge by reducing defects in their industrial and commercial processes.” A
few key characteristics of lean and six sigma are compared in Table shown.

There are some explanations from the points of view of lean and six sigma purists.

• From Improvement perspective, Six Sigma reduces variation and Lean reduces
waste.
• Six Sigma aims at a process performance of 3.4 Defects per Million opportunity and
Lean focuses on improving speed.

• Six Sigma has a flair to improve cost of poor quality and Lean improves
Operatingcosts
• Six Sigma has a longer learning curve and lean has a shorterone.
• Six Sigma uses various approaches for process improvements whereas lean mainly
uses value stream mapping.

• The project length for a six sigma project is 2 to 6 months and for a lean is 1 week to
3 months.

• Data is the main driver in a six sigma project while demand is the main driver for
lean.
• Six Sigma projects are of higher complexities while Lean projects are of
moderate complexities.

Comparing Six Sigma and Lean


Essentially, Six Sigma and Lean systems have the same goal. They both seek to
eliminate waste and create the most efficient system possible, but they take different
approaches toward how achieving this goal. In simplest terms, the main difference
between Lean and Six Sigma is that they identify the root cause of waste differently.
Lean practitioners believe that waste comes from unnecessary steps in the production
process that do not add value to the finished product, while Six Sigma proponents
assert that waste results from variation within the process.
Should six sigma and lean coexist in any organization? The answer to this question
is self-evident is “Yes”.
Lean approaches should precede and coexist with the application of six sigma methods.
Why? Put simply, lean provides stability and repeatability in many basic processes.
Once stability has taken hold, much of the variation due to human processes goes
away. The data collected to support six sigma activities thereby becomes much more
reliable and accurate.
Lean and six sigma tools can be depicted on a linear continuum with lean six sigma
in the middle.
In fact, these two disciplines have proven to be especially successful when working in tandem -
hence the creation of Lean Six Sigma.

Major business problems fall into the following categories:

• There seems to be a lot of waste

• There is a need to minimize inventories and redundancies

• There is a need to improve work flows

• There is a need to speed up processes

• There are human mistakes

If so, then lean tools should be utilized to:

• Eliminate wastes: Waste is a part of every process and should be identified and
eliminated.

• Simplify processes: If processes are more complex, we would require a highly


trained task force to execute the activities of those processes. Hence, processes
should be simplified as much as possible and Lean tools could be used to do that.

• Increase speeds: Speed is a very important factor for the customer. If speed
component is improved, it has a direct correlation with customer satisfaction.
• Improve flows: If the flow of product is not streamlined during the production
phase, it could lead to bottlenecks, increase in idle time and much more. Hence,
wherever possible, Lean tools should be used to improve flows within our
processes. • Minimize inventories: Inventories are critical aspect of a given
process. Larger inventories lead to higher costs of real estate, it also leads to raw
material wear and tear and much more. Similarly, smaller levels of inventories
lead to a hault in production due to non-availability of raw materials when
required.

• Mistake proof processes: Lean is effectively used to reduce or eliminate mistakes


in given processes.

However, if organizational challenges exhibit the following attributes:

• There are quality issues

• There is excessive variation

• There are complex problems

• There are challenging root cause identifications

• There are numerous technical considerations

In these cases, six sigma tools should be utilized to:

• Minimize variation

• Apply scientific problem solving

• Utilize robust project chartering

• Focus on quality issues

Employ technical methodologies on of both sets of issues. Placing lean six sigma
in the middle of this continuum, reflects a more holistic and synergistic approach.
If a specific problem requires only lean or six sigma tools, then that is perfectly
ok.

Lean six sigma is a relatively new paradigm providing a broader selection of


approaches. If the only tool in a company’s bag is a hammer, then all problems start to
look like a nail. It is best to have a tool kit with a broader set of tools, principles and
ways of thinking.
An increasing number of organizations (manufacturing, service, hospitals,
municipalities, military, insurance, etc.) have been unifying their efforts into a lean
six sigma approach. The mechanisms of these combinations vary widely.
Applications of the various lean and six sigma tools at various problem solving stages.
Define
• Value Stream Mapping
• Charter – Problem Statement
• Voice of Customer
• Communication Plans

• CTQ Issues
• Business Results
• Benchmarking
Measure
• Prioritization Matrix
• MSA Studies
• Capability Studies
• Videotaping
• Time Studies
• SIPOC
• Collecting Data
Analyze
• Regression Analysis

• 5 – Whys
• Cause – Effect Diagrams
• Root Cause Analysis

• ANOVA

• Multi-vari analysis
• Hypothesis Testing
Improve

• DOE
• Kaizen Events
• TOC
• Pull System
• SMED/SUD
• 5S or 6S
• Work Flow Improvement
Control
• SPC
• Visual Controls
• Control Plans

• TPM

• Standard Work
• Procedures and Work Instructions
• Training Requirements

Lean Implementation

5 Principles of Lean

Identify Customers and Specify Value - Only a small fraction of the total time and
effort adds value for the end customer. By clearly defining Value for a specific product
or service from the end customer’s perspective, all the non value activities - or waste
- can be targeted for removal.
Identify and Map the Value Stream - The Value Stream is the entire set of activities
involved in delivering the product or service. Once you understand what your customer
wants the next step is to identify how you are delivering (or not) that to them.
Create Flow by Eliminating Waste - Typically when you first map the Value Stream
you will find that only 5% of activities add value, this can rise to 45% in a service
environment.
Eliminating this waste ensures that your product or service “flows” to the customer
without any interruption, detour or waiting.
Respond to Customer Pull - This is about understanding the customer demand on
your service and then creating your process to respond to this. Such that you
produce only what the customer wants when the customer wants it.
Pursue Perfection - Creating flow and pull starts with radically reorganising individual
process steps, but the gains become truly significant as all the steps link together. As
this happens more and more layers of waste become visible and the process continues
towards the theoretical end point of perfection, where every asset and every action
adds value for the end customer.
1.4.4 The Seven Elements of Waste

Over Over
Correction Inventory Motion conveyance waiting
production processing

As discussed previously, Taiichi Ohno, considered the father of Toyota Production


System, created a lean manufacturing framework, which was based on the idea of
preserving (or increasing) value with less work. Anything that doesn't increase value
in the eye of the customer must be considered waste, or "Muda", and every effort
should be made to eliminate that waste. Non-value added activities are classified as
muda.

This term describes the waste that exists in a process. At each process step work is
applied. The useful activities that the customer will pay for is considered value
added. The other activities are not important to the customer or contain elements
that the customer will not pay for. Those non-paying activities are muda.
A list of seven waste categories are widely used:
Overproduction: The muda of overproduction is producing too much at a particular
point in time.
Correction: The correction of defective parts involves a second attempt at producing a
good item.
Inventory: Parts, raw materials, work-in-process (semi-finished goods), inventory,
supplies, and finished goods are all forms of inventory.
Motion: The efficient use of the human body is critical to the well being of the operator.
Overprocessing: Processing muda consists of additional steps or activities
in the manufacturing process.
Conveyance: All forms of Conveyance are muda.
Waiting: The muda of waiting occurs when an operator is ready for the next operation,
but must remain idle.
Overproduction
The muda of overproduction is producing too much at a particular point in
time. Overproduction is characterized by:

• Producing more than what is needed by the next process or customer

• Producing earlier than needed by the next processor customer

• Producing faster than is needed by the next process or customer


In the just-in-time environment, producing too early is as bad as producing too late.
Parts need to be available at a certain location at a certain time according to the
customer’s schedule. Having the product too early, too late, or in quantities that are
too great will result in undesirable consequences, such as:

• Extra space used at the customer’s plant

• Extra space used at the organization’s plant

• Extra raw materials in use

• Extra utilities used

• Extra transportation for the customer and organization

• Extra scheduling costs


Correction:
The correction of defective parts involves a second attempt at producing a good item.
Scrapping the whole part is a definite waste of resources. Having rejects on a
continuous flow line defeats the purpose of continuous flow.
Line operators and maintenance will be used to correct problems, putting the taken
time off course. Correction may require nonconforming product forms to be filled out
by suppliers, generating additional muda. Various design changes can be muda also.
Design changes are often considered correction or an extra development effort. Both
of these activities will create the need for additional labour.
Reworking the defects in a transaction, calling the customers to correct the
information that was earlier given as incorrect, etc are some of the examples of
correction.
Parts, raw materials, work-in-process (semi-finished goods), inventory, supplies, and
finished goods are all forms of inventory.
Inventory:
Excess inventory is considered muda since it does not add value to the product.

Inventory will:

• Require space in the shop

• Require transportation

• Require forklifts
• Require conveyor systems
• Require additional labor

• Require interest on material costs


Inventory sitting around in various process stages can be adversely affected in the
following ways:

• It may gather dust

• It can deteriorate

• It can become obsolete

• It may get wet

• It can experience handling damage

Motion

The efficient use of the human body is critical to the well being of the operator.
Extra unneeded motions are wasteful. Operators should not have to walk excessively,
lift heavy loads, bend awkwardly, reach too far, repeat motions, etc.
New tools should be designed to help with strenuous hand or body motions. The layout
of the workplace should be designed to take advantage of proper ergonomics. Each
work station should be analyzed for ergonomic and motion requirements.
Ergonomics can eliminate factors in the workplace that may cause injuries and
lost production. Some guidelines for providing sound ergonomic principles in the
workplace include:

• Emphasize safety at all times

• Fit the employee to the job

• Change the workplace to fit the employee (not vice versa)

• Design the workplace so that neutral body positions are maintained

• Redesign tool handles to reduce stress and injury

• Vary the tasks through job rotations (every 2 to 4 hours)

• Make the machine serve the human


Processing
Processing muda consists of additional steps or activities in the manufacturing process.

Examples include the following:

• Having to remove burrs from a manufacturing process

• Reshaping a piece due to poor dies

• Adding an extra handling process due to lack of space


• Performing an inspection step (all inspection is non-value added)
• Repeating product changes that are unnecessary

• Maintaining extra copies of information


Conveyance
All forms of Conveyance are muda.
This describes the use of forklifts, conveyors, pallet movers, and
trucks. This can be caused by

• poor plant layouts,

• poor cell designs,

• use of batch processing,

• long lead times,

• large storage areas,

• or scheduling problems.
Conveyance should be eliminated whenever possible.
Waiting
The muda of waiting occurs when an operator is ready for the next operation, but
must remain idle.
The operator is idle due to machine downtime, lack of parts, unwarranted monitoring
activities, or line stoppages. A maintenance operator waiting at a tool bin for a part is
muda.
The muda of waiting can be characterized by:

• Idle operators

• Breakdowns in machinery

• Long changeover times

• Uneven scheduling of work

• Batch material flows

• Long and unnecessary meetings

1.4.5 5S

Implementing 5S is a fundamental first step for any manufacturing company wishing


to call itself world class. The presence of a 5S program is indicative of the
commitment of senior management to workplace organization, lean manufacturing,
and the elimination of muda (Japanese for waste).
The 5S program mandates that resources be provided in the required location, and
be available as needed to support work activities.
The five Japanese “S” words for workplace organization are:

• Seiri (proper arrangement)

• Seiton (orderliness)

• Seiso (cleanup)

• Seiketsu (standardize)

• Shitsuke (personal discipline)

For American companies, the translated English equivalents are:

• Sort: Separate out all that is unneeded and eliminate it

• Straighten: Put things in order, everything has a place

• Scrub (or shine): Clean everything, make the workplace spotless

• Standardize: Make cleaning and checking routine

• Sustain: Commit to the previous 4 steps and improve on them

The 5S approach exemplifies a determination to organize the workplace, keep it neat


and clean, establish standardized conditions, and maintain the discipline that is
needed to do the job. Numerous modifications have been made on the 5S structure.
It can be reduced to 4S. It can be modified to a 5S + 1S or 6S program, where the sixth
S is safety. The 5S concept requires that discipline of will be installed and maintained.

5S – Sort (Organize) & Straighten


Details of a 5S program are itemized below in a step-by-step approach. Step 1:
Sort (Organize):

• Set up a schedule to target each area

• Remove unnecessary items in the workplace

• Red tag unneeded items, record everything that is thrown out

• Keep repaired items that will be needed

• Major housekeeping and cleaning is done by area

• Inspect the facility for problems, breakages, rust, scratches and grime

• List everything which needs repair

• Deal with causes of filth and grime

• Red tag grime areas and prioritize conditions for correction

• Perform management reviews of this and other steps


Step 2: Straighten

• Have a place for everything and everything in its place to ensure neatness
• Analyze the existing conditions for tooling, equipment, inventory and supplies

• Decide where things go, and create a name and location foreverything

• Decide how things should be put away, including the exact locations

• Use labels, tool outlines, and color codes

• Obey the rules. Determine everyday controls and out-of-stock conditions

• Define who does the reordering and reduce inventories • Determine who has

missing items or if they are lost

• Use aisle markings, placement for dollies, forklift, boxes

• Establish pallet zones for work in process (WIP)

Step 3: Scrub (Shine and Clean)

• This is more than keeping things clean, it includes ways to keep things clean

• Establish a commitment to be responsible for all working conditions

• Clean everything in the workplace, including equipment

• Perform root cause analysis and remedy machinery and equipment problems

• Complete training on basics of equipment maintenance

• Divide each area into zones and assign individual responsibilities

• Rotate difficult or unpleasant jobs

• Implement 3-minute, 5-minute and 10-minute 5S activities

• Use inspection checklists and perform white glove inspections


Step 4: Standardize & Sustain

• Make 5S activities routine so that abnormal conditions show up

• Determine the important points to manage and where to look

• Maintain and monitor facilities to ensure a state of cleanliness

• Make abnormal conditions obvious with visual controls

• Set standards, determine necessary tools, and identify abnormalities

• Determine inspection methods

• Determine short-term countermeasures and long-term remedies

• Use visual management tools such as color coding, markings and labels

• Provide equipment markings, maps, and charts


Step 5: Sustain
• Commit to the 4 previous steps and continually improve on them

• Acquire self-discipline through the habit of repeating the 4 previous steps

• Establish standards for each of the 5S steps

• Establish and perform evaluations of each step

Visual Factory
The term “visual factory” refers to an approach to lean manufacturing processes
based on visual information throughout the workplace. A visual factory uses a system
of communication tools to share information at the time and place it is needed.

Visual controls improve working areas by answering questions; identifying


equipment, materials, and locations; describing actions and procedures; and
providing safety warnings and precautionary information. Visual tools also boost
productivity by giving employees the information they need, where and when they
need it.

How Does a Visual Factory Work?

As part of the lean manufacturing process, a visual factory uses signs, labels,
charts, infographics, and other communication tools to provide data in an efficient
manner. Conveying this information visually reduces the time invested in reading,
processing, and interpreting text-based instructions.

• Work processes
• The information an employee needs to complete their tasks
• General layout of the jobsite and key access areas (including aisles,
entrances,and paths) where information would help an employee
• What an employer hopes to accomplish by developing a visual factory

In assessing these areas, employers might consider implementing 5S, which is


another lean manufacturing tool that eliminates waste, promotes standardization,
and encourages facility-wide organization. One of the core principles of 5S is visual
order, making it a natural tool for developing a visual factory.

The next step is to determine how to share that information.

This is accomplished through a series of visual cues and systems that reflect the needs
of a specific workplace. Those may include:

• Electrical display boards, which provide real-time information about the status
of production (or a process) within a facility and alerts workers whenever
assistance is needed
• Floor marking aids and wayfinding signs, which can help employees find
walkways, entrances, exits, navigational aids, and other instructions.
• Signs and labels that designate inventory, storage, tools, and other vitalresources

Benefits of a Visual Factory

Visual factories provide numerous benefits throughout a facility, including:

• Improved productivity and output


• Clear, simplified instructions in complex environments
• Safer workplace (given the clear, intentional visual communication)
• Higher profits (due to more efficient processes)
• Fewer errors and less guesswork throughout a work process

Visual Factory Solutions

Labeling and signage are key components of a visual workplace. Thoughtful, well-
placed labels and signs, as well as floor and aisle marking, give employees the
information they need and contribute toward the success of their company through
clear communication.

These signs and labels are used by health and safety managers around the
world to keep employees safe, direct them around a jobsite, and alert them
to important information.
Strategic floor and aisle marking is another component of implementing a visual
factory. PathFinder floor marking, wayfinding, and safety tape by Graphic
Products can establish clear, safe paths; identify aisles, storage areas, and key
items; and direct traffic between key areas. This communication boosts
productivity and leads to safer work processes.

Other Lean Techniques


KAIZEN - The word Kaizen means “Continuous Improvement".
Poka-Yoke Also known as Mistake Proofing - It employs visual signals that
make mistakes clearly stand out from the rest.
KANBAN utilizes visual display cards to signal movement of material between
steps of a product process. It is a scheduling system for LEAN and
JIT.
Just-in-Time (JIT) A philosophy which leads to "Producing the necessary units,
in the necessary quantities at the necessary time with the required quality”
JIDOKA Means “automation with a human touch”. It implements a supervisory
function in the production line and stops the process as soon as a defect is
detected. The process does not start until the root cause of the
defect is eliminated
Takt-time It is the frequency at which a product or service must be completed
in order to meet customer needs. The formula for Takt-time is: TAKT Time =
Available Time / Required Output .The best definition of “takt time is the Pace
of a manufacturing system as determined by the customer demand
Heijunka It means Production leveling/smoothing. It is a technique to reduce
the waste, which occurs due to fluctuating customer demand.
A lean enterprise technique used to document, analyze and improve the flow
of information or materials required to produce a product or service for a
customer is called Value Stream Mapping
2.0 Measure Phase

2.1 Process Definition


In the second phase of DMAIC methodology, it is important to focus on how does the
process currently perform? Or in other words, what is the magnitude of the problem?
Measurement is critical throughout the life of the project. As the team starts collecting data
they focus on both the process as well as measuring what customers care about. That means
initially there are two focuses: reducing lead time or improving quality. In the Measure Phase,
the team refines the measurement definitions and determines the current performance or
the baseline ofthe process.

When it comes to quality and efficiency, variation is your enemy. Whatever your business is,
you don’t want to leave anything up to chance. From the moment your client contacts you, a
predictable process should be followed with its aim being complete customer satisfaction.
Variation in the process will mean variation in the product.
Fishbone diagrams help you to determine the variables that may enter the equation. They
allow you to make your plans so that you know how to deal with them in such a way that the
quality of your final product is still up to standard and without significant variation.

2.1.1 Cause and Effect /Fishbone Diagram

Cause and effect diagrams are effective team based tools to determine the potential root
causes of a problem. It is also known as Ishikawa diagrams, fishbone diagrams, herringbone
diagrams, cause-and-effect diagrams, or Fishikawa. A cause and effect diagram examines
why something happened or might happen by organizing potential causes into smaller
categories. It can also be useful for showing relationships between contributing factors.

A cause and effect (fishbone) diagram:


• Breaks problems down into bite-size pieces
• Displays many possible causes in a graphic manner
• Is also called a cause and effect, 4-M or lshikawa diagram
• Shows how various causes interact
• Follows brainstorming rules when generating ideas

Common uses of the Ishikawa diagram are product design and quality defect prevention to
identify potential factors causing an overall effect. Each cause or reason for imperfection is a
source of variation. Causes are usually grouped into major categories to identify and classify
these sources of variation.
In the diagram, the defect is shown as the fish's head, facing to the right, with the causes extending
to the left as fishbones; the ribs branch off the backbone for major causes, with sub- branches for
root-causes, to as many levels as required.
A fishbone session is divided into three parts: brainstorming, prioritizing, and development
of an action plan. Identify the problem statement and brainstorm the categories in a fishbone
diagram. To prioritize problem causes, polling is often used. The three most probable causes
may be circled for the development of an action plan.
It is recommended to categorize factors in cause and effect analysis so that readers can
understand them better. During the discussion and draft phases of cause and effect analysis,
the 6M method can be applied. The 6M stands for manpower, machinery, materials, methods,
measurement and mother-nature.
Below is the detailed illustration of the method.

1. Manpower: Whether the personnel's technology proficiency and experience are up to


standard? Does the staff have quality consciousness, sense of responsibility anddisciplines?

2. Machinery: Check the facilities' stability, functionality, such as the GPS device's precision
and the cooling and lubrication state of equipment. If the machinery is rusting or eroding, the
production efficiency may decrease. Find out solutions to regularly maintain and repair
equipment.

3. Materials: Think about the materials' components, physical and chemical properties.
Examine whether different parts match well. Are the material suppliers stable or not?

4. Method: To be more specific, the methodology, methods or techniques will affect the
action's result. Other factors concerned are workflow, choice of technical parameters,
technical guidance and the preciseness and execution of workflow.

5. Mother-nature: The environment in production field, including temperature, humidity,


noise disturbance, vibrancy, lightening, and indoor pollution will all influence the products or
service.

6. Measurement: When it comes to measurement, the following factors need to be


considered for correct results: measurement gauge, measurement method, calibration,
measurer's fatigue, and readability of the results.

Generally, the 4-M (manpower, material, method, machine) version of the fishbone diagram
will suffice. Occasionally, the expanded version must be used. In a laboratory environment,
measurement is a key issue. When discussing the brown grass in the lawn, environment is
important. In the figure you will observe that the problem statement (the eye of the fish) has
several bones such as Measurement, Material, Machine, Method, Manpower and
Environment.
Cause and Effect (Fishbone) Diagram – Example
Here is an example of a Fish Bone Analysis for situation to assess High Turn Around Time (TAT) for
employess in an organisation.
As you can see in the diagram, the possible root causes categorized under “Methods” are
Staffing, Training, Approval, Processing, Escalation. The categorization under People include:
Inadequate Staffing, Inadequate Skills, Lack of Experience, Lack of Process knowledge. The
categorization of Measurement include collection consistency and CTP’s not measured. The
categorization under Mother Nature include Day of the week, week of the month, month of
the year and holiday season, the categorization under materials include: Lack of
Documentation, Lack of Knowledge Base and Lack of Escalation Matrix and the categorization
under Machine include: Server Downtime and Workstation Downtime.

2.1.2 Process Mapping – Overview

Process mapping consists of a collection of tools and methods used to understand an


organization and its processes. Those tools allow us to document, analyze, improve,
streamline and redesign business processes to realize organizational efficiencies.
A process map is a visual aid for picturing work processes and shows how inputs and tasks are
linked and highlights the steps required to consistently produce a desired output. A pictorial
illustration which identifies the steps, inputs and outputs, and other related details of a
process by providing a step-by-step picture of the process “as-is”.
A process map encourages new thinking about how work is done, where it is done, who
performs it, what problems frequently occur and how best to solve them.It is a team effort
and is documented by everyone who contributes to the process and/or is a part of the
process. Process maps help in characterizing the functional relationships between various
inputs and outputs.

Process Map – Flow charting Tools


Three commonly used process mapping tools to create detailed process maps are:
• Process Flowchart
• Deployment Flowchart
• Alternate Path Flowchart
Process Flow Chart
As you can see in the diagram, the process flowchart is a regular flowchart of the process. A flowchart
is a picture of the separate steps of a process in sequentialorder.

Elements that may be included are: sequence of actions, materials or services entering or
leaving the process (inputs and outputs), decisions that must be made, people who become
involved, time involved at each step and/or process measurements.

In this example, an individual lifts a receiver, dials number, it rings the opposite phone, if the
call is answered, we hear a “Hello”, then we have a conversation and finally we hang-up the
phone. If the phone is not answered, we will hang-up the phone.

Example of Deployment Flowchart:

In the deployment flowchart, a dealer places advertisement, an agent will initiate a call and
the buyer will answer, will discuss the details, agree on the details and then hang-up. Similarly,
the agent and buyer discusses further, there is an agreement, then a hang-up, we write the
contract and the attorney reviews the contract.
In a deployment flowchart, we create a flowchart using swimlanes and each swimlane is the
role of each individual. A deployment flowchart (sometimes referred to as a cross functional
flowchart) is a business process mapping tool used to articulate the steps and stakeholders of
a given process.
"Deployment flowcharts consist of a sequence of activity steps and also the interactions
between individuals or groups." Each participant in the process is displayed on the map (which
is constructed as a matrix) - tasks/activity are then articulated in sequence under the column
corresponding to that stakeholder.

Alternate Path Flowchart:

In this example, it is decided whom to call, if we have the number we dial the number, if we
do not have the number, we lookup the number in the telephone directory and then dial the
number. If the receiver is picked up, we say “Hello”, have a conversation and then hang-up.
Likewise, if the phone is not picked up and the voicemail is setup, we will leave a message and
then hang-up. If the voicemail is not setup, the phone is hung-up.

Commonly used Symbols in Process Mapping

SIPOC
A key concept in six sigma methodology is the SIPOC high level process map in which SIPOC
stands for suppliers, inputs, process, outputs, and customers. This diagram identifies major
core processes. The SIPOC tool is particularly useful when it is notclear:

• Who supplies inputs to the process?


• What specifications are placed on the inputs?
• Who are the true customers of the process?
• What are the requirements of the customers?

Example SIPOC: Automobile repair

Supplier Input Process Output Customer


• Vehic • Repair inquiry • Schedule • Appointment • Vehic
le • Vehicle visit date and time le
own for repair • Diagno • Repair own
er • Permission to se proble recommendati er
• Customer proceed with m ons and cost • Mechanic
service individual • Prepare estimates • Custome r
representati recommendati work • Work order service
ve o ns order • Parts for represen
• Facilit y • Open bay • Source approved tati ve
mana • Parts for parts repairs
ger approved • Telephon e/e-
• Parts wind repairs mail/text
ow • Observations

• Perfor message
m notification
repair • Repair
s ed
• Notify vehicl e
that
service
is
complet
e

As you can see in the diagram, the Suppliers provide inputs which are processed as outputs and
received by the customer. The feedback is shared when Inputs and Outputs are provided, similarly,
the knowledge repository is updated with the process details.

The ultimate goal is to identify essential work flows and sources of variation in work over time.
The diagram can also be adapted to a number of essential support processes. SIPOC captures
the key components of success from suppliers through internal processes and on to key
customers. Other tools such as process mapping, flow charting, and affinity diagrams can be
used to further identify the major steps in a process or system.
The advantages of using a SIPOC model include:
• A display of cross functional activities in a single, simple diagram
• A “big picture” perspective to which additional detail can be added
• A framework applicable to both large and small organizations

The SIPOC process map is designed to be a high level process view with 4 – 7 displayed steps.
This is a flowchart viewed at the 50,000 foot level.
The map enables all team members to view the process in the same light. It offers an
overview of a project at a glance.

The following steps can be used for developing a SIPOC diagram:


• Have the team create the process map • The process may have 4 or 5 keysteps.
• List the outputs of the process.
• List the customers of the output of the process.
• List the inputs of the process.
• List the suppliers of the process.
• As an optional step, identify some preliminary requirements of thecustomers.
• Involve the team leader, champion, and other stakeholders for verification.

Value Stream Mapping

A value stream map is created to identify all of the activities involved in product
manufacturing from start to finish. This value stream may include suppliers, production
operations and the end customer. VSMs typically focus on material and information flow. For
product development, value stream mapping includes the design flow from product concept
to launch. This is the large view, looking at the entire system for improvement opportunities.
(Rother, 1999)
Benefits of a value stream map include:
• Seeing the complete process flow
• Identifying sources and locations of waste
• Providing common terminology for process discussions
• Helping to make decisions about the flow
• Tying multiple Lean concepts and techniques together
• Providing a blueprint for Lean ideas
• Showing the linkage between the information and material flows
• Describing how the process can change
• Determining effects on various metrics (Rother, 1999)
• Define Product Family

The recommended value stream approach is to map one product family. A product family is
defined as a group of products that pass through similar processing steps and over common
equipment. A product and equipment matrix can be used to indicate common features. See
Table for an example of the matrix.
The matrix shows products that go through a series of common processes. A work cell could
be formed to handle a particular flow. Another method is to create a Pareto chart of the
various products. The product with the highest volume should be used for the model line.

In the table provided in the Product Family Matrix, there are five products and there are four
processes. In this example, the equipment is used in four processes. Process 1 uses its
equipment for all five products. Process 2 uses equipment in Process A and Process D. Process
3 uses equipment in Process A, Process B and Process E. And finally, process 4 uses equipment
in Process A, Process B

Current State Map


A current state map of the process is developed to facilitate a process analysis. Basic tips on drawing
a current state map include:
• Start with a quick orientation of process routes
• Personally follow the material and information flows
• Map the process with a backward flow, from shipping dock to the beginning
• Collect the data personally, do not trust the engineering standard times
• Map the whole stream
• Create a pencil drawing of the value stream

Some of the typical process data included are: cycle time (CT), changeover time (COT), uptime
(UT), number of operators, pack size, working time (minus breaks, in seconds), WIP, and scrap
rate. An analysis of the current status can provide the amount of lead and value- addedtime.
In many situations teams take on the task of data collection. Both individuals and teams
find it beneficial to develop a VSM data box in advance.
Important Value stream mapping definitions include:
Value-added time (VAT) - The amount of time spent transforming the product, which the
customer is willing to pay for.
Lead time (L/T) - The time it takes one piece of product to move through all of the processes. Cycle
time (C/T) - The time it takes a piece to complete an individual process.

Future State Map


A future value stream map, is an attempt to make the process lean. This involves creativity
and teamwork by the value stream manager and the lean team to identify creative solutions.
Everything the team knows about lean manufacturing principles is used to create the process
of the future.

Questions to ask when developing a future state map are:


• What is the required takt time?
• Do manufactured items move directly to shipping?
• Are items sent to a finished goods supermarket for customer pull?
• Is continuous flow processing applicable?
• Where isthe pacemaker process? (This process controls the tempo of the value
stream.)
• Can the process be levelled?
• What is the increment of work to be released for kanban use?
• What process improvements can be used: changeover, machine uptime, kaizen events,
SMED, etc.?
• Implementation Planning

The final step in the value stream mapping process is to develop an implementation plan for establishing
the future state. This includes
• a step-by-step plan
• measurable goals
• and checkpoints to measure progress.

A Gantt chart may be used to illustrate the implementation plan. Several factors determine
the speed of the plan. These include available resources and funding. The plan could take
months or years to complete and even then, there may be a need to improve upon it in the
future.
Value Stream Mapping Icons

The following icons may be used with value stream mapping, kanban, and other lean
manufacturing areas.

2.1.3 X-Y Diagram

The X-Y Matrix is a Six Sigma tool mostly used during the DMAIC measure phase and the
DMADV measure phase to show the relationship between X and Y factors.
The XY Matrix table illustrates the correlation of process inputs to customer’s outputs. The
table is illustrated using a group of columns and rows, with factor X (input) represented by
the horizontal axis and factor Y (output) represented by the vertical axis.
This is also called the C&E Matrix or the Cause and Effect Matrix or the priority matrix. Most
of the time a Process Map, which is the origin of related information, is used to rate the
relation between two factors. Numerical ranking is created to identify which amongst the
proposed plan is the most feasible and least expensive.
The objective of the XY Matrix is to numerically calculate the correlation of Y (output) which
is equal to the frequency multiplied by X (input). This filters all possible scenarios and probable
contributors to the problem at hand. The tool also studies and analyzes the relationship
between what is being proposed in the process and what benefit the customer would get
from it. The XY Matrix allows the group to determine areas of development and process gaps.
There are many purposes of the XY Matrix. Since the process uses numerical ranking the most
obvious use of this method is to narrow down several lists of imaginable inputs (Xs) into a
more controllable one.
The idea of the YX matrix is to multiply the weight with the score. If you look at the YX matrix
example, the score of 208 for input 2 is calculated as 8(score)×10(weight) + 8×8+4×7+9×4 =
208.
Similarly for others. The one with the maximum ranking can be used to identify the vital few factors.

In this example, an X-Y Diagram is used to identify the key actionable. The output variables
are described in the X axis. The output variables are Weld Strength, Weld Appearance, Pinhole
Density and Weld Flexibility. A weight is given for each of the Y variable such as 10, 8, 7 and 4
respectively.
Ultrasonic Weld Frequency, Power Amplitude, Initial Gap, Contact Pressure, Dwell Time and
Hold Time are factors impacting the Y-variables. On a scale of 1 to 10, the impact of X-variable
on Y variable is identified and documented. Finally, the ranking numbers are created.

2.1.4 Failure Mode and Effects Analysis (FMEA)


FMEA is a step-by step approach for anticipating potential failure modes in a product or in
delivering a service, or a process, or in a design. It is a qualitative tool used to also identify the
causes and the effects of the failure modes as perceived by an end user.

The Relationship between Failure and their Cause & Effect is shown below.
Failure mode effects analysis (FMEA) is a tool you can use in Six Sigma to quantify and
prioritize risk within a process, product, or system and then track actions to mitigate that
risk.

An FMEA provides the design engineer, reliability engineer, and others a systematic technique
to analyze a system, subsystem, or item for all potential or possible failure modes. This
method then places a probability that the failure mode will actually occur and what the effect
of this failure is on the rest of the system. If criticality of failure is considered, the technique is
called FMECA. The criticality portion of this method allows one to place a value or rating on
the criticality of the failure effect on the entire system. It is not uncommon to omit the
criticality portion from the methodology. A FMEA or FMECA (in most cases there is little
difference) is a detailed analysis of a system down to the component level. Once all items are
classified as to 1) failure mode, 2) effect of the failure and 3) probability that failure will occur,
they are rated as per their severity via an index called a RPN (risk priority number).
This RPN is dimensionless from the aspect that there is no real meaning to a value of say 600
versus 450 except in the difference in magnitude. Once all components or items have been
analyzed and assigned a RPN value, it is common to work from the highest RPN value down.
FMEA (Failure Mode Effect Analysis)

Definition of Terms
Failure Mode – The way in which a specific process input fails Effect
– The impact the failure has on the Critical Quality Parameter Cause
– The source of variation that caused the process to fail
Current controls – Systemized devices in place to prevent or detect the failure
Severity – Importance of an effect on critical quality parameter (1-Not severe; 10 – very severe)
Occurrence – Frequency with which a cause occurs (1-Not likely; 10-Very likely)
Detection – Ability of current control to detect the cause before creating a failure mode (1- likely
to detect; 10-not likely to detect)

The following are the steps that are taken to prepare an FMEA:
• FMEA number: This should be a log controlled number assigned for tracking the
document.
• The part number, name, or other appropriate description.
• The design responsibility: Which department or group is responsible for this design?
• The person responsible for FMEA preparation.
• The date the FMEA was prepared on and any necessary revision level.
• The subsystem or component part number being analyzed.
• The component function.
• The potential failure mode.
• The potential effect offailure. • The potential cause offailure.
• What are the current controls in place to prevent the cause from occurring?

FMEA Types
Design FMEA (DFMEA)
is done to identify and eliminate failures during product design, considering all types of failures
during the entire life-span of the product Process FMEA (PFMEA) is focused on failures of
processes of how the product is manufactured, maintained or operated System FMEA (SFMEA)
looks for failures and bottlenecks in larger processes, such as entire production lines or systems
FMEA Template

Risk Priority Number(RPN)


Risk Priority Number is calculated by multiplying the ratings of severity, Occurrence and
Detection.After the RPN is calculated, the potential failure modes are tabulated in descending
order of their RPNs.The team must decide upon the failure modes for which corrective action
is required based on their priorities
The table indicates the Ranking of Severity, Occurrence and Detectability from 1 to 10.

For ranking 1, the severity has no effect, the occurrence is remote: failure is unlikely less than
1 in fifteen lakhs and detectability has design control that will detect potential cause or the
mechanism and subsequent failure mode.

For ranking 2, the severity has a system operable with minimal interference. The occurrence
is Low: relatively few failures less than 1 in one lakh fifty thousand. It has very high chance
where the design control will detect potential cause/mechanism and subsequent failure
mode.

For ranking 3, the system is operable with some degradation of performance. The
occurrence is Low: Relatively few failures less than 1 in fifteen thousand.
The detectability has high chance where the design control will detect potential cause/mechanism and
subsequent failure mode.
For ranking 4, the system is operable with significant degradation of performance. The
occurrence is moderate: It has occasional failures less than 1 in 2000. It has a low chance of
the design control that will detect potential cause/mechanism and subsequent failure mode.

For ranking 5, the system is inoperable with no damage. The occurrence is moderate with
occasional failures of less than 1 in 400. It has a moderate chance where the design control
will detect potential cause/mechanism and subsequent failure mode.

For ranking 6, the system is inoperable with minor damage, the occurrence is moderate with
occasional failures with less than 1 in 80 instances. It has low chance where the design control
will detect potential cause/mechanism and subsequent failure mode.

For ranking 7, the system is inoperable with equipment damage. The occurrence is high:
repeated failures are less than 1 in 20 instances. It has a very low chance where the design
control will detect potential cause/mechanism and subsequent failure mode.

For ranking 8, the system is inoperable with destructive failure without compromising safety.
The occurrence is high: repeated failures are less than 1 in 8 instances. There is a remote
chance where the design control will detect potential cause/mechanism and subsequent
failure mode.

For ranking 9, the system is has very high severity ranking when a potential failure mode
affects safe system operation with warning. The occurrence is very high where failure is
almost inevitable less than 1 in 3 instances. There is a very remote chance that the design
control will detect potential cause/mechanism and subsequent failure mode.

For ranking 10, the system has very high severity ranking when a potential failure mode
affects operation without warning. The occurrence is very high: failure is almost inevitable
less than 1 in 2 instances. The design control cannot detect potential cause/mechanism and
subsequent failure mode.
2.2 Six Sigma Statistics

2.2.1 Basic Statistics


It is very important to measure the process inputs and outputs to optimize the process being
measured. Process inputs may be raw materials, human resources or services. All inputs have
some quantifiable measurement, including human effort and skill level. Process input
requirements should be stated so that key measures of input quality can be controlled.
Measurements within the process can also be used as effective controls. Once process
capabilities are known, output measures can be used to monitor if the process has remained in
control. Feedback from downstream process measurements can be used to improve an upstream
process. The Lean Six Sigma approach is data-driven, therefore Six Sigma statistics play a large
role in the Six Sigma problem-solving process. It stresses the importance of using Six Sigma
statistics to visualize and analyze the data collected for each metric that is important to the
customer and that must be tightly controlled.

For exp: A company had a high speed manufacturing process. The R&D engineers were working
on a major process change and were worried about the effect of the change on a quality defect.
They did their testing and wanted to verify the results. The methodology they used was correct
as well as the calculations. They had assumed (guessed) that the change would not increase
defects, and they set the risk (alpha) of abandoning a good process change at 5 percent. The
calculations from the data indicated that the actual risk was marginally better by 2.5 percent of
their limit, and they accepted the change and put it into production. The results were disastrous.

The defects increased from 0.08 percent to over 20 percent. The costs to recover ran into the
millions of dollars. So how could we have such a failure if we did the statistics correctly? One
reason was that the depth of the evaluation was very shallow; they needed to go further than
just the one test.

It's also important to have an understanding of what statistics really tells us. Statistical testing
for the sake of testing is worthless. It is in understanding what we gather from the analysis that
we find value. Statistical evaluation is only a tool to improve our decisions based on the limited
data we can gather. Making decisions on marginal results based on subjective limits provides little
more than throwing darts at a dart board. In the case above, making a critical decision based on
a 2.5 percent margin on risk as if it were gospel was a mistake. The thing to remember is, all we
are trying to do is fix a problem, and applying statistical reasoning aids for DMAIC projects will
aid us in making a good decision--but it is up to the users, not the math itself, to interpret what
the analysis means.

Data
Data is organized information. It can be in the form of numbers, words, measurements, observations
or even just descriptions of things. Data can be Qualitative or Quantitative
Data Types
Discrete data is countable. Example : [0, 1,2,3….countable infinte].For instance, the number of cancer
patients treated by a hospital each year is discrete
Continuous Data cannot possibly be counted and it can take any value in an interval on a real
line. Example: [-10 C, 50 C] for temperature. Temperature can be 15.5 C. Similarly distance, cycle
time, profit can take discrete values.
Attribute Data is Qualitative in nature. It is a type of data that has clear boundaries between values.

Following are the types of Attribute Data:-


Binary Data: A categorical type of data made up of dual classifications. For e.g. Yes / No, Pass /
Fail, On/Off, Male / Female. It can take numerical values but those numbers don’t have
mathematical meaning. Example : “1” indicating male and “2”indicating female.
Nominal Data: A type of data that is descriptive with more than two categories. For e.g. Names, Phone
numbers, Colors, Type of car
Ordinal Data: This type of data can be equated to Discrete values. This type of data is the
information organized in a particular order, without indicating specific relationship between
items.

Example : Rating a restaurant between 0 and 5 star. If you survey 100 people. You can add these
numbers and take an average and that numerical value has meaning.
2.2.2 Descriptive Statistics
Descriptive statistics are used to describe the basic features of the data in a study. They provide
simple summaries about the sample and the measures. Together with simple graphics analysis,
they form the basis of virtually every quantitative analysis of data.

Measures of central tendency


There are three main measures of central tendency: the mode, the median and the mean. Mean,
median, and mode are different measures of center in a numerical data set. They each try to
summarize a dataset with a single number to represent a "typical" data point from the dataset.
Mean: Mean is the most common measure of central tendency. It is simply the sum of the data
divided by the number of data in a set of data. This is also known as average.The "average"
number; found by adding all data points and dividing by the number of data points.
It can be used with both discrete and continuous data, although its use is most often with
continuous data. The mean is equal to the sum of all the values in the data set divided by the
number of values in the data set. So, if we have n values in a data set and they have values x1,
x2,
..., xn, the sample mean, usually denoted by (pronounced x bar), is:

This formula is usually written in a slightly different manner using the Greek capitol letter, ,
pronounced "sigma", which means "sum of...":

Example: The mean of 444, 111, and 777 is (4+1+7)/3 = 12/3 = 4(4+1+7)/3=12/3=4left parenthesis, 4,
plus, 1, plus, 7, right parenthesis, slash, 3, equals, 12, slash, 3, equals, 4.

Median: The median is the middle score for a set of data that has been arranged in order of
magnitude. The median is less affected by outliers and skewed data. In order to calculate the
median, suppose we have the data below:
65 55 89 56 35 14 56 55 87 45 92
We first need to rearrange that data into order of magnitude (smallest first):
14 35 45 55 55 56 56 65 87 89 92
Our median mark is the middle mark - in this case, 56 (highlighted in bold). It is the middle mark
because there are 5 scores before it and 5 scores after it. This works fine when you have an odd
number of scores, but what happens when you have an even number of scores? What if you had
only 10 scores? Well, you simply have to take the middle two scores and average the result. So,if
we look at the example below:
65 55 89 56 35 14 56 55 87 45
We again rearrange that data into order of magnitude (smallest first):

14 35 45 55 55 56 56 65 87

Only now we have to take the 5th and 6th score in our data set and average them to get a median of
55.5.

Example: The median of 444, 111, and 777 is 444 because when the numbers are put in order (1(1left
parenthesis, 1, 444, 7)7)7, right parenthesis, the number 444 is in the middle.

Mode: The most frequent number—that is, the number that occurs the highest number of times.
On a histogram it represents the highest bar in a bar chart or histogram. You can, therefore,
sometimes consider the mode as being the most popular option. An example of a mode is
presented below:

0 .00 10.00 20 .00 30 .00 40.00 50.00 60.00

Comparison of various measures

Measure Advantages Disadvantages


Mean 1. 'In tu itive 1 . Afflected by extreme values
2. Considers every data point 2. Considers every data point
3. Possess nice mathema t ic a l and 3. Te di o u s to compute
statistical properties 4 . Can not be computed for open ended
crass intervaJs
Median 1. Not affected by extreme values L Employs more complex statistical
2. Can be computed for open ended crass procedures
intervals
3. Can also be used for qualitative data

Mode 1. Not affected by extreme values L Difficult to interpret and compute in


2. Can als o be used for qualitative data case of multi- modals.

Probability Theory
Probability deals with calculating the likelihood of a given event’s occurrence. Probability is the
chance of something happening. For example, this chance could be getting a heads when we toss
a coin. Here something is 'getting a heads'. Probability is expressed as a fractional value between
'0' and '1'.
For example, the team wants to find out how many defects are expected ‘per’ so many linear
feet of extruded plastic. The operative word is ‘per’. The Poisson distribution would result in some
probability from 0 to 1 of those defects occurring. (exactly so many, more than so many, less than
so many) Probability of defect is the statistical chance that a product or process will not meet
performance specifications or lie within the defined upper and lower specification limits. It is the
ratio of expected defects to the total output.
For example, if hospital stays for admitted patients at a certain hospital are measured in hours
and were found to be normally distributed with some average hours and some standard
deviation; maybe the team would want to find out how many stays can be expected to last over
so many hours. Or, it could be expectations for being less than so many hours. You would be using
probability if you are using a Weibull analysis to determine the probability of something lasting
so many years, or so many miles. Probability theory is used with binomial data where the team
might be trying to find the probability of a specific number of ‘successes’.

Inferential Statistics
Inferential statistics use a random sample of data taken from a population to describe and make
inferences about the population. Inferential statistics are valuable when examination of each
member of an entire population is not convenient or possible.
For example, to measure the diameter of each nail that is manufactured in a mill is impractical.
You can measure the diameters of a representative random sample of nails. You can use the
information from the sample to make generalizations about the diameters of all of the nails.
Homogeneity Question

WHY STATISTICAL HOMOGENEITY IS IMPORTANT


Homogeneity and its opposite, heterogeneity, arise in describing the properties of a dataset, or
several datasets. They relate to the validity of the often convenient assumption that the statistical
properties of any one part of an overall dataset are the same as any other part.
When we are about to apply any apparatus from probability theory, we must perform a very
important test - we must check that our data represents a single process. Only when this check
is done, we can call values "random variable" and apply statistics to it.This need is usually
overlooked. And, eventually, models became imprecise or BI shows us wrong results, etc.
Let's have a look at a simple example: one of the pages on your site was under attack by the
bots. If you do not separate bot traffic from regular activity, you will count bots as users and come
to a conclusion that your recent ad campaign was very effective.
In the very heart of our product, we have math helping you to distinguish information of different
nature and then process it separately. Without it, you cannot be sure that you are comparing
apples to apples.
It is often important to determine if a set of data is homogeneous before any statistical technique is
applied to it. Homogeneous data are drawn from a single population.

Measures of Dispersion:
A measure of dispersion, is used to describe the variability in a sample or population. It reveals
how the items are spread out on either side of the center. The measures of central tendency are
not adequate to describe data. Two data sets can have the same mean but they can be entirely
different. Thus to describe data, one needs to know the extent of variability. This is given by the
measures of dispersion. It is usually used in conjunction with a measure of central tendency, such
as the mean or median, to provide an overall description of a set ofdata.
The degree of variation among the values is evaluated by various measures of dispersion, which are
as follows: -
Range: The difference between Largest (L) and Smallest (S) values (L – S) . The range is the
difference between the largest and the smallest observation in the data. The prime advantage of
this measure of dispersion is that it is easy to calculate. Example: In {4, 6, 9, 3, 7} the lowest value
is 3, and the highest is 9.

On the other hand, it has lot of disadvantages. The range can sometimes be misleading when
there are extremely high or low values.It is very sensitive to outliers and does not use all the
observations in a data set. It is more informative to provide the minimum and the maximum
values rather than providing the range. An outlier is an observation that lies an abnormal
distance from other values in a random sample from a population.

Quartile Deviation (Q.D.): Quartile deviation is based on the lower quartile Q1 and the upper
quartile Q3. The difference Q3−Q1 and Q3−Q1 is called the inter quar le range. Hence the
interquartile range describes the middle 50% of observations. If the interquartile range is large
it means that the middle 50% of observations are spaced wide apart. The important advantage
of interquartile range is that it can be used as a measure of variability if the extreme values are
not being recorded exactly (as in case of open- ended class intervals in the frequency
distribution). Other advantageous feature is that it is not affected by extreme values. The main
disadvantage in using interquartile range as a measure of dispersion is that it is not amenable to
mathematical manipulation.
IEx a1 m p lle
P r o b l e m S t"a t e 1m e n t ::

C a l cu l a t e th e q u a r 1:Ji1 l e d e via t io n a n d co e ff ic ie n t o f q u art il e d e v i a t ion from t:lh e data


given bello w :

'short-tons)

of
Solution:

Maximum c um ulativ e
Class
Number Frequencies
Load ( short- Cables Bounder
toins )
(f) ies
9.3-9.. 7 2 9.25-9.75 2

9 . 8 - ]0 . 2 5 9.75- 2+5=7
10.25
10.3-10.7 12 10.25- 7 + 12 = 19
10.75
10.8-11.2 17 10.75- 19 + 17 =
11.25 36
11.3-11.7 14 11.25- 36 + 14 =
11.75 50
11.8-12.2 6 11.75- 50 + 6 = 56
12.25
12.3-12.7 3 12.25- 56 + 3 = 59
12.75
12.8-13.2 1 12.75- 59 + 1 = 60
13.25

The quartile deviation is a slightly better measure of absolute dispersion than the range, but it
ignores the observations on the tails. If we take difference samples from a population and
calculate their quartile deviations, their values are quite likely to be sufficiently different. This is
called sampling fluctuation, and it is not a popular measure of dispersion. The quartile deviation
calculated from the sample data does not help us to draw any conclusion (inference) about the
quartile deviation inthe population.

Standard Deviation: Standard deviation (SD) is the most commonly used measure of dispersion.
It is a measure of spread of data about the mean. SD is the square root of sum of squared
deviation from the mean divided by the number of observations.
This formula is a definitional one and for calculations, an easier formula is used. The
computational formula also avoids the rounding errors during calculation.

In both these formulas n - 1 is used instead of n in the denominator, as this produces a more accurate
estimate of population SD.
The reason why SD is a very useful measure of dispersion is that, if the observations are from a
normal distribution, then 68% of observations lie between mean ± 1 SD 95% of observations lie
between mean
± 1 2 SD and 99.7% of observations lie between mean ± 1 3 SD.
The other advantage of SD is that along with mean it can be used to detect skewness. The
disadvantage of SD is that it is an inappropriate measure of dispersion for skewed data.
For example to calculate S.D for the following data. 6,2,3,1
Mean Deviation (M.D.): Mean Deviation is the Arithmetic Mean of the differences of the values
from their average. The average used is either the Arithmetic Mean or Median.Also called mean
absolute deviation, it is used as a measure of dispersion where the number of values or quantities
is small, otherwise standard deviation is used.
Variance – The variance measures how far each number in the set is from the mean. It is the square
of the Standard Deviation.

2.2.3 Normal Distribution Curve and Normality


DEFINITION:
The Normal Distribution Curve (NDC) is a Probability Distribution graph where the most
frequently occurring value is in the middle and the other probabilities tail off symmetrically in
both directions

CHARACTERISTICS:
• The tails are asymptotic (curve theoretically does not reach zero)
• Curve Can Be Divided In Half With Equal Pieces Falling Either Side Of The Most
Frequently Occurring Value Mean = Median = Mode
• The area under the curve is 1 (100%)
• A Normal Curve Indicates Random Or Chance Variation
• The Peak Of The Curve Represents The Center Of The Process
• Normal distributions are divided into 3 standard deviations on each side of the
mean
LSL = Lower Specification Limit. The lowest acceptable limit as set by a customer, USL = Upper
Specification Limit. The highest acceptable limit as set by a customer

Normal Distribution Curve


Specification limits

A normal distribution is characterized by the mean and standard deviation.


Mean (mu), a measure of central tendency, is the average of all values in the population. (sigma)
is a measure of dispersion or variability. With smaller values of, values tend to be closer to the
mean or average.

Normal Distribution Curve

The normal distribution curve is one of the most important statistical concepts in Lean Six Sigma.
Lean Six Sigma solves problems where the number of defects is too high. A high number of defects
statistically equals high variation in the process. The normal distribution curve visualizes the
variation in a dataset. The dataset represented by the normal distribution curve could refer to
downtime in manufacturing or the amount of time it takes to take a call in a call center. If the
data follows a normal distribution curve, it means that the data is eligible for certain statistical
tests that are used in the analyze stage of the Six Sigma process.

2.2.4 Graphical Analysis


Graphical analysis includes boxplots, stem and leaf plots, scatter diagrams, run charts,
histograms and normal probability plots.

Let us now understand how to draw Box Plot, Stem and Leaf Plot, Scatter Diagram, Run Charts and
Histogram using Minitab.

Box & Whisker Plot


To determine its central tendency, spread, skewness, and the existence of outliers. A
box plot is a set of summary measures of the distribution of the data.
Example: Cycle time – Box Plot

45.91 49.61 47.67

54.59 45.66 49.46

52.84 51.04 48.72

49.20 54.59 50.17

49.13 49.68 48.04

48.99 52.96 47.72

49.19 51.69 49.61

51.18 53.26 49.15


45.95 50.58 47.02

48.06 48.12 50.76

An outlier is an observation point that is distant from other observations. An outlier may be due
to variability in the measurement or it may indicate experimental error; the latter are sometimes
excluded from the data set.

A box and whisker plot (sometimes called a boxplot) is a graph that presents information from a
five- number summary.It is especially useful for indicating whether a distribution is skewed and
whether there are potential unusual observations (outliers) in the data set. Box and whisker plots
are also very useful when large numbers of observations are involved and when two or more
data sets are being compared. (See the section on five-number summaries for more information.)

Box and whisker plots are ideal for comparing distributions because the centre, spread and overall
range are immediately apparent.

A box and whisker plot is a way of summarizing a set of data measured on an interval scale. It is
often used in explanatory data analysis. This type of graph is used to show the shape of the
distribution, its central value, and its variability.

In a box and whisker plot:

• the ends of the box are the upper and lower quartiles, so the box spans the
interquartile range
• the median is marked by a vertical line inside the box
• the whiskers are the two lines outside the box that extend to the highest and
lowest observations.

Example 1 – Box and whisker plots

Carl works at a computer store. He also recorded the number of sales he made each month. In the
past 12 months, he sold the following numbers of computers:

51, 17, 25, 39, 7, 49, 62, 41, 20, 6, 43, 13.

1. Give a five-number summary of Carl's sales.

2. Make two box and whisker plots, one for Angela's sales and one for Carl's. 3.

Briefly describe the comparisons between their sales.

Answer:

First, put the data in ascending order. Then find the median.

6, 7, 13, 17, 20, 25, 39, 41, 43, 49, 51, 62. Median
= (12th + 1st) ÷ 2 = 6.5th value
= (sixth + seventh observations) ÷ 2
= (25 + 39) ÷ 2
= 32

There are six numbers below the median, namely: 6, 7, 13, 17, 20, 25.
Q1 = the median of these six items
th
= (6 + 1 ) ÷ 2= 3.5 value
= (third + fourth observations) ÷ 2
= (13 + 17) ÷ 2
= 15

Here are six numbers above the median, namely: 39, 41, 43, 49, 51,
62. Q3 = the median of these six items
th
= (6 + 1) ÷ 2= 3.5 value
= (third + fourth observations) ÷ 2
= 46

The five-number summary for Carl's sales is 6, 15, 32, 46, 62.
If your improvement project involves a relatively limited amount of individual quantitative data,
a box- and-whisker diagram can give you an instant picture of the shape of variation in your
process. Often this can provide an immediate insight into the search strategies you could use to
find the cause of that variation.
Box-and-whisker diagrams are especially valuable to compare the output of two processes
creating the same characteristic or to track improvement in a single process. They can be used
throughout the phases of the Lean Six Sigma methodology, but you will find box-and-whisker
diagrams particularly useful in the analyze phase.

Histograms
A graphical method for displaying the shape of a distribution
To determine its central tendency, spread, skewness, and the existence of outliers.

Example: Yield – Histogram


Histograms

Histograms

If skewness is positive, the data are positively skewed or skewed right, meaning that the right tail
of the distribution is longer than the left.
If skewness is negative, the data are negatively skewed or skewed left, meaning that the left tail is
longer. If skewness = 0, the data are perfectly symmetrical. But a skewness of exactly zero is quite
unlikely for real- world data
If skewness is less than −1 or greater than +1, the distribu on is highly skewed. If
skewness is between −1 and −½ or between +½ and +1, the distribution is moderately
skewed. If skewness is between −½ and +½, the distribution is approximately symmetric.

A normal distribution has kurtosis exactly 3 (excess kurtosis exactly 0). Any distribution with kurtosis
≈3 (excess ≈0) is called mesokurtic.
A distribution with kurtosis <3 (excess kurtosis <0) is called platykurtic. Compared to a
normal distribution, its tails are shorter and thinner, and often its central peak is lower
and broader. A distribution with kurtosis >3 (excess kurtosis >0) is called leptokurtic.
Compared to a normal distribution, its tails are longer and fatter, and often its central
peak is higher and sharper.

Histograms

Although the histogram is essentially a bar chart, it creates a "lumpy distribution curve" that can
be used to help identify and eliminate the causes of process variation. Histograms are especially
useful in the measure, analyze and control phases of the Lean Six Sigma methodology.
Stem & Leaf Plot

A Stem and Leaf Plot is used to organize numbers (data). The stem represents the greatest place
value of all numbers. The leaves represent the next place value. Example:
Scatter Diagrams

A scatter diagram is a graphic display of many XY coordinate data points which represent the
relationship between two different variables.
It is also referred to as a correlation chart.
For example, temperature changes cause contraction or expansion of many materials. Both time
and temperature in a kiln will affect the retained moisture in wood. Examples of such
relationships on the job are abundant. Knowledge of the nature of these relationships can often
provide a clue to the solution of a problem. Scatter diagrams can help determine if a relationship
exists and how to control the effect of the relationship on the process.
In most cases, there is an independent variable and a dependent variable. Traditionally, the
dependent variable is represented by the vertical axis and the independent variable is
represented by the horizontal axis.
The ability to meet specifications in many processes is dependent upon controlling two
interacting variables and therefore, it is important to be able to control the effect one variable
has on another. For instance, if the amount of heat applied to plastic liners affects their durability,
then control limits must be set to consistently apply the right amount of heat. Through the use
of scatter diagrams, the proper temperature can be determined ensuring a quality product.

The dependent variable can be controlled if the relationship is understood.


Correlation originates from the following: • A cause-effect relationship
• A relationship between one cause and another cause
• A relationship between one cause and two or more other causes
• Not all scatter diagrams reveal a linear relationship. The examples here definitely
portray a relationship between the two variables, even though they do not
necessarily produce a straight
line. If a center line can be fitted to a scatter diagram, it will be possible to interpret it. To
use scatter diagrams, one must be able to decide what factors will best control the process
within the specifications.

As shown in the diagram, the first graph shows “Strong Positive Correlation”, the second one
shows “Strong Negative Correlation”, the third one shows “No Correlation”, next one is “Possible
Positive” followed by “Possible Negative” and some other patterns at the end.
Run (Trend) Charts
An average human brain is not good at comparing more than a few numbers at a time.
Therefore, a large amount of data is often difficult to analyze unless it is presented in some easily
digested format.
Data can be presented in either summary (static) or time sequence (dynamic) fashion.

Consider the following examples:


Run Chart – Same Value Plot: A run chart having seven or more points with same value in a sequence
indicates bias in the process.

Run Chart – Clustering Plot


Such a plot has lot of data points next to each other on one side of the median and the same way
on the side of the median over time and indicates a periodic shift in process average due to lot-
to- lot or setup variability. Also known as too few runs plot.
Run Chart – Mixtures Plot
Such a plot has an absence of points near the center line with a sequence of fourteen or more
points in a row alternating up and down. It indicates a bias or systematic sampling from different
sources or processes. Also known as cycle or too many runs plot.

Run Chart – Oscillation Plot


Such a plot, has data points constantly fluctuating up and down rapidly around the median and
indicates that the process is not steady or stable.
Run Chart – Trend Plot
Such a plot has a sequence of seven or more data points continuously increasing or decreasing and
indicates a gradual increase or decrease trend in the data measurement methods.

Run Chart – Shift Plot


Such a plot has a sequence of eight or more points on the same side of the median and indicates
a gradual shift in the process
Measurement System Analysis
Precision and Accuracy
There are two critical characteristics to examine in a gage system.

Graphical Distinction between Accuracy and Precision

Calibrating the measuring instruments is necessary to maintain accuracy, but it does


not necessarily increase precision.
In order to improve the accuracy and precision of a measurement process, it must
have defined test method and it must be statistically stable.

For example, you’re a snack foods manufacturer producing 12 oz. bags of potato chips. You
test the weight of the bags using a scale that measures the bags precisely (in other words,
there is little variation in the measurements), but not accurately – measuring 13.2 oz., 13.33
oz., and
13.13 oz. for three samples.

Or maybe the scale is accurate, measuring the three samples at 12.02 oz., 11.74 oz., and
12.43 oz., but not precise. In this case, the measurements have a larger variance, but the
average of the measurements is very close to the target value of 12 oz.
Or maybe your measurements are all over the place, with samples measuring at 11.64 oz., 12.35
oz., and 13.04 oz., in which case your scale may be neither accurate nor precise.

if you detect accuracy and/or precision errors, focus on improving your measurement system
before relying on your data and moving forward with your improvement project. Allow the
results of your Gage R&R Study to help you decide if recalibrating a scale or conducting more
training for new hires might be just what you need to get your measurement systems back on
track.

2.3.2 Bias, Linearity & Stability Bias

Bias is the difference between the output of the measurement system and the
true value of it. It is often referred to as accuracy.
Linearit
y
Measurement system linearity is found by obtaining reference part measurement values with
in the full operating range of the instrument and plotting the bias against their reference values.

Stabilit
y

Variation in measurement is when the same person measures the same unit using
the same measuring gage (or tool) over an extended period oftime.

2.3 Measurement System Analysis

A Measurement System Analysis (MSA) is a specially designed experiment that to identify


the component of variation in a measurement.
An MSA is done to verify if the measurement system is producing valid data.
This is generally done before taking any data-based decision
MSA is also known as Gage R & R (GRR) method.
It is an experimental and mathematical method of determining the extent to which a
variation, within a particular measurement process, contributes to overall variation in
the process.
The statistical analysis varies according to the data type.
Measurement System Analysis
Measurement Resolution: It is the capability of a measurement system to
detectthe smallest tolerable changes
Example: A tea pack being weighed on a truck weighing scale.

MSA Crossed
Determine the number of appraisers, trials, and parts, which may vary from study to study. A rule
of thumb is 2-3 appraisers, 2-3 trials, and 5-10 parts –
Identify three appraisers all of whom are trained in the procedure for measuring
accurately and identify them as 1, 2 and 3 respectively.
Collect six parts that represents the range of process variation. If the parts don't vary as much as
the process, the gage error will be overstated.
Identify each part with a number 1-6 in such a way that the appraisers cannot see the numbers
as they take the measurements.
Start with Appraiser 1 and measure each of the six parts in random order.
Proceed to Appraisers 2 & 3 following the same process.

Then repeat the process for trials two and three

Part Operator Measurement

1 1 6.02

2 1 5.96

3 1 6.08

4 1 5.9

5 1 5.98

6 1 5.96

1 2 5.97

2 2 6.01

3 2 5.9

4 2 5.96

5 2 6.1

6 2 6.14

1 3 5.98

2 3 6.09

3 3 5.98

4 3 5.88

5 3 6.06
6 3 5.99

1 1 6.04

2 1 5.94

3 1 6.06

4 1 5.88

5 1 5.94

6 1 6

1 2 5.96

2 2 6.03

3 2 5.84

4 2 5.89

5 2 6.12

6 2 6.09

1 3 5.91

2 3 6.07

3 3 6.04

4 3 5.92

5 3 6.02

6 3 5.98

1 1 5.99

2 1 5.88

3 1 6.01

4 1 5.93

5 1 5.98

6 1 6.03
1 2 5.92

2 2 5.99

3 2 5.8

4 3 5.98

5 2 6.06

6 2 6.12

1 3 5.96

2 3 6.05

3 3 6.01

4 3 5.9

5 3 6.05

6 3 5.97

MSA Crossed
Gage R&R Study - XBar/R Method
%Contribution

Source VarComp (of VarComp)

Total Gage R&R 0.001134 36.39


2
Repeatability 0.0011203 35.94

Reproducibility 0.0000139 0.45


Part-To-Part 0.0019827 63.61
Total Variation 0.003116 100.00
9
Process tolerance = 0.6
Study Var %Study Var %Toleranc

Source StdDev (SD) (6 * SD) (%SV) (SV/Toler)


Total Gage R&R 0.0336782 0.202069 60.32 33.68
Repeatability 0.0334712 0.200827 59.95 33.47
Reproducibility 0.0037287 0.022372 6.68 3.73
Part-To-Part 0.0445277 0.267166 79.76 44.53
Total Variation 0.0558295 0.334977 100.00 55.83
Number of Distinct Categories = 1

MSA Crossed

Source VarComp %Contribution

Total Gage R&R 0.0011342 36.39

Repeatability 0.0011203 35.94

Reproducibility 0.0000139 0.45

Part-To-Part 0.0019827 63.61

Total Variation 0.0031169 100

Study Var %Study Var %Tolerance

Source StdDev (SD) (6 * SD) (%SV) (SV/Toler)

Total Gage R&R 0.0336782 0.202069 60.32 33.68

Repeatability 0.0334712 0.200827 59.95 33.47

Reproducibility 0.0037287 0.022372 6.68 3.73

Part-To-Part 0.0445277 0.267166 79.76 44.53

Total Variation 0.0558295 0.334977 100 55.83

MSA Crossed
Variation from the measurement device, or Repeatability, equals 36.94% of the total
part variation
Variation from the appraisers, or Reproducibility, is equal to 0.45% of the
total part variation,

Total variation from Repeatability and Reproducibility combined (they are not directly
additive) is 36.39% of the total variation,
The rule of thumb for acceptance of a measurement system is a total Gage R&R of
30% or less of the Total Variation.. In this case, the measurement system is not
capable, and cannot be used as a basis of decision making.
MSA Crossed is a generally part of nondestructive testing MSA

Nested is a part of destructive testing

MSA Attribute
Identify the number of appraisers, trials, and parts, which may vary from study to study.
A rule of thumb is 2-3 appraisers, 2-3 trials, and 5-10 parts
Identify three appraisers who are all trained in the proper measurement
procedureand identify them as Tom, Joe and Paul.
The appraisers will judge whether the part is acceptable or not acceptable.
Collect 15 parts that represents the range of process variation. If the parts don't vary
as much as the process, the gage error will be overstated.
Identify each part with a number 15 in such a way that the appraisers can not see
the numbers as they take the measurements.
Start with Appraiser omT and measure each of the six parts in randomorder.
Proceed to Appraisers Joe and Paul following the same process.
Then repeat the process for trials two and three.
Get the true status of all the 15 parts from a Subject Matter Expert and mark as Attribute.
MSA Attribute

Tom Tom Tom

Sample Sheet 1 Sheet 2 Sheet 3

Attendence F F F

Credential P P P

Expediate P P F

Excavate F F F

Exemplery P P P
Judgement P P P

etiquette P P P

Conveyance P P P

Cortoon F F F

embassy P P P

equivalent P P P

Exuberent P P P

euthanasia F F F

Exasparate F F F

Extradite F F F

MSA Attribute

Joe Joe Joe

Sample Sheet 1 Sheet 2 Sheet 3

Attendence F F P

Credential P P P

Expediate F F F

Excavate P F F

Exemplery P P F

Judgement P P P

etiquette P P P

Conveyance P P F

Cortoon F F F

embassy F F F
equivalent P P P

Exuberent F P F

euthanasia F F F

Exasparate F P F

Extradite F F F

MSA Attribute

Paul Paul Paul

Sample Sheet 1 Sheet 2 Sheet 3

Attendence F P F

Credential P P P

Expediate P F F

Excavate P P F

Exemplery P P P

Judgement P P P

etiquette P P P

Conveyance P P F

Cortoon F F P

embassy P P P

equivalent P P P

Exuberent P P P

euthanasia F F F

Exasparate P F P
Extradite F P F

MSA Attribute

Tom Tom Tom Tom Tom

Sample Attribute Sheet 1 Sheet 2 Sheet 3 Repeatability Standards

Attendence F F F F P P

Credential P P P P P P

Expediate F P P F F F

Excavate P F F F P F

Exemplery F P P P P F

Judgement F P P P P F

etiquette P P P P P P

Conveyance P P P P P P

Cortoon F F F F P P

embassy P P P P P P

equivalent P P P P P P

Exuberent F P P P P F

euthanasia P F F F P F

Exasparate F F F F P P
Extradite P F F F P F

Total 15

Agree 14 8

Over Rej 3

Under Rej 3

Unsure 1

% Appriser 93.3

% Attribute 53.3
MSA Attribute

Joe Joe Joe Joe Joe


Sample Attribute Sheet 1 Sheet 2 Sheet 3 Appriser Attribute

Attendence F F F P F F

Credential P P P P P P

Expediate F F F F P P

Excavate P P F F F F

Exemplery F P P F F F

Judgement F P P P P F

etiquette P P P P P P

Conveyance P P P F F F

Cortoon F F F F P P

embassy P F F F P F

equivalent P P P P P P

Exuberent F F P F F F

euthanasia P F F F P F

Exasparate F F P F F F

Extradite P F F F P F

Total 15
Agree 9 5

Over Rej

Under Rej

Unsure

% Appriser 60.0

% Attribute 33.3

MSA Attribute

Paul Paul Paul Paul Paul


Sample Attribute Sheet 1 Sheet 2 Sheet 3 Appriser Attribute

Attendence F F P F F F

Credential P P P P P P

Expediate F P F F F F

Excavate P P P F F F

Exemplery F P P P P F

Judgement F P P P P F

etiquette P P P P P P

Conveyance P P P F F F

Cortoon F F F P F F

embassy P P P P P P

equivalent P P P P P P

Exuberent F P P P P F

euthanasia P F F F P F

Exasparate F P F P F F

Extradite P F P F F F

Total 15

Agree 8 4

Over Rej
Under Rej

Unsure

% Appriser 53.3

% Attribute 26.7

MSA Attribute

All
Tom Tom Joe Joe Paul Paul operato
r s All
agre e operato r
Attribu Repeatabili Standar Appris Attribu Appris Attribu
Sample within s agre e
te ty ds er te er te
and with
betwee std
n each
other
Attenden F P P F F F F 0
ce
Credential P P P P P P P 1 1

Expediate F F F P P F F 0

Excavate P P F F F F F 0

Exemplery F P F F F P F 0

Judgement F P F P F P F 1

etiquette P P P P P P P 1 1

Conveyan P P P F F F F 0
ce
Cortoon F P P P P F F 0

embassy P P P P F P P 1

equivalent P P P P P P P 1 1

Exuberent F P F F F P F 0

euthanasia P P F P F P F 1

Exasparate F P P F F F F 0

Extradite P P F P F F F 0

Total 15 15 15 0
Agree 14 8 9 5 8 4 6 3

Over Rej 3

Under Rej 3

Unsure 1

% Appriser 93.3 60.0 53.3

% 53.3 33.3 26.7


Attribute
% Effective 40.0
Score
% Effective Score Vs Standard 20.0

Measure Phase – Measurement System Analysis


2.4 Process Capability

Process capabilities the repeatability and consistency of a manufacturing process relative to the
customer requirements in terms of specification limits of a product parameter. This measure is
used to objectively measure the degree to which your process is or is not meeting the
requirements.

The output of this measurement is usually illustrated by a histogram and calculations that predict how
many parts will be produced out of specification (OOS).
Two parts of process capability are: 1) measure the variability of the output of a process, and 2) compare
that variability with a proposed specification or product tolerance.
2.4.1 Capability Analysis
The determination of process capability requires a predictable pattern of statistically stable
behaviour where the chance causes of variation are compared to the engineering specifications.

A capable process is a process whose spread on the bell-shaped curve is narrower than the
tolerance range or specification limits. USL is the upper specification limit and LSL is the lower
specification limit.

As you can see in the figure, the Lower Specification and Upper Specification Limits are described, the
process is targeted in the centre. It has minimum and maximum values as well.
It is often necessary to compare the process variation with the engineering or specification
tolerances to judge the suitability of the process. Process capability analysis addresses this issue.
A process capability study includes three steps:
• Planning for data collection

• Collecting data
• Plotting and analyzing the results

Measure Phase – Process Capability


Study Sigma Level

Sigma is a universal measure of process performance

Cp is Capability potential, which indicates how well a process would perform, if it were
centered around its target.

This need not necessarily be the actual performance of the process, as while calculating Cp
does not consider the location of the process, only it’s spread.
Also, It does not take into account the closeness of the estimated process Mean to the specification limits.
Cp = (USL – LSL) / 6S OR VoC / Process Behavior
CPK shows how variation affect’s the ability of a process to meet desired CTQs.
CPK measures how close you are to the target and how consistent you are to ‘around average
performance’.

CPK takes the location of the data, into account, by considering the closeness of Mean to the specification
limits.

Measure Phase – Process Capability Study


Sigma Level & Cpk
For continuous data, the Sigma level, or Z, is calculated from the mean, standard deviation, LSL and USL.
Z = (USL or LSL – mean ) / sigma
‘Z’ can be both greater than or less than zero

Z = 3 * Cpk
if cpk > 1.33, Z = 3 *
1.33 = 4
if cpk = 2, Z = 3 * 2 = 6

Long Term Process Capability Indices Pp and PpK). Long Term Process Capability Indices shows how
a process performs over a period of time
IT can be applied to Stable or unstable processes.It also Includes the effects of common cause
variations. It is calculated by replacing Short term X Bar and S with Long term X Bar and S. A
Process is considered capable if Pp and PpK are greater than or equal to 1.33. If the Pp index of a
process is 2, it means the process is performed at Six Sigma level quality.
Capability Indices – A Comparison

Cp and Cpk Pp and Ppk


Calculated using sample data collected Calculated using data collected cross a broad
across a narrow inference space and time inference space

Reflect how a process has performed in a


specified period.
Predict the performance of a process.
Cannot be used to predict the future
performance of a process.

Measure the best performance of a process Provides realistic measures of performance.


under controlled conditions. Represents the Represents the variability of both common causes
variability od common causes only and special causes

Calculated using samples collected as sub Calculated using data from entire process without
groups any sub groups

Example: Capability Short & Long terms

Within for Short


term

Overall for Long term

“Within” indicates that overall


data has many sub-groups, and
is used to calculate Cpk

The objective of process quality control is to establish a state of control over the manufacturing process
and then maintain that state of control through time.
When the natural process limits are compared with the specification range, any of the following possible
courses of action may result:
Other capability applications:
• Providing a basis to setup variables control chart
• Evaluating new equipment
• Reviewing tolerances based on the inherent variability of a process

• Assigning more capable equipment to tougher jobs

• Performing routine process performance audits


• Determining the effects of adjustments during processing

Identifying Characteristics:
The identification of characteristics to be measured in a process capability study should meet the
following requirements:
The characteristic should be indicative of a key factor in the quality of the product or process.
It should be possible to adjust the value of the characteristic.

The operating conditions that affect the measured characteristic should be defined and

controlled. Identifying Specifications/Tolerances

The process specifications or tolerances are determined either by

• Customer requirements
• Industry standards or
• The Organization’s Engineering department
The process capability study is used to demonstrate that the process is entered within the
specification limits and that the process variation predicts the process is capable of producing
parts within the tolerance requirements.
When the process capability study indicates the process is not capable, the information is used to
evaluate and improve the process in order to meet the tolerance requirements.
Sigma Level

Acceptance and Rejection region

Standard Normal Distribution


Standard Normal Distribution
Standard Normal Distribution

Standard Normal Distribution


Standard
Norma
l Distribution

Technical Proposal
Standard Normal Distribution

Process Non conformance


1.5 sigma shift
Sigma level calculation

Calculating Sigma Level

2.4.2 Concept of Stability

Process Stability and Process Capability are both extremely important aspects of any
manufacturing process. Often the concepts behind process stability and process capability and
the relationship between them are misunderstood.
Process Stability refers to the consistency of the
process with respect to important process
characteristics such as the average value of a key
dimension or the variation in that key dimension.
If the process behaves consistently over time, then
we say that the process is stable or in control.The
next graphic illustrates an unstable process. The
process distribution average is shifting over
time.below illustrates a stable process.

While there is no direct relationship between process stability and process capability, there is an
important connection: Process capability assessment should only be performed after first
demonstrating process stability. process capability is an assessment of the ability to meet
specification. However, if the process is unstable, we cannot predict its capability. Any estimate
of process capability we make depends entirely on where the process happens to be when we
collect the data. Suppose the process average is shifting about over time. An estimate of the
process capability is only reflective of where the process is at that point in time … not where it
may go next.

If only common causes of variation are present in a process, then the output of the process
forms a distribution that is stable over time and is predictable
Note, the process may also be unstable if either the process average or variation is out-of-control.
• Common Causes
• Special Causes

Common cause variability is a source of variation caused by unknown factors that result in a
steady but random distribution of output around the average of the data. Common cause
variation is a measure of the process's potential, or how well the process can perform when
special cause variation is removed. Consider an employee who takes a little longer than usual to
complete a certain task. He is given two days to do a task and instead he takes two and a half
days; this is considered a common cause of variation. His completion time would not have
deviated very much from the mean, since you would have had to consider the fact that he could
submit it a little late.
Here’s another example: you estimate 20 minutes to get ready and ten minutes to get to work.
Instead, you take five minutes extra getting ready because you had to pack lunch and 15
additional minutes to get to work because of traffic.

Other examples that relate to projects are inappropriate procedures, which can include the lack
of clearly defined standard procedures, poor working conditions, measurement errors, normal
wear and tear, computer response times, etc. These are all common cause variations.

Special cause variation is a shift in output caused by a specific factor such as environmental
conditions or process input parameters. It can be accounted for directly and potentially removed
and is a measure of process control.
Examples relating to project management are if a machine malfunctions, a computer crashes,
there is a power cut, etc. These kinds of random things that can happen during a project are
examples of special cause variation.One way to evaluate a project’s health is to track the
difference between the original project plan and what is actually happening. Use of control charts
helps to differentiate between the Common Causes and the Special Causes of Variation making
the process of making changes and amends easier.

2.4.3 Attribute and Discrete Capability

The control chart represents the process capability, once special causes have been identified and
removed from the process. For attribute charts, capability is defined as the average proportion
or rate of non-conforming product.
For p charts, the process capability is the process average non-conforming, and is preferably
based on 25 or more in-control periods. If desired, the proportion conforming to specification 1
- p, may be used.
For np charts, the process capability is the process average nonconforming p, and is preferably based
on 25 or more in-control periods.
For c charts, the process capability is the average number of nonconformities, c, in a sample of fixed
size n.
For u charts, the process capability is the average number of nonconformities per reporting unit, u.

The average proportion of nonconformities may be reported on a defects per million opportunities scale
by multiplying p times 1,000,000.
Normality of data
Normality of data is required to calculate Cpk and Ppk.

Normality test is used to check if


data is normal based on its p-
value.
Example

P-value = 0.693 > 0.05 (α) Data


is normal. Confidence level =
1- 0.05
= 0.95 or 95%

Attribute Capability - Binomial


Binomial Capability Analysis is used when data provided indicates the number of defective
items, out of the total number sampled.

Conditions to apply Binomial capability analysis:


Each item can result in one or two possible outcomes (Pass/ Fail,
Yes/No) The probability of a success (or failure) is constant for each item

The outcomes are independent of each other


Stat >

Quality Tools >

Capability Analysis >

Binomial
Attribute Capability - Poisson
Poisson Capability Analysis is used when data provided gives the number of defects inan a specific
amount of time or space

Conditions to use Poisson capability analysis:


• the rate of defects per unit of space or time is same for each item

• the number of defects observed in the items are mutually exclusive Stat >
Quality Tools >

Capability Analysis >

Poisson
Process Stability
• Any process output has an inherent naturalvariation.
• These limits of a process are determinedstatistically.
• Natural process limits are also knownas natural process variation or normal process
variation,
• In normally distributed data, ±3σ limits spread around the process average and cover
99.73% of process variation.

Example: Stability through Control Charts


- There are no point s which are out of the control limit.
- There are no observable patterns or trends. Process is in control andstable
Example: Capability and stability

Process in control Data is normal Cpk is 1.16 Data has sub-group

2.4.4 Monitoring Techniques

Monitoring Plan is a data collection plan for checking the ongoing health of the improved
process. It lists the measure, the targets for each measure, how each measure will be checked,
how and who will check the measures. It sets the stage for the Response Plan.

Transfer Tools
Transfer tools (such as spring calipers) have no reading scale. Jaws on these instruments measure
the length, width, or depth in question by positive contact. The dimension measurement is then
transferred to another measurement scale for direct reading.
Attribute Gages
Attribute gages are fixed gages which typically are used to make a go/no-go decision. Examples
of attribute instruments are master gages, plug gages, contour gages, thread gages, limit length
gages, assembly gages, etc. Attribute data indicates only whether a product is good or bad.
Attribute gages are quick and easy to use but provide minimal information for production control.

Type of Gage Accuracy Application

Micromete Mechanical accuracy is about Normally used to check diameter or


r (inside) 0.001”. Some digital models are thickness. Special models can check
accurate to 0.00005”. thread diameters.

Mechanical accuracy is about Normally used to check diameter or


Micromete
0.001”. Some digital models are thickness. Special models can check
r (outside)
accurate to 0.00005”. thread diameters.

Optical The accuracy can be within Measures difficult contours and


comparator 0.0002”. part configurations.

Depending on operator skill,


Optical flat Used only for very precise tool room
accurate to a few millionths of an
work. Best used for checking flatness.
inch.

Accuracy very good for checking Checking the diameter or drilled or


Plug gages
the largest or smallest hole rimmed holes. Will not check for out
diameter. of roundness.

Used to check flatness, waviness or


Precision Visual 0.10”. With a feeler gage squareness of a face to a reference
straight 0.003”. plane.
edge
Radius Used to check small radii,
an Accuracy is no better than 0.015”. and contours.
d template
gages

Will only discriminate against Best application is to approximate a


Ring gages
diameters larger or smaller than mating part in assembly. Will not
the print specification. check for out of roundness.
Split sphere and Used for measuring small
No better than 0.00005” using
telescope hole diameters.
a micrometer graduated in
0.0001”.
Steel ruler or Used to measure heights, depths,
scale No better than 0.015”. diameters, etc.

Used to measure the overall flatness of


Surface plates the object.
Flatness expected to be no better
than 0.005” between any 2 points.
Using an accurate micrometer, the Used to measure bore sizes in low
Tapere
accuracy is about 0.005”. volume applications.
d
parallel
s

Variable Gages
Variable measuring instruments provide a physical measured dimension. Examples of variable
instruments are rulers, Vernier callipers, micrometres, depth indicators, run out indicators, etc.
Variable information provides a measure of the extent that a product is good or bad, relative to
specifications. Variable data is often useful for process capability determination and may be
monitored via control charts.
Reference/Measuring Surfaces
A reference surface is the surface of a measuring tool that is fixed. The measuring surface is
movable. Both surfaces must be free from grit or damage, secure to the part and properly aligned
for an accurate measurement.

Instrument Selection
The terms measuring tool, instrument and gage are often used interchangeably in this text.
Appropriate gage should be used for the required measurement.

Monitoring Techniques – Instrument Selection

Accuracy
Type of Gage Application
About 0.001”. Some
Vernier depth gage digital models are Used to check depths.
accurate to 0.00005”.

Accuracy is no better than


0.0005” depending upon the Used with a surface plate and gage
Tool maker’s flat instrument used to measure the blocks to measure height.
height.

Vernier calipers About 0.001”. Some digital models Used to check


are accurate to 0.00005”. diameters and
thickness.

Attribute Screens
Attribute screens are screening tests performed on a sample with the results falling into one of
two categories, such as acceptable or not acceptable. Because the screen tests are conducted on
either the entire population of items or on a significantly large proportion of the population, the
screen test must be of a non-destructive nature. Screening programs have the following
characteristics:

• A clearly defined purpose


• High sensitivity to the attribute being measured (a low, false negative rate)
• High specificity to the attribute being measured (a low, false positive rate)
• Benefits of the program outweigh the costs
• Measured attributes identify major problems (serious and common)
• Results lead to useful actions
Common applications of screening tests occur in reliability assessments and in the medical
screening of individuals.
In reliability assessments, an attribute screen test may be conducted to separate production units
that are susceptible to high initial failure rates. This period is also known as the infant mortality
period. The test simulates a customer use of the unit, or perhaps an accelerated condition of use.
The number of failures, per unit of time, is monitored and the screen test continues until the
failure rate has reached an acceptable level. The screen test separates acceptable items from
failed items, and an analysis of the failed components is performed to find the cause of the
failure.
In medical screening, a specific symptom or condition is targeted and members of a defined
population are selected for evaluation. Examples of this type of screening include a specific type
of cancer or a specific disease. In many cases, the members of the selected population may not
be aware that they have the condition being screened. Medical screening tests have the ultimate
objective of saving lives.
• Tool Care

• Measuring instruments are typically expensive and should be treated with care to
preservetheir accuracy and longevity.
• Some instruments require storage in a customized case or controlled environment when
not in use.
• Even sturdy hand tools are susceptible to wear and damage.
• Hardened steel tools require a light film of oil to prevent rusting.
• Care must be taken in the application of oil since dust particles will cause build-up on
the gage‘s functional surfaces.

Measuring tools must be calibrated on a scheduled basis as well as after any suspected dam
Gage Blocks
Near the beginning of the 20th century, Carl Johansson of Sweden, developed steel blocks to an
accuracy believed impossible by many others at that time. His objective was to establish a
measurement standard that not only would duplicate national standards, but also could be used
in any shop. He was able to build gage blocks to an accuracy within a few millionths of an inch.
When first introduced, gage blocks or “Jo” blocks as they are popularly known in shops, were a
great novelty. Seldom used for measurements, they were kept locked up and were only brought
out to impress visitors.
Today gage blocks are used in almost every shop manufacturing a product requiring mechanical
inspection. They are used to set a length dimension for a transfer measurement, and for
calibration of a number of other tools.
We generally distinguish three basic gage block forms - rectangular, square and round. The
rectangular and square varieties are in much wider use. Generally, gage blocks are made from
high carbon or chromium alloyed steel. Tungsten carbide, chromium carbide, and fused quartz
are also
used. All gage blocks are manufactured with tight tolerances on flatness, parallelism, and surface
smoothness.
Gage blocks should always be handled on the non-polished sides. Blocks should be cleaned prior
to stacking with filtered kerosene, benzene or carbon tetrachloride. A soft clean cloth or chamois
should be used. A light residual oil film must remain on blocks for wringing purposes.
Block stacks are assembled by a wringing process which attaches the blocks by a combination of
molecular attraction and the adhesive effect of a very thin oil film. Air between the block
boundaries is squeezed out. The sequential steps for the wringing of rectangular blocks is shown
below. Light pressure is used throughout the process.
As you can see in the table, the US Federal Accuracy standard gage block grades have new designations and
old designations and have appropriate accuracy (in length).

The old designation of AAA has a new designation of 0.5 and an accuracy of + or – 0.000001.

The old designation of AA has a new designation of 1 and an accuracy of + or –

0.000002.

The old designation of A+ has a new designation of 2 and an accuracy of + 0.000004 and -0.000002.
The old designation of A and B has a new designation of 3 and an accuracy of + 0.000008 and – 0.000004.

Monitoring Techniques – Gage Blocks Sets


Individual gage blocks may be purchased up to 20" in size. Naturally, the length tolerance of the
gage blocks increases as the size increases. Typical gage block sets vary from 8 to 81 pieces based
upon the needed application. The contents of a typical 81 piece set are:
Ten-thousandth blocks (9): 0.1001, 0.1002, ..., 0.1009
One-thousandth blocks (49): 0.101, 0.102, ................, 0.149
Fifty-thousandth blocks (19): 0.050, 0.100, 0.950
One inch blocks (4): 1.000, 2.000, 3.000, 4.000
For the purpose of stack protection, some gage manufacturers provide wear blocks that are
either 0.050" or 0.1 00" in thickness.
Calipers are used to measure length. The length can be an inside dimension, outside dimension,
height, or depth. Some calipers are used for only one of these lengths, while other calipers can
be used to measure all four types of lengths.

Calipers are generally one of four types:


• Spring calipers

• Dial calipers

• Vernier calipers

• Digital calipers

Monitoring Techniques – Spring Calipers & Vernier Caliper's


Spring callipers are transfer tools that perform a rough measurement of wide, awkward or
difficult to reach part locations.
Although these callipers are referred to as spring callipers, there are different varieties (spring
joint, firm joint, lock joint etc.) which describe the type of mechanical joint that connects the two
sides of the-unit. Vernier Callipers
Vernier callipers use a Vernier scale to indicate the measurement of length. Length, depth and height
are variations of the length measurement capability they provide.
Resolution of Vernier callipers is often 0.001 inch.

Vernier Scale & Dial Calipers The


Vernier Scale

Vernier scales are used on a variety of measuring instruments such as height gages, depth
gages, inside or outside Vernier callipers and gear tooth Vernier.
Except for the digital varieties, readings are made between a Vernier plate and beam scales.
Dial Callipers
Dial callipers functions in the same way as Vernier callipers, however the measurement is
indicated by a combination of a scale reading to the nearest 0.1 of an inch and a dial indicating
the resolution to 0.001 of an inch.
The dial hand typically makes one revolution for each 0.1 of an inch of travel on the calliper jaws.

Digital Calipers & Optical Comparators


Digital Calipers

Digital calipers use a-digital display instead of the dial and scale found in dial calipers. Most digital
calipers have the ability to be read in either inches or millimeters, and the zero point can be set
at any point along the travel. Display resolutions of 0.0005 of an inch are common. Errors in
reading the digital display are greatly minimized, however like the dial calipers, digital calipers
are often used in applications which require a different device to attain the required accuracy.
For example, some digital calipers have data interface capabilities to send measurement data
directly into a computer program. Digital caliper improvements have made them more reliable
for use in machine shop conditions including locations where cutting oil and metal chips come
incontact with the calipers.

Optical Comparators
A comparator is a device for comparing a part to a form that represents the desired part
contour or dimension. The relationship of the form with the part indicates acceptability.
A beam of light is directed upon the part to be inspected, and the resulting shadow is magnified
by a lens system, and projected upon a viewing screen by a mirror. The enlarged shadow image
can then be inspected and measured easily and quickly by comparing it with a master chart or
outline on the viewing screen. To pass inspection, the shadow outline of the object must fall
within the predetermined tolerance limits.

Monitoring Techniques – Surface Plates &

Micrometers Surface Plates

To make a precise dimensional measurement, there must be a reference plane or starting


point. The ideal plane for dimensional measurement should be perfectly flat.

Since a perfectly flat reference plane does not exist, a compromise in the form of a surface
plate is commonly used.
Micrometers
Micrometers or “mics,” are commonly’ used hand-held measuring devices.

Micrometers may be purchased with frame sizes from 0.5"

to 48".

Normally, the spindle gap and design permits a 1" reading span.

Thus, a 2" micrometer would allow readings from 1" to 2". Most common “mics” have an accuracy
of 0.001 of an inch.

Monitoring Techniques – Ring Gages and Plug Gages

Ring Gages
Ring gages are used to check external cylindrical dimensions and may also be used to check
tapered, straight or threaded dimensions.
A pair of rings with hardened bushings are generally used.
One bushing has a hole of the minimum tolerance and the other has a hole of the maximum tolerance.

Plug Gages
Plug gages are generally go/no-go gages and are used to check internal
dimensions. The average plug gage is a hardened and precision ground cylinder
about an inch long.

The go/no-go set is usually held in a hexagonal holder with the go plug on one end and the no- go
plug on the other end.
To make it more readily distinguishable, the no-go plug is generally made shorter.
Dial Indicators
Dial indicators are mechanical instruments for measuring distance variations.

Most dial indicators amplify a contact point reading by use of an internal gear train mechanism.
The vertical or horizontal displacement of a spindle with a removable contact tip is transferred to
a dial face.

Pneumatic Gages and Interferometry Pneumatic Gages

There are two general types of pneumatic amplification gages in use.


One type is actuated by varying air pressure and the other by varying air velocity at constant
pressure. Depending upon the amplification and the scale, measurements can be read to
millionths of an inch.
Interferometry
The greatest possible accuracy and precision are achieved by using light waves as a basis for
measurement.

A measurement is accomplished by the interaction of light waves that are 180°out of phase.
This phenomenon is known as interference. Interference occurs when two or more beams of
monochromatic light of the same wavelength are reunited after travelling paths of different
lengths.
When the light waves pass from a glass medium to an air medium above the surface of an
object, a 180°phase change takes place.
Laser Designed Gaging and CMM

Laser Designed Gaging

The use of lasers have been prevalent when the intent of inspection is a very accurate, non-
contact measurement. The laser beam is transmitted from one side of the gage to a receiver on
the opposite side of the gage.Measurement takes place when the beam is broken by an object
and the receiver denotes the dimension of the interference to the laser beam.

Coordinate Measuring Machines (CMM)


Coordinate measuring machines are used to verify work piece dimensions using computer
controlled measurements which are taken on three mutually perpendicular axes.
Work pieces are placed on a surface plate and a probe is manoeuvred to various contact points to
send an electronic signal back to the computer that is recording the measurements.

NDT/NDE and Visual Inspection


Non-Destructive Testing (NDT) and Evaluation (NDE)
Non-destructive testing (NDT) and non-destructive evaluation (NDE) techniques evaluate
material properties without impairing the future usefulness of the items being tested. Today,
there is a large range of NDT methods available, including ultrasonic, radiography, fluoroscopy,
microwave, magnetic particle, liquid penetrant, eddy current and holography.
Visual Inspection
One of the most frequent inspection operations is the visual examination of products, parts, and
materials.The colour, texture and appearance of a product gives valuable information if
inspected by an alert observer. Lighting and inspector comfort are important factors in visual
inspection. In this examination, the human eye is frequently aided by magnifying lenses or other
instrumentation.

Ultrasonic Testing
The application of high frequency vibration to the testing of materials is a widely used and
important non-destructive testing method. Ultrasonic waves are generated in a transducer and
transmitted through a material which may contain a defect. A portion of the waves will strike any
defect present and be reflected or “echoed” back to a receiving unit, which converts them into a
“spike” or “blip” on a screen.
Ultrasonic inspection has also been used in the measurement of dimensional thickness. One
useful application is the inspection of hollow wall castings, where mechanical measurement
would be difficult because of part interference.
The ultrasonic testing technique is similar to sonar. Sonic energy is transmitted by waves
containing alternate, regularly spaced compressions and refractions. Audible human sound is in
the 20 to 20,000 Hertz range. For non-destructive testing purposes, the vibration range is from
200,000 to 25,000,000 Hertz. (Where 1 Hertz = 1 cycle per second.)

Magnetic Particle Testing


Magnetic particle inspection is a non-destructive method of detecting the presence of many types
of defects or voids in ferromagnetic metals or alloys. This technique can be used to detect both
surface and sub-surface defects in any material capable of being magnetized.The first step in
magnetic particle testing is to magnetize a part with a high amperage, low voltage electric
current. Then fine, steel particles are applied to the surface of the test part. These particles will
align themselves with the magnetic field and concentrate at places where magnetic flux lines
enter or leave the part. The test part is examined for concentrations of magnetic particles which
indicate that discontinuities are present.

Liquid Penetrant Testing and Eddy Current Testing Liquid


Penetrant Testing

Liquid penetrant inspection is a rapid method for detecting open surface defects in both ferrous
and nonferrous materials. It may be effectively used on non-porous metallic and non-metallic
materials. Tests have shown that penetrants can enter material cracks as small as 3,000
angstroms.
Eddy Current Testing
Eddy currents involve the directional flow of electrons under the influence of an electromagnetic
field. Non-destructive testing applications require the interaction of eddy currents with a test
object. Eddy currents are permitted to flow in a test object by passing an alternating current
through a coil placed near the surface of the test object. Eddy currents are induced to flow, in any
part that is an electrical conductor. The induced flow of electrons produces secondary
electromagnetic field which opposes the primary field produced by the probe coil. This resultant
field can be interpreted by electronic instrumentation.

Radiography
Many internal characteristics of materials can be photographed and inspected by the
radiographic process. Radiography is based on the fact that gamma and X-rays pass through
materials at different levels and rates. Therefore, either X-rays or gamma rays can be directed
through a test object onto a photographic film and the internal characteristics of the part can be
reproduced and analysed.Because of their ability to penetrate materials and disclose subsurface
discontinuities, X- rays and gamma rays have been applied to the internal inspection of forgings,
castings, welds, etc. for both metallic and non- metallic products.

Neutron Radiograph
Neutron radiography is a fairly recent radiographic technique that has useful and unique
applications. A neutron is a small atomic particle that can be produced when a material, such as
beryllium, is bombarded by alpha particles.
Neutrons are uncharged and move through materials unaffected by density. When X-rays pass
through an object, they interact with electrons. Therefore, a material with a high electron density,
such as lead, is nearly impenetrable. N-rays, on the other hand, are scattered or absorbed by
particles in the atomic nuclei rather than by electrons. A metal that is opaque to X- rays is nearly
transparent to N-rays. However, materials rich in hydrogen or boron, such as leather, rubber,
plastics and many fluids are opaque to N-rays. The methods used to perform neutron radiography
are fairly simple. The object is placed in a neutron beam in front of an image detector.

Related Techniques
There have been new developments in the radiographic field of non-destructive testing, several
common recent applications include fluoroscopy, gamma radiography, televised x-ray (TVX),
microwave testing and holographic inspection.
Force Measurement Techniques
A brief description of common force measurement tests is listed below.
Tensile Test
Tensile strength is the ability of a metal to withstand a pulling apart tension stress.
Sheer Test
Shear strength is the ability to resist a “sliding past” type of action when parallel, but slightly off-
axis, forces are applied.

Compression Test
Compression is the result of forces pushing towards each other. The compression test is run
much like the tensile test.
Titration and Hardness Measurement Titration
A titration is a method of analysis that allows determination of the precise endpoint of a reaction
and therefore the precise quantity of reactant in the titration flask. A buret is used to deliver the
second reactant to the flask and an indicator or pH Meter is used to detect the endpoint of the
reaction.

Hardness Measurement
Hardness testing (which measures the resistance of any material against penetration) is
performed by creating an indentation on the surface of a material with a hard ball, a diamond
pyramid or cone and then measuring the depth of penetration. Hardness testing is often
categorized as a non- destructive test since the indentation is small and may not affect the future
usefulness of thematerial.

Brinell Hardness Testing


The Brinell hardness testing method is primarily used for bulk hardness of heavy sections of softer
steels and metals. Compared to other hardness tests the imprint left by the Brinell test is
relatively large. This type of deformation is more conducive to testing porous materials such as
castings and forgings.
Vickers Hardness Testing
Vickers hardness testing uses a square-based pyramid with loads of 1 to 120 kg. The surface
should be as smooth, flat and clean as possible.

Mohs Hardness Testing


In 1824, an Austrian mineralogist by the name of F. Mohs chose ten minerals of varying hardness
and developed a comparison scale. This scratch test was probably the first hardness testing
method developed. It is very crude and fast and is based on the hardness of ten minerals.
File Hardness Testing
File hardness is a version of the scratch testing method where a metal sample is scraped with a
1/4" diameter double cut round file. If the file “whites” into the material, the material is “not file
hard.” lf there is no mark, the material is “file hard.”
Sonodur Hardness Testing Method
The Sonodur is one of the newer test methods and uses the natural resonant frequency of
metal as a basis of measurement.

Shore Scleroscope Hardness Testing


The Shore Scleroscope is a dynamic hardness test that uses a material’s absorption factor and
"measures the elastic resistance to penetration. It is unlike the other test methods- in that there
is no penetration. In the test, a hammer is dropped and the bounce is determined to be directly
proportional to the hardness of the material. The Shore method has a negligible indention on the
sample surface. A variety of materials, shapes, and sizes can be tested, and the equipment is very
portable.
Torque Measurement
Torque measurement is required when the product is held together by nuts and bolts. The wrong
torque can result in the assembly failing due to a number of problems. Parts may not be
assembled securely enough for the unit to function properly or threads may be stripped because
the torque was too high, causing the unit to fail. Torque is described as a force producing rotation
about an axis.
The formula for torque is:

Torque = Force x
Distance

Torque is measured by a torque wrench. There are many types of torque wrenches. However,
the two types most commonly used are the flexible beam" type, and the rigid frame type. Torque
wrenches may be preset to the desired torque. The wrench will either make a distinct “clicking”
sound or “slip” when the desired torque is achieved.

Impact Test and Steel Rule

Impact Test:
Impact strength is a material's ability to withstand shock. Tests such as Charpy and lzod use
notched samples which are struck with a blow from a calibrated pendulum.
The Steel Rule:
The steel rule is a widely used factory measuring tool for direct length measurement. Steel
rules and tapes are available in different degrees of accuracy and are typically graduated on
both edges.
MODULE 3.0 ANALYZE PHASE

3.0 Analyze Phase – Overview and Objectives

The Analyze phase of DMAIC helps project teams identify problems in the production process that
cause product defects. This phase of Six Sigma methodology is loaded with tools to help spot the
problems in the production process and to determine if these problems are the root causes of
defects. The ANALYZE phase is the beginning of the statistical analysis of the problem.

3.1 Pattern of Variation Multi-Vari chart

Multi-Vari charts are a way of presenting variance data in a graphical form. This helps to
carry out an investigation and study patterns of variation from many possible causes on a
singlechart
Graphically represent relationship between factors and response
Example: Strength by time - Metal type data
Time Metal Strengt Used to identify possible variable ( X) or families of variation such as
type h variation within piece/ transaction or Positional variation , piece to piece/
transaction to transaction or Cyclical variation over time
0.5 15 23
Used to analyze the effects of categorical inputs on continuous output or
0.5 15 20
Temporal variation.
0.5 15 21

0.5 18 22

0.5 18 19

0.5 18 20

0.5 21 19

0.5 21 18

0.5 21 21

1 15 22

1 15 20

1 15 19

1 18 25

1 18 25

1 18 22

1 21 20

1 21 19

1 21 22

2 15 18

2 15 18

2 15 16

2 18 21

2 18 23

2 18 20

2 21 20

2 21 22

2 21 24
3.1.2 Classes of Probability Distribution

In probability theory and statistics, a probability distribution is a mathematical function that provides
the probabilities of occurrence of different possible outcomes in an experiment.
A probability distribution tells you what the probability of an event happening is. Probability
distributions can show simple events, like tossing a coin or picking a card. They can also show much
more complex events, like the probability of a certain drug successfully treating cancer.
Binomial Distribution: This is the probability distribution for discrete data .Discrete data is
information that can be categorized into a classification. Discrete data is based on counts. Only a
finite number of values is possible, and the values cannot be subdivided meaningfully. For example,
the number of parts damaged in shipment. It is typically things counted in whole numbers.
It is an application of the population knowledge to predict the sample behavior
A process is called a Bernoulli processes when

▪ Process output has only two possible values (Defective / Non Defectives, Pass / Fail, Yes
/ No)

▪ The Probability of each outcome remains constant over time. i.e. Outputs are
statistically independent
The Mean of a Binomial Distribution = np
The Standard Deviation of a Binomial Distribution, σ= n p (1 – p)
Binomial distribution is measured on Defectives and not on Defects
It is most suitable when n< 30 & p> 0.1
It gives the percentage of non-defective items, provided the probability of creating a defective
item remains the same over time.
Applies when populations is large (N>50) & sample size is small when compared to population
Approximate to hyper-geometric distribution
Best applied when sample size is less than 10% of N(n<0.1N)
For example, A manufacturing process creates 3.4% defective parts. A sample of 10 parts from
the production process is selected. What is the probability that the sample contains exactly 3
defective parts?

SOLUTION:
There are two outcomes: Defective / Not-Defective, therefore the Binomial Distribution equation
is applied.

p = 0.034 n = 10 then q = 1-p = 1 -


0.034 = 0.966 (96.6%) Need the
probability that x = 3.

Substitute the values into the Binomial Probability Function and solve:

10!
P(x) = (0.034 )(03.966 ) 7

3!(10−3)!

P(x) = 120(0.0343)(0.9667)

P(x) = 003702 = 0.372%

Poisson Distribution: This is also a probability distribution for discrete data


It is an application of the population knowledge to predict the sample behavior
Unlike Binomial distribution that deals with discrete binary data, a Poisson distribution deals

with integers that can take any value

This distribution is generally used for describing the probability distribution of an event that
tend to occur at predictable intervals over time
It is suitable for analyzing situations where the no. of trials is very large (tending towards
infinity) and the probability of success in each trial is very small (tending towards zero).
It is applicable for predicting the occurrence of relatively rare events like plane crashes, car
accidents etc. and is, therefore, used in the Insurance industry
It can be used for prediction of no. of defects, if the defect occurrence rate is low

Closely related to exponential distribution


Can be an approximation to binomial distribution when p is less than or equal to 0.1, and
sample size is fairly large

Unlike the Binomial Distribution that has only two possible outcomes as a success or fail, this
distribution focuses on the number of discrete occurrences over a defined interval. It is used to
estimate the probabilities of events that occur randomly per some unit of measure.
This formula describes rare events and is referred to as the law of improbable events.

The Poisson Distribution may apply when studying the:

o number of customers per minute in a bookstore


o number of transactions per hour at a bank o number of
long distance telephone calls per day o number of cracks
per windshield
o number of cars passing through an intersection per day o
number of late shipments per 1,000 shipments o number
of bugs per byte of code
o number of pieces scrapped per 1,000,000 pieces
produced o number of planes arriving per hour at airport
o number of sick days per month (or vacation days per) o
number of vacant houses per county in a state o number
of defects found per form o defects per unit

For example, A new website has an average random hit rate of 2.9 unique visitors every 4
minutes. What is the probability of getting exactly 50 unique visitors every hour?
Given information:
Chi Square Distribution :
It is one of the most widely used probability distribution measure in inferential statistics. The Chi
Square Test is a statistical test which consists of three different types of analysis 1) Goodness of fit,
2) Test for Homogeneity, 3) Test of Independence.

The Test for Goodness of fit determines if the sample under analysis was drawn from a population that
follows some specified distribution.

The Test for Homogeneity answers the proposition that several populations are homogeneous with
respect to some characteristic.

The Test for independence (one of the most frequent uses of Chi Square) is for testing the null
hypothesis that two criteria of classification, when applied to a population of subjects are
independent. If they are not independent then there is an association between them.

Where,

Chi-Square - Goodness of Fit


For instance a problem that starts with "a manager thinks that 50% of the company's employees
were educated on the east coast and 50% were educated on the the west coast," is the type of claim
that can be tested using a Chi- square goodness of fit test.

If we know that there are 100 employees then our expected cell counts would be 50 east coast
employees (100 *
.50) and 50 employees from the west coast (also 100*.50). Notice we had to know our total number
of employees to convert our hypothesis (a 50-50 split) into real numbers. Those real numbers
became our expected cell counts. (There are two cells - (1) east coast employees and (2) west cost
employees).

A hypothesis such as "the manager expects that more employees educated on the west coast than
the east coast" would not work. The hypothesis has to specify exact numbers for the expected cell
counts. Once we have used our theory to determine the expected cell counts, we use our actual
data to see what the actual cell counts are. For instance, if the data from this firm said that 44
employees were educated on the west coast and 56 employees were educated on the east coast
then we could then create the following table

East We
Co ast st
Coa
st
Actual Cell Count 56 44

Expected Cell (50 (50)


Count )

Using this table we can ask and answer the question as to whether the manager was correct in
his/her hypothesis that 50% of the employees were educated on the East Coast and 50% were
educated on the West Coast. We follow the standard hypothesis testing procedure.

Hypothesis: H0: The data fits the model, i.e., a 50-50 split; H1: the data does not fit the model.
Critical Value: Chi-square critical values for the goodness of fit test can be obtained from the chi-
square table (back page of book). The test is always right tailed, thus there is no need to divide
alpha by 2. DF are obtained by taking the number of categories and subtracting 1. For this case,
we have two categories and thus the df are (2-1)=1 and alpha = .05, so the critical value is 3.84146
Calculated Value: the Chi-square calc. is obtained by taking the (actual-expected)sqrd/expected
for each cell in our problem. Add these up and you have chi-square calc. In this case you have 2
cells, (1) (56-50)sqrd/50 = (6)sqrd/50 = 36/50 = .72. For cell (2) it equals (44-50)sqrd/50 = (-
6)sqrd/50 = 36/50 = .72. Add cell one and cell two and we get
.72 + .72 = 1.44. This is Chi-square calculated.

Compare: The chi-square critical (3.84)is larger than the chi-square calc.(1.44), thus we do not reject
the null. (This uses the same rule we always use on a right tailed test; calc < crit = accept, calc > crit
= reject.)
Conclusion: If we accepted the null and the null was that the data fits the model, then we can
conclude that the manager's hypothesis that the employees were educated in equal numbers from
the east and west coast was supported.

Chi-Square - Test of Independence

This use of Chi-square is somewhat different. In this case we have two (or more) variables, both of
which are categorical, and we want to determine if they are independent or related. In these
problems we first lay out our actual or observed data and then calculate the expected cell
frequencies. Let's look at an example.

EXAMPLE:
A manger wants to see if geographical region is associated with ownership of a Macintosh computer.
The manager surveys 100 people and the data breaks down as follows:

M N Row
ac o total

M
ac

North East 1 1 26
2 4
South 2 1 39
West 1 8
Mid 1 1 35
West 7 8
Colum 5 5 100
n Total 0 0
Our next step is to calculate the expected cell frequency. The formula for each cell is (row
total*column total)/grand total. The grand total equals the total number of individuals that make
up the sample. We have to repeat this for each cell. Starting with North East - Mac Owners we would
have (26*50)/100 = 13. We have five more cells to do this for. We put our expected cell frequencies
in parantheses below our actual cell values for each cell. The table below then is our completed
data table:

Mac No Mac Row total

North East 12 (13) 14 (13) 26

South West 21 18 39
(19.5) (19.5)

Mid West 17 18 35
(17.5) (17.5)

Column Total 50 50 100

Note: in this case because the column totals are both 50, the expected cell frequencies in the rows are
equal because the column total in each equation is 50 - this is just the way these data worked out and
is not always the case.

Now that we have the table worked out we can run through our 5-step hypothesis testing procedure
to determine if owning a mac and the geographical region where the owner lives are related.

Hypothesis: H0: the two variables are independent, H1: The two variables are not independent
(they are related). Critical value: Again look to the Chi-square table and again do not divide alpha
by 2. The df for this test are (rows- 1)(columns -1). In this case we have 3 rows and 2 columns, thus
df = (3-1)(2-1) = 2. Thus chi-square with 2 df at alpha = .05 is 5.99
Calculated value: The sum of (actual-expected)sqrd/expected for each cell. In this case we have 6 cells,
so we have to do this formula six times and sum the answers.

Cell 1 = (12-13)sqrd/13 = 1/13 = .077.

Cell 2 = (14-13)sqrd/13 = .077.

Cell 3 =(21-19.5)sqrd/19.5 = .115

Cell 4 = (18-19.5)sqrd/19.5 = .115


cell 5 = (17-17.5)sqrd/17.5 = .014

and Cell 6 = (18-17.5)sqrd/17.5 = .014.


Sum up all of these and you get: .077+.077+.115+.115+.014+.014=.412 . This is our calculated
value of chi-square statistic.
Compare: Chi-sqr calc is less than chi-sqr crit so we do not reject the NULL.
Conclusion: The Null was that the two variables were independent and that hypothesis was
accepted. The managerial conclusion is that ownership of a mac and geographical region are NOT

related (i.e., they are independent). Had we rejected the null hypothesis (i.e., chi-square calc > chi-
square crit), we would have concluded that mac onwership and geographical region are related in
some way (i.e., not independent).

T Distribution
F Distribution
The main use of F-distribution is to test whether two independent samples have been drawn for
the normal populations with the same variance, or if two independent estimates of the
population variance are homogeneous or not, since it is often desirable to compare two variances
rather than two averages.

✓ The F distribution is a ratio of two Chi-square distributions,and a specific F distribution is


denoted by the degrees of freedom for the numerator Chi-square and the degrees of freedom
for the denominator Chi- square
✓ F-test is performed to assess whether the standard deviations/ variances of two processes are
significantly different
✓ Often,project teams target the variance to be reduced, of the process

✓ Where S1 & S2 = standard deviations of two samples, S1 > S2 (numerator should be greater than
denominator),
df1 = n1 -1 & df1 = n2 -1
✓ From the F table one can find out F critical at α and degrees of freedom of samples of wto
different process (df1 and df2)
F critical = F α, df1, df2

Suppose you randomly select 7 women from a population of women, and 12 men from a population
of men. The table below shows the standard deviation in each sample and in each population.

Po Population standard Sample standard


pu deviation deviation
lat
io
n
W 30 35
o
m
en

M 50 45
en

Compute the f statistic.

Solution A: The f statistic can be computed from the population and sample standard deviations, using
the following equation:
f = [ s12/σ12 ] / [ s22/σ22 ]

where σ1 is the standard deviation of population 1, s1 is the standard deviation of the sample drawn
from population 1, σ2 is the standard deviation of population 2, and s1 is the standard deviation of
the sample drawn from population 2.

As you can see from the equation, there are actually two ways to compute an f statistic from these
data. If the women's data appears in the numerator, we can calculate an f statistic as follows:
f = ( 352 / 302 ) / ( 452 / 502 )
f = (1225 / 900) / (2025 / 2500)
f = 1.361 / 0.81 = 1.68

For this calculation, the numerator degrees of freedom v1 are 7 - 1 or 6; and the denominator degrees
of freedomv2 are 12 - 1 or 11.
On the other hand, if the men's data appears in the numerator, we can calculate an f statistic as follows:
f = ( 452 / 502 ) / ( 352 / 302 )
f = (2025 / 2500) / (1225 / 900)
f = 0.81 / 1.361 = 0.595

For this calculation, the numerator degrees of freedom v1 are 12 - 1 or 11; and the denominator degrees
of freedom v2 are 7 - 1 or 6.

3.2 Inferential Statistics

3.2.1 Understanding Inferences

Inferential statistics makes inferences about populations using data drawn from the population.
Instead of using the entire population to gather the data, the statistician will collect a sample or
samples from the millions of residents and make inferences about the entire population using
the sample. Inferential statistics is a set of methods used to draw conclusions or inferences about
characteristics of populations based on data from a sample.

Whenever we execute a project, it has to be managed under many constraints such as time,
cost, resources, among others. Thus, it may not be always feasible for the project to study
100%
of the population to derive its inferences. For example, if we are improving the quality of
ammunition manufactured in an ammunition factory, we may not be able to do quality test of
100% of the products. This is mainly because the product will get destroyed after testing. Thus,
sampling is used in these cases where only a sample of products is taken in for quality
and inferences are made for the testing
population basis the result of this sampling.

Some other examples of sampling include manufacturing of cars in specific lots i.e. a car
manufacturing company manufactures its cars in lots. If it is a lot of 400 cars, they will only
test 10 – 15 cars and make an inference of whether to accept the lot or reject it.

Sampling helps in managing the project by utilizing lesser resources and is still effective
in getting results.

– The mean calculated for a population

ο – The standard deviation calculated


for a population

As you would see in the figure, inferences are not taken out of the population “N” but a sample
which is a subset of the data representing the population “n”.
The objective of statistical inference is to draw conclusions about population characteristics
based on the information contained in a sample.
The steps involved in statistical inference are:
Define the problem objective precisely
Decide if the problem will be evaluated by a one-tail or two-tail test
Formulate a null hypothesis and an alternate hypothesis
Select a test distribution and a critical value of the test statistic reflecting the
degree of uncertainty
Calculate a test statistic value from the sample information
Make an inference about the population by comparing the calculated value to
the critical value
Communicate the findings to interested parties
Everyday, in our personal and professional lives, individuals are faced with decisions between
choice A or choice B. In most situations, relevant information is available; but it may be
presented in a form that is difficult to digest. Quite often, the data seems inconsistent or
contradictory. In these situations, an intuitive decision may be little more than an outright
guess. While most people feel their intuitive powers are quite good, the fact is that decisions
made on gut-feeling are often wrong.

3.2.2 Sampling Techniques and Uses

Sampling is the process of selecting a small number of elements from a larger defined target
group of elements.

• Population is the total group of elements we want tostudy

• Sample is the subgroup of the population we actuallystudy

• Sample would mean a group of ‘n’ employees chosen randomly from organization
of population ‘N’

• Sampling is done in situations :

• When the process involves destructive testing, e.g. taste tests, car crash tests, etc.

• When there are constraints of time and costs

• When the populations cannot be easily captured


Sampling is NOT done in situations:
• When the events and products are unique and cannot be replicable

Sampling Techniques can be classified into:

1. Probability Sampling: Probability sampling is when there is a probability of an event to


occur. Probability sampling techniques use the science of Probability for picking samples
and arriving at sample size.

2. Non Probability Sampling: Non probability sampling does not depend on the chance cause
for an event to occur. Non-probability sampling techniques are used as per convenience
andrequirement.

Probability Sampling includes:

1. Simple Random Sampling

2. Stratified Random Sampling

3. Systematic Sampling

4. Cluster Sampling

1. Simple Random Sampling:

It is a method of sampling in which every unit has equal chance of being selected.The process
of selecting samples in random & without any particular order or classification. This is the most
used method of all methods.

2. Stratified random sampling:

It is a method of sampling in which stratum/groups are created and then units are picked randomly.
Members of the population are put in some order. A starting point is selected at random, and every
n^{\text{th}}nthn, start superscript, t, h, end superscript member is selected to be in the sample.
Dividing the population into subgroups of interest and sampling either sequentially or randomly
within each subgroup. This is important to make sure there is representation from all
stratifications in the population.

A subgroup may be data taken at certain temperature range, specific shift, under certain
pressure, different machine groups, slower speed versus higher speed, and other different
conditions.

3.Systematic Sampling

Systematic sampling is a method of sampling in which every nth unit is selected from the population
Acquiring data at specified intervals such as every hour, every 5th form, or on a particular shift.
Ensure the interval does not introduce a pattern that may be biased to a specific person, machine,
or part each time the data point is collected.

An appropriate and disciplined plan needs to be clearly understood by those collecting the data.
Since the collection process can be expensive and time consuming there may be bias introduced
by people making educated guesses, predictions of data, and collecting data that is convenient
and simple.

When your process has a continuous flow (no lots or batches) then the most representative
sampling is to pull your samples on a constant count or time basis

4.Cluster sampling:

Cluster sampling is a method of sampling in which clusters are sampled every tth time.
The population is first split into groups. The overall sample consists of every member from some
of the groups. The groups are selected at random.Example—An airline company wants to survey
its customers one day, so they randomly select 555 flights that day and survey every passenger
on those flights.
Why it's good: A cluster sample gets every member from some of the groups, so it's good when
each group reflects the population as a whole.
No units from non-selected clusters are included in the sample. They are represented by those
from selected clusters.
This differs from stratified sampling, where some units are selected from each group. Examples
of clusters may be factories, schools and geographic areas such as electoral subdivisions.
Cluster sampling and stratified sampling are two very different sampling methods. With
cluster sampling, one can

• Divide the population into groups (clusters)


• Obtain a simple random sample of so many clusters from all possible clusters
• Obtain data on every sampling unit in each of the randomly selectedcluster

Non Probability Sampling includes:

1. Convenience Sampling

2. Judgment Sampling

3. Quota Sampling

4. Snowball Sampling

Non Probability Sampling techniques

1. Convenience, haphazard or accidental sampling members of the population are chosen

based on their relative ease of access. To sample

friends, co- workers, or shoppers at a single mall, are all examples of convenience sampling.

2. Judgmental sampling or purposive sampling - The researcher chooses the sample based on
who they think would be appropriate for the study.

3. In quota sampling, a population is first segmented into mutually exclusive sub-groups, just
as in stratified sampling. Then judgment is used to select the subjects or units from each
segment based on a specified proportion.

4. Snowball sampling (or chain sampling, chain-referral sampling, referral sampling[) is a


technique where existing study subjects recruit future subjects from among their
acquaintances. Thus the sample group appears to grow like a rolling snowball. As sample
members are not selected from a sampling frame, snowball samples are subject to numerous
biases.
Sample size calculation
Determining sample size is a very important issue because samples that are too large may waste
time, resources and money, while samples that are too small may lead to inaccurate results. In
many cases, we can easily determine the minimum sample size needed to estimate a process
µ
parameter, such as the population mean

When taking a sample from a larger population you must make sure that the samples are an
appropriate size and are sampled without bias.

For example, it is very helpful if the sample size is large enough for the data to follow normal
distribution as this opens the door to use an array of statistical tools.

Sampling Error

The error arising from drawing inferences on the basis of observations on a part(sample) is
termed as Sampling Error. It decreases with increase in sample size does not result in substantial
reduction in error. The optimum sample size is worked out based on this behavior,taking into
account the required precision and cost consideration.
3.2.3 Central Limit Theorem (CLT)

The Central Limit Theorem (CLT) tells us that the sampling distribution of the sample mean is
normally distributed, regardless of the distribution of the underlying random sample.
In fact, the CLT applies regardless of whether the distribution of the Xi is discrete (for example,
Poisson or binomial) or continuous (for example, exponential or chi-square).
The distribution of average values of samples (multiple X bars) drawn from that universe
will tend toward a normal distribution as the sample size grows without bound with mean
μ and standard deviation σ/sqrt(n).
The key to this theorem is the whole s2/ n part of the formula. As n, sample size, increases we
see s2, the variance, decrease. And less variance means a tighter, more normal, distribution.

Inferential Statistics – Central Limit Theorem


If a random variable, X, has mean µ, and finite variance σ2, as n increases, X approaches a normal
distribution with mean µ anXd variance Xσ 2. Where, σ 2 = σ2/n and n is the number of observations
on which the mean is based.
The Central Limit Theorem states:
The sample means Xi will be more normally distributed around µ than individual readings Xi.
The spread in sample means Xi is less than Xj; with the standard deviation of Xi equal
to the standard deviation of the population (individuals) divided by S the square root
of the sample size.
In this picture, we generate two populations using Minitab. One population is normal with mean
= 0 and standard deviation=1; the other population is a log normal distribution with mean=1.5
and standard deviation =1.8. By randomly sampling these distributions, we can construct
sampling distributions of the mean.

For the normal population, all of the sampling distributions are “normal.” For the log normal
population, the sampling distribution does not become “normal” until the sample size is n=30.
This is the crux for the Central Limit Theorem.

Sampling Distributions
Central Limit Theorem (CLT)

3.3 Hypothesis Testing

3.3.1 General concepts and Goals of Hypothesis Testing

Hypothesis Testing is a powerful statistical tool used to validate the root causes for the
problem. Doing a Hypothesis Test is just like conducting a trial in a court of law. We assume
that there is no change (the convict is innocent) in the outcome of a process if the input is
changed .If we want to prove that there is change (the convict has committed the crime) in
the outcome of the process, because of the change in the input, we need to collect the data
and prove with evidence that there is a change.
If it is proved that the change in input has caused a significant change in the output, then this
input is validated as one of the significant X
Key points about Hypothesis Testing
A Hypothesis is a statement about a population parameter derived from one ormore
populations.
Hypothesis testing is a statistical analysis method, wherein a hypothesis is stated,
sample data are collected, and a decision is made, based on the sample data and
related probability value.
This testing can be used to detect differences such as differences between a process mean
and the target for the process or differences between multiple employees.
Hypothesis Testing helps you determine if one population (or sample) of items is
statistically different from another.

Hypothesis Testing Methodology


Null Hypothesis (H0): The null hypothesis is the hypothesis to be tested. This is a statement
of what is to be disproved.

Alternative Hypothesis (H1): The alternative hypothesis is the one accepted if the null
hypothesis is rejected. This is a statement of what is to be proved

The hypothesis will focus on a parameter of the population such as the Mean, Standard
Deviation, Variance, Proportion or Median.
The type of hypothesis test that could be conducted is based on the data type (Discreteor
Continuous) of the Y data.
Hypothesis Testing – Steps

• Specify the Ho (Null hypothesis) and Ha(Alternative Hypothesis)

Ho: The Hypothesis you are trying to

challenge Ha: The Hypothesis you have in

mind

* Setting Ho and Ha differs across hypothesis tests.

• Setting Alpha level

Since the true state of nature is seldom known with 100%

certainty, Concluding either Ho or Ha results in some probability

of error.

“Alpha” risk (α) is the maximum tolerable risk of concluding a difference when , in reality,
such a difference does not exist.
α is usually set at .01 or .05,1 – α is the confidence level
• Calculate the P-value
Statistical definition of P-value:
The probability of getting the results(data) you have got, if the Null Hypothesis(Ho) is true.
Pratical definition of P-value:
The p-value is the analyst’s confidence towards the Null
Hypothesis Hence;

If p-value is low -> p-value < 𝛼𝛼𝛼𝛼, we reject the null

hypothesis(reject Ho) If p-value is high -> p-value > 𝛼𝛼𝛼𝛼, we keep

the null hypothesis(keep Ho)

• proving or Disproving the Hypothesis

Because of the way they are set up, if you decide that there is a difference between two sets of
ata(rejecting Ho), a hypothesis DOES NOT tell you how big that difference is, but only that is
there.
This is because you are not PROVING the alternative Hypothesis(Ha), but just finding enough
evidence to Disaprove the null hypothesis(Ho).
3.3.2 Significance: Practical vs Statistical

Statistics provide a methodology to detect differences. Examples might include differences in


suppliers, shifts or equipment. Two types of significant differences occur and must be well
understood, practical and statistical.
Failure to tie these two differences together is one of the most common errors in statistics.
Significant differences (i.e., reject the null hypothesis) means that differences in group means
are not likely due to sampling error. The problem is that statistically significant differences can
be found even with very small differences if the sample size is large enough.
In fact, differences between any sample means will be significant if the sample is large enough.
For example – men and women have different average IQs. A difference or change to the
process that probably (with some defined degree of confidence) did not happen by chance.
Examples might include differences in suppliers, markets or servers.

Practical (or clinical) significance asks the larger question about

differences “Are the differences between samples big enough to have

real meaning.”

Although men and women undoubtedly have different IQs, is that difference large enough to have some
practical implication.

The difference which results in an improvement of practical or economic value to the company.

Example, an improvement in yield from 96 to 99 percent.

Acceptance / Rejection Criteria for Hypothesis


A null hypothesis can be accepted / rejected by using 3 methods

In critical value method, we reject null hypothesis when the calculated value is greater than the
tabular critical value for the corresponding distribution. We do not reject null hypothesis when the
calculated value is not greater than the tabular critical value for the corresponding distribution.
In probability method, the null hypothesis is rejected when the p-value is less than alpha. We
fail to reject null hypothesis when p-value is not lesser than alpha.
In confidence interval method, we reject the null hypothesis when the hypothesized parameter
value is not in the calculated confidence interval. Whereas, we do not reject the hypothesis
when the hypothesized parameter value is within the calculated confidence interval.

3.3.3 Risk: Alpha and Beta

Type I Error or Alpha Error: It is the probability of rejecting the null hypothesis when it is true.
A typical Alpha risk is 5%.
If the calculated p-value (probability value) is less than 5%, the analysis will reject the
null hypothesis.

Alpha risk is the risk of incorrectly deciding to reject the null hypothesis. If the confidence interval
is 95%, then the alpha risk is 5% or 0.05.

For example, there is a 5% chance that a part has been determined defective when it actually is
not. One has observed or made a decision that a difference exists but there really is none. Or when
the data on a control chart indicates the process is out of control but in reality the process is in
control.

Alpha risk is also called False Positive and Type I Error.

Confidence Level = 1 - Alpha Risk

Alpha is called the significance level of a test. The level of significance is commonly between 1%
or 10% but can be any value depending on your desired level of confidence or need to reduce Type
I error.
Selecting 5% signifies that there is a 5% chance that the observed variation is not actually the truth.

The most common level for Alpha risk is 5% but it varies by application and this value should be
agreed upon with your Black belt/ Master Black Belt.

In summary, it's the amount of risk you are willing to accept of making a Type I error.

If a carbon monoxide alarm goes off indicating a high level alert but there is actually not a high
level then this is Type I error.

If conducting a 2-sample T test and your conclusion is that the two means are different
when they are actually not would represent Type I error.
The a-level is represented by the clouded

Sample results in this area lead to rejection

Region of Region of
DOUBT DOUBT

areas. of

H0

Type II Error or Beta Error: It is the probability of failing to reject the null hypothesis when it is
actually false.
The beta error is a function of the sample size.
One minus beta is called the power.
Power is the ability of the test to detect a difference, if oneexists.
Beta risk is the risk that the decision will be made that the part is not defective when it
really is. In other words, when the decision is made that a difference does not exist when
there actually is. Or when the data on a control chart indicates the process is in control
but in reality the process is out of control.
If the power desired is 90%, then the Beta risk is 10%.
There is a 10% chance that the decision will be made that the part is not defective when
in reality it is defective.

Power = 1 - Beta risk

Beta risk is also called False Negative and Type II Error.

The Power is the probability of correctly rejecting the Null Hypothesis.

The Null Hypothesis is technically never proven true. It is "failed to reject" or "rejected".
"Failed to reject" does not mean accept the null hypothesis since it is established only to be
proven false by testing the sample of data.

Guidelines: If the decision from the hypothesis test is looking for:

• Large effects or LOW risk set Beta = 15% (which is Power of 0.85)
• Medium effects, MEDIUM risk but not catastrophic, legal or safety related the set Beta= 10%
• Small effects, HIGH risk, legal, safety, or critical set Beta from 5% to near 0%.

If conducting an F-test and your conclusion is that the variances are the same when they are
actually not would represent a Type II error.

Same note of caution as for Alpha, the assumption for Beta should be agreed upon with your
BB/MBB. Example include ,Fire breaks out but fire alarm doesn’t ring, A survey showing that
stress doesn’t hamper one’s life when it actually does.

Reject H0 α = Pr(Type I error)

Accept Ho β = Pr(Type II
error)

3.3.4 Types of Hypothesis tests

We use the following grid to select the appropriate hypothesis test depending on the data types:
Output ( Dependant Variable) Y
Selectthe
appropriate lest
Continuous Discrete
Re
Continu Logistic
Simple Linear
os gression
Regression
Correlat ion
l put Normal Data
(Independe T Test (1,2 S.imple &
nt Paired)
Variable)
X's
Discrete ANOVA, F test , HOV Chi Square
Non-Normal Data
Moods Median , HIOV

Objective Type of Test Assumption


Normality Anderson-Darling
Test Normali Normality Test F-Test Bartlett's Test Leve ty Not
Assume ne 1s Test

d 2 Sample T-Test Paired T-Test


Equality of
Variance Analysis of Variance (ANOVA) Mann-Whitney Test

Equalit
Kruskal- Wallis Test
Equality of y of
Means - Propor
Mood's Median Test tion
Normality 2 Sample PTest Chi-Square Test
Assumed

Equality of M eans -
Data from s Data from Non-
contin
Normal Groups
uous D Data from 2 different continuous data
Normal Data sets
set a
Data from same continuous data set

D t treated with 2 conditions,


distribution of logical differences
a a
should be Normal
t Data from >2 different continuous a f Normal data sets; Variances
across data sets are not
r
significantly different f o

Data from 2 different continuous


r m Non-Normal data sets

o
Data from >2 different m > continuous Non-Normal data

2 sets without any obvious outliers


Data from >2 different continuous Non-
2 Normal data sets with
N any obvious outliers

Data from 2x2 cross-tabulated


o
N discrete data sets o r Data from row•column

cross-

r m tabulated discrete data sets; eachcell frequency should be> S

m aa l

l
G

G r

r o

o u

u p

p s

Situatio n Statiistical Test

Comparing a population mean against a giv e n 1-sampl1 et-Test:


standard For exa m p le : Is the mean TAT of thread ns
m inut e s .
Comparing means of t w o different populations 2-sampte t-Test
For exa m p le : Is the mean of morning shift=
mean ofnight sh ift .
Comparing the means of more than t wo populations ANOVA
For exa m p le : Is the mean of staff A = mean of staff B =
mean of staff C.
Comparing the va ria nce oft wo or more than t wo Homogeneity
populations For exam p le : Is the va riat io n of staff Of v .aliiance
A = v ariat ion of staff B
= va riat ion of staff C.

Normal Continuous Y and Discrete X

Non-Normal Continuous Y and Discrete X

Continuous Y and Continuous X

Discrete Y and Discrete X


3.4 Hypothesis Testing with Normal

Data Analyze Phase - Overview

3.4.1 One sample t-test

The One Sample t Test is commonly used to test the following: Statistical difference between a
sample mean and a known or hypothesized value of the mean in the population. Statistical
difference between the sample mean and the sample midpoint of the test variable.Use the one-
sample variance test to compare the variance of a sample to a reference.

Example: Does the mean of the sample meets the target value of 2650?

Ho : Mean = 2650 Ha : Mean # 2650

One-Sample T: Mean
Test of mu = 2650 vs not = 2650
Variable N Mean StDev SE Mean 95% CI T P
Mean 10 2648.40 16.78 5.31 (2636.40, 2660.40) -0.30 0.770

Since P< 0.05, fail to reject Null Hypothesis

i.e. Ho holds good, Mean meets the target value of 2650


Sample data 2661
2624
2641
2670
2650
2638
2645
2652
2628
2675

Example: Paired t-test


The Paired Samples t Test compares two means that are from the same individual, object, or
related units. The two means typically represent two different times (e.g., pre-test and post- test
with an intervention between the two time points) or two different but related conditions or
units (e.g., left and right ears, twins). The purpose of the test is to determine whether there is
statistical evidence that the mean difference between paired observations on a particular
outcome is significantly different from zero. The Paired Samples t Test is a parametric test.

This test is also known as:

• Dependent t Test
• Paired t Test
• Repeated Measures t Test

The Paired Samples t Test is commonly used to test the following:

• Statistical difference between two time points


• Statistical difference between two conditions
• Statistical difference between two measurements
• Statistical difference between a matched pair
Note: The Paired Samples t Test can only compare the means for two (and only two) related
(paired) units on a continuous outcome that is normally distributed. The Paired Samples t Test is
not appropriate for analyses involving the following: 1) unpaired data; 2) comparisons between
more than two
units/groups; 3) a continuous outcome that is not normally distributed; and 4) an ordinal/ranked
outcome.

Example : Paired T-Test and CI: Yield before, Yield after

Paired T for Yield before - Yield after

N Mean StDev SE Mean

Yield before 8 92.26 2.39 0.84


Yield after 8 92.73 2.98 1.05
Difference 8 -0.48 4.48 1.58

95% CI for mean difference: (-4.22,

3.26) T-Test of mean difference = 0


Two-sample t test:
Two-sample t hypothesis testing is statistical analysis designed to test if there is a difference
between two means from two different populations. For example, a two-sample hypothesis
could be
used to test if there is a difference in the mean salary between male and female doctors in the
New York City area.
It helps to answer questions like whether the average success rate is higher after implementing
a new sales tool than before or whether the test results of patients who received a drug are
better than test results of those who received a placebo.

Here is an example starting with the absolute basics of the two-sample t-test. The question being
answered is whether there is a significant (or only random) difference in the average cycle time
to deliver a pizza from Pizza Company A vs. Pizza Company B. This is the data collected from a
sample of deliveries of Company A and Company B.

Table 1: Pizza Company A Versus Pizza Company B Sample Deliveries


A B
20.4 20.2
24.2 16.9
15.4 18.5
21.4 17.3
20.2 20.5
18.5
21.5
Since both samples have a p-value above 0.05 (or 5 percent) it can be concluded that both
samples are normally distributed. The test for normality is here performed via the Anderson
Darling test for which the null hypothesis is “Data are normally distributed” and the alternative
hypothesis is “Data are not normally distributed.”
Anderson Darling normality test rejects the hypothesis of normality when the p-value is less
than or equal to 0.05. Failing the normality test allows you to state with 95% confidence the
data does not fit the normal distribution.
Using the two-sample t-test, statistics software generates the output in Table

Difference = mu (A Sample) – mu (B
Sample) Estimate for difference: 1.54857
95% CI for difference: (-1.53393, 4.63107)
T-test of difference = 0 (vs not =): T-value = 1.12, P-value = 0.289, DF = 10
Both use pooled StDev = 2.3627

Since the p-value is 0.289, i.e. greater than 0.05 (or 5 percent), it can be concluded that there is
no difference between the means. To say that there is a difference is taking a 28.9 percent risk
of being wrong.

F Test: An Example
The F-Test used in the hypothesis testing of variances (not means) as in ANOVA. This test can be
used for any continuous distribution to compare variances. The value of F represents the ratio
of two variances, and comparing the F-test value to the F-critical value is used to make a decision
on the null hypothesis.

The F-Test used in the hypothesis testing of variances (not means) as in ANOVA.

The F-Test assumes a normally distribution, as well as Bartlett's Test. The samples should exhibit
normal conditions within each set of experiments (or trials).

Levene's Test is similar, but is used when analyzing data that is not from (or can not be assumed)
a normal distribution. This test can be used for any continuous distribution to compare
variances.
The value of F represents the ratio of two variances, and comparing the F-test value to the F-
critical value is used to make a decision on the null hypothesis.
It is used to compare:

• 1 Sample Variance to a Target


• 2 Sample Variances
• >2 Variances use ANOVA

In ANOVA, the value of F is the ratio of treatment variance to the error variance.

3.4.2 Use of the one-sample variance test

Use this tool to compare the variance of a normally-distributed sample with a given value.

Description of the one-sample variance test

Let us consider a sample of n independent normally distributed observations. One shows that
the sample variance, s² follows a scaled chi-squared distribution with n-1 degrees of freedom.
s² ~ [σ²/(n-1)] * Χ²n-1

where s² is the theoretical sample variance. This allows us to compute a confidence interval
around the variance.

To compare this variance to a reference value, a parametric test is proposed. It is based on the
following statistic:

Χ0² = (n-1) s²/σ0²


which follows a chi-square distribution with n-1 degrees of freedom.

This test is said to be parametric as its use requires the assumption that the samples are
distributed normally. Moreover, it also assumed that the observations are independent and
identically distributed. The normality of the distribution can be tested beforehand using the
normality tests.

Three types of test are possible depending on the alternative hypothesis

chosen: For the two-tailed test, the null H0 and alternative Ha hypotheses

are as follows:

• H0 : s² = s0²

• Ha : s² ≠ s0²

In the left one-tailed test, the following hypotheses are used:


• H0 : s² = s0²
• Ha : s² < s0²

In the right one-tailed test, the following hypotheses are used:

• H0 : s² = s0²

• Ha : s² > s0²

Example:A new filtering device is installed in a chemical unit. Before its installation, a random
sample yielded the following information about the percentage of impurity: y1 bar = 12.5, S12 =
101.17 and n1 =
8. After installation, a random sample yielded y2 bar = 10.2, S22 = 94.73 and n2 = 9. Can you
conclude that the two variances are equal?

Two Sample T Test: An Example


A new filtering device is installed in a chemical unit. Before its installation, a random sample
yielded the following information about the percentage of impurity: y1 bar = 12.5, S12 = 101.17
and n1 = 8. After installation, a random sample yielded y2 bar = 10.2, S22 = 94.73 and n2 = 9. Has
the filtering device reduced the percentage of impurity significantly?

Two-Sample T-Test and CI

Sample N Mean StDev SE Mean

1 8 12.5 10.1 3.6

2 9 10.2 94.7 32

Difference = mu (1) - mu

(2) Estimate for

difference: 2.3

95% CI for difference: (-69.7, 74.3)


T-Test of difference = 0 (vs not =): T-Value = 0.07 P-Value = 0.947 DF = 15
Since P>0.05, there is no significance difference in the mean impurities

3.4.3 One way ANOVA

The one-way analysis of variance (ANOVA) is used to determine whether there are any
statistically significant differences between the means of two or more independent (unrelated)
groups (although you tend to only see it used when there are a minimum of three, rather than
two groups).
When we deal with more than two populations or one population with three or more
number of levels, the tool used to compare the population means (or level means) is
Analysis of Variance (ANOVA).
The comparison procedure involves analysis of variability components (within population
variability and between populations variability) and their comparison.
Although we compare population means, the name of the procedure is Analysis of
Variance (ANOVA)

Assumptions:

1. Model errors are assumed to follow


3 5 100.40 4.56 (----
)
normal distribution with ------+---------
mean 0 and variances2. +---------+---------+--
-
2. Model errors are independent of 80.0 88.0 96.0 10
each other.
Weeks of Life
3. Variance of model errors,s2, is constant for all levels.
Brand 1 Brand 2 Brand 3

Example: 1-way ANOVA


100 76 108
Three brands of batteries are under study. It is suspected that the
lives (in weeks) of the three brands are different. Five batteries of
96 80 100
each brand are tested with the following results. Are the lives of the
three batteries are different.
92 75 96
One-way ANOVA: Life versus Brand
96 84 98
Source DF SS MS F P
92 82 100

Brand 2 1196.1
598.1 Reject Null. 38.34 0.000
Error 12 At least one 187.2 15.6
Total 14 department is 1383.3

S = 3.950 R- less stressed. Sq = 86.47% R-Sq(adj) =


84.21%

Individual 95% CIs For Mean Based on

Pooled StDev

Level N Mean StDev ------+---------+---------+--


-------+--
-
1 5 95.20 3.35 (----*----)

2 5 79.40
3.85 (----
*----)
Since
P<0.05
reject
Null
Two Sample T Test - Demonstration in
Hypothesis,
Minitab
i.e.
At least one
brand has
got different
mean life

Two catalysts are being analyzed to determine how they affect the mean yield of a chemical
process. Specifically, catalyst 1 is currently in use, but catalyst 2 is acceptable. Since catalyst 2 is
cheaper, it should be adopted providing it does not change the process yield. A test is run in the
pilot plant and results in the data shown in the following Table. Is there any difference between
the mean yields? Use
= 0.05, and assume equal variances.
S.No Catalyst 1 Catalyst 2

1 91.5 89.19

2 94.18 90.95

3 92.18 90.46

4 95.39 93.21

5 91.79 97.19

6 89.07 97.04

7 94.72 91.07

8 89.21 92.75

Analyze Phase – Overview


Data Analysis – ScopeTo understand the current level of the process, analyse the collected data
with the help of statistical tools as shown below
Use a test for equal variances to test the equality of variances between populations or factor
levels. Many statistical procedures, such as analysis of variance(ANOVA) assume that
although different samples can come from populations with different means, they have the
same variance.
4 IMPROVE PHASE

4.0 Simple Linear Regression

4.1.1 Correlation

Correlation has a significant place in lean six sigma. Correlation measures relation between two
or more variables. It investigates the relationship between two quantitative and continuous
variables. If the value of one variable increases when the value of the other also increases, they
are said to be positively correlated. If the value of one variable decreases when the value of other
variable is increasing, it is said to be negatively correlated. If one variable does not affect the
other, they are not correlated.

• Correlation coefficient is used to quantify relationship between two continuous variables


and denoted by the symbol r.

• When two sets of data are strongly linked together we say they have a High Correlation.

• Correlation is Positive when the values increase together.

• Correlation is Negative when one value decreases as the other increases

Linear correlations (correlations that follow a line).


Correlation can have a value:

• 1 is a perfect positive correlation

• 0 is no correlation (the values don't seem linked at all)


• -1 is a perfect negative correlation

• The r value shows how good the correlation is (not how steep the line is), and if it is
positive or negative.
Correlation test will discover the key inputs variable change in a process or products. For instance-
the effect of advertising or promotions on the sales of a product or service sets as a good
example. Example: Ice Cream Sales

The local ice cream shop keeps track of how much ice cream they sell versus the temperature on
that day, here are their figures for the last 12 days:

Ice Cream Sales vs


Temperature
Temperature Ice Cream Sales
°C

14.2° $215
16.4° $325

11.9° $185

15.2° $332
18.5° $406

22.1° $522

19.4° $412
25.1° $614
23.4° $544

18.1° $421
22.6° $445
17.2° $408
let us call the two sets of data "x" and "y" (in our case Temperature is x and Ice Cream Sales is y):

• Step 1: Find the mean of x, and the mean of y


• Step 2: Subtract the mean of x from every x value (call them "a"), do the same for y
(callthem "b")

• Step 3: Calculate: ab, a2 and b2 for every value


2 2
• Step 4: Sum up ab, sum up a and sum upb

• Step 5: Divide the sum of ab by the square root of [(sum of a2) × (sum ofb2)]

Here is how I calculated the first Ice Cream example (values rounded to 1 or 0 decimal places):
As a formula it is:

Where:

• Σ is Sigma, the symbol for "sum up"


• is each x-value minus the mean of x (called "a" above)
• is each y-value minus the mean of y (called "b" above)

We can easily see that warmer weather and higher sales go together. The relationship is good
but not perfect.

Correlation does not always imply Causation. Correlation of X and Y variable does not mean
that the variation in Y is caused by (or) due to a variation in X. It is just that they happen
simultaneously. It is with the business knowledge, that the user should decide whether the
relationship is causation or not.
When r = 1 or r = -1, all points fall on a straight line; when r = 0, they are scattered and give no
evidence of a linear relationship. Any other value of r suggests the degree to which the points
tend to be linearly related.
Note: Regression and correlation involve testing a relationship rather than testing of means or
variances. They are used to find out the variables and to the degree the impact the response so
that the team can control the key inputs. Controlling these key inputs is done to shift the mean
and reduce variation of an overall Project "Y".

4.1 Simple Linear Regression

Simple linear regression is a statistical method that allows us to summarize and study
relationships between two continuous (quantitative) variables:

• One variable, denoted x, is regarded as the predictor, explanatory,or independent


variable.
• The other variable, denoted y, is regarded as the response, outcome, ordependent
variable.

Regression analysis is a set of statistical processes for estimating the relationships among
variables. It includes many techniques for modeling and analyzing several variables, when the
focus is on the relationship between a dependent variable and one or more independent
variables (or 'predictors'). More specifically, regression analysis helps one understand how the
typical value of the dependent variable (or 'criterion variable') changes when any one of the
independent variables is varied, while the other independent variables are held fixed.

While Correlation Analysis assumes no causal relationship between variables, Regression


Analysis assumes that one variable is dependent upon: A) another single independent variable
(Simple Regression) , or B) multiple independent variables (Multiple Regression).

Regression models involve the following parameters and variables:

• The unknown parameters, denoted as β , which may represent a scalar or avector.


• The independent variables,
• The dependent variable,

Regression equation is used to find out what relationship, if any, exists between sets of data. For
example, if you measure a child’s height every year you might find that they grow about 3 inches
a year. That trend (growing three inches a year) can be modeled with a regression equation. In
fact, most things in the real world (from gas prices to hurricanes) can be modeled with some kind
of equation; it allows us to predict future events.
Only a single predictor variable or independent variable ‘X’ (e.g.: cutting speed) and a response
variable or dependent variable ‘Y’ (e.g: tool life).
The simple linear regression model is
Yi = β0 + β1 xi + εi i=1,2,...,n
Where β0 is the intercept and β1 is the slope of the line and εij is the random error.

What does a Regression analysis do?


The entire concept of regression analysis is very simple. The foundation lies in the ‘Straight Line
Equation’.

Where:

▪ y: Output variable
▪ x: Input or Predictor Variable
▪ b0: Intercept or constant
▪ b1: Slope
▪ e: Error or Residual value
Like in Correlation, the ‘x’ and ‘y’ values are plotted in a Scatter plot. A line that indicates the
slope of the data points is drawn.

Regression Plot

The above graph demonstrates how a Regression Plot looks like. Any model will not be 100%
accurate in predicting the y value. Thus there will be an error component or residual value
involved in the equation. However, a good model is one that tries to minimize the error/residual
values, and has a line (Regression line) that tries to be close to a maximum number of data points.

Terminologies in Regression Analysis


Before understanding how to perform a regression analysis, let us understand the key
terminologies involved.

1. Coefficients: These are the intercept (b0) and slopes (b1, b2,….,bn) of individual input
variables. The values of these coefficients are extremely important in determining the
Regression Equation. They are also, one of the key outputs of a RegressionAnalysis.

2. Standard Error (SE): It denotes the standard deviation of the Residual values from the
Regression line. A good model should try to reduce the Standard Error.

3. Coefficient of Determination (r2): This value denotes how good and efficient the regression
equation is, in predicting the y value. Typically, r2 values range from 0 to 1; 0 denoting a worst
model and 1 denoting the best model. Higher is the r2 value, better is the prediction. Often,
for Multiple Regression, another value known as r2adj is calculated. This value is different
from r2 by eliminating the effect of multiple interrelated variables
(Multicollinearity).
4. Multicollinearity: It is a condition denoting the input variables being strongly correlated. In
such a condition, this correlation may mask another input variable’s significance in the
prediction. So, to avoid this, before proceeding to Regression, a Correlation Matrix for all the
input variables is plotted and the masking effect is identified. Any one of the pair of variables
with strong correlation is removed based on the business knowledge of the problem.

How to Perform a Regression Analysis?

1. Identify the relevant input variables and collect data on all the input variables and output
variable.
2. Select the suitable regression model (Simple Regression, Multiple Regression etc)
3. Calculate r2 value. Now a days a lot of software packages, add-ins are available to perform
the entire calculation and analysis.
4. Assess the p value for each of the input variables (p-value less than the confidence level
considered indicates that the variable is a useful predictor). Remove any variables with p-
value > confidence level.
5. Test the variables for Multicollinearity and remove duplicate variables.
6. Develop the predictor equation with significant and shortlisted variables only.
Thus the new equation will be a good predictor for a given problem, provided a correct model,
accurate data and a good calculation methodology is used.

Simple Linear Regression – Example


A study was performed on wear of a bearing Y and its relationship to X1 = oil viscosity. The
following data were obtained. Fit a simple regression model to the data (Yvs. X1) = 0.05
Regression Analysis: Y versus X1

The regression equation is

Y = 228 - 2.86 X1
Predictor Coef SE Coef T

P Constant 227.83 26.01


About 66% Wear can be predicted by oil
8.76 0.001 viscosity

X11 -2.8560 0.8780 -3.25 0.031

S = 31.1112 R-Sq = 72.6% R-Sq(adj) = 65.7%


Analysis of Variance

Source DF SS MS F P There is a regression between Wear and oil


viscosity
Regression 1 10240 10240 10.58 0.031
Residual Error 4 3872 968

Total 5 14112

Y X1
193 1.6

230 15.5

172 22.0

91 43.0

113 33.0

125 40.0
Coefficient of Determination (R2)
Analysis of the validity of Regression
Equation The regression equation is y =
400 - 0.231 X2
P value of X2 factor is 0.015 - Means X2 is a significant X

P value of regression equation as per ANOVA table is 0.015 - Means the equation is valid
R sq adj value is 75.9 - Means only 76% of variation in Y is because of X2. Some other factors also
contribute to the variation.

Need to look for more potential X.2 adj should be min 85%

Coefficient of Determination

2
(R ) Residual Analysis:
Residuals: The difference between the observed value of the dependent variable (y) and the
predicted value (ŷ)is called the residual (e). Each data point has one residual. It is the vertical
distance from the actual plotted point to the point on the regression line.

Residual = Observed value - Predicted value

e=y-ŷ

Both the sum and the mean of the residuals are equal to zero. That is, Σ e = 0 and e = 0.
Residual Plots

A residual plot is a graph that shows the residuals on the vertical axis and the independent
variable on the horizontal axis. If the points in a residual plot are randomly dispersed around the
horizontal axis, a linear regression model is appropriate for the data; otherwise, a non-linear
model is more appropriate.

The table below shows inputs and outputs from a simple linear regression analysis.

x y e
6 7 65.411 4.589
0 0
7 6 71.849 -6.849
0 5
8 7 78.288 -8.288
0 0
8 9 81.507 13.493
5 5
9 8 87.945 -2.945
5 5
And the chart below displays the residual (e) and independent variable (X) as a residual plot.

The residual plot shows a fairly random pattern - the first residual is positive, the next two are
negative, the fourth is positive, and the last residual is negative. This random pattern indicates
that a linear model provides a decent fit to the data.

Below, the residual plots show three typical patterns. The first plot shows a random pattern,
indicating a good fit for a linear model.

Random pattern

Non-random: U-shaped

Non-random: Inverted U

The other plot patterns are non-random (U-shaped and inverted U), suggesting a better fit for a
non- linear model.

Normal Probability Plot - Looks normal - ok


Residual vs Fits - Randomly above and below the line with no trends - ok
Histogram - shows a normal distribution - ok

Residual vs Time order - Randomly above and below the line with no trends - ok
Overall conclusion: The regression equation is valid, but X2 accounts for only 76% of variation in Y
4.2 Multi Linear Regression Analysis

Multiple regression is an extension of simple linear regression. It is used when we want to predict
the value of a variable based on the value of two or more other variables.
For example, you could use multiple regression to understand whether exam performance can
be predicted based on revision time, test anxiety, lecture attendance and gender. Alternately,
you could use multiple regression to understand whether daily cigarette consumption can be
predicted based on smoking duration, age when started smoking, smoker type, income and
gender.
Multiple regression also allows you to determine the overall fit (variance explained) of the model
and the relative contribution of each of the predictors to the total variance explained. For
example, you might want to know how much of the variation in exam performance can be
explained by revision time, test anxiety, lecture attendance and gender "as a whole", but also the
"relative contribution" of each independent variable in explaining the variance.
A Multiple Regression is one that has more than one input variable to predict a given y. With
more than one input variable, the mathematical model looks like below:

Yi = β0 + β1X1 + β11X12 + β2X2 + β22X22 + β12X1X2 + ε

This equation features five distinct kinds of terms:

β0: This term is the overall effect. It sets the starting level for all the other effects,

regardless of what the X variables are set at.

βiXi: The β1X1 and β2X2 pieces are the main effects terms in the equation. Just like in the

simple linear regression model, these terms capture the linear effect each Xi has on the

output Y. The magnitude and direction of each of these effects are captured in the

associated βi coefficients.

βiiXi2: β11X12 and β22X22 are the second-order or squared effects for each of the Xs.

Because the variable is raised to the second power, the effect is quadratic rather than

linear. The magnitude and direction of each of these second-order effects are indicated

by the associated βii coefficients.

β12X1X2: This effect is called the interaction effect. This term allows the input variables

to have an interactive or combined effect on the outcome Y. Once again, the magnitude

and direction of the interaction effect are captured in the β12 coefficient.
ε: This term accounts for all the random variation that the other terms can’t explain. ε is a

normal distribution with its center at zero.

When there are more variables, there are possibilities that there exists an interaction or correlation
among the input variables. So, the model should try to identify that duplication effect, which will
inflate the y value.

Definition of terms
Response Variables - The output variables which are tested in regression analysis - TheY’s
Predictor Variables - The input variables, which impact the Ys - The X’s
P-value - provides the probability value of significance testing. If p<0.05, then there
is a relationship between the Y and X
Multi collinearity - Statistical phenomenon in which two or more Xs are highly co-related.
Mallow’s Cp - A value that compares the precision and bias of full models to the models
of X’s resulting from best subsets

Adjusted R2 - The co-efficient of Determination, to be usRed instead of R2 when multiple


X’s are regressed against a Y

Predicted R2 - Is used to predict responses for new observations (predictedvalues)


Least squares method: The method used to calculate estimates by fitting a regression line
to the points from a data set in such a way that the square of the distance from the data
point to the line is the minimum

4.2.1 Non Linear Regression

Nonlinear regression is a form of regression analysis in which observational data are modeled by
a function which is a nonlinearcombination of the model parameters and depends on one or
more independent variables.
Many processes do not have normal data. Examples such as :

▪ Call time

▪ Waiting time

In such cases we use Mathematical functions like logx, 1/x etc to transform the data
to give normal distribution
Box Cox and Johnson transformations are normally used to find out the right
mathematical function for transforming the data to normal distribution

4.2.2 Multiple Linear Regression

Many predictor variables or independent variables ‘X1, X2, ….Xk’ (e.g.: gender, height) and a
response variable or dependent variable ‘Y’ (e.g.: weight).

Example: Multiple Linear Regression


A study was performed on wear of a bearing Y and its relationship to X1 = oil viscosity and X2 = load.
The following data were obtained. Fit a simple regression model to the data (Y vs X1) and fit a
multiple regression model for the data. = 0.05

Y X1 X2

193 1.6 851

230 15.5 816

172 22.0 1058

91 43.0 1201

113 33.0 1357


125 40.0 1115

Example: Multiple Linear Regression


The regression equation is Y = 351 - 1.27 X1 - 0.154 X2

Predictor Coef SE Coef T P

Constant 350.99 74.75 4.70 0.018

X1 -1.272 1.169 -1.09 0.356

X2 -0.15390 0.08953 -1.72 0.184

S = 25.4979 R-Sq = 86.2% R-Sq(adj) = 77.0%


Analysis of Variance

Source DF SS MS F P

Regression 2 12161.6 6080.8 9.35 0.051

Residua 3 1950.4 650.1


l Error
Total 5 14112.0

The Regression is not valid since the P value is 0.051 > 0.05
X1 and X2 are not significant X’s since P values are 0.356 and 0.184 respectively In this
case remove the X with higher P Value and re-run the Regression
Analysis of the validity of Regression Equation
The regression equation is y = 400 - 0.231 X2
P value of X2 factor is 0.015 - Means X2 is a significant X
P value of regression equation as per ANOVA table is 0.015 - Means the equation isvalid
R sq adj value is 75.9 - Means only 76% of variation in Y is because of X2. Some other
factors also contribute to the variation. Need to look for more potential X.2 adj should be
min85%
Residual Analysis:
Normal Probability Plot - Looks normal - ok
Residual vs Fits - Randomly above and below the line with no trends - ok
Histogram - shows a normal distribution - ok
Residual vs Time order - Randomly above and below the line with no trends - ok

Overall conclusion: The regression equation is valid, but X2 accounts for only 76% of
variation in Y

Multiple Linear Regression


RESIDUAL ANALYSIS

Regression Model:

Where is the observed response Y is the expected response = a + bX


ˆ
And ei, is the error or residual; assumed ~ NID (0, σ2)

Y = ˆY + ei i

Hence, there is a need to verify that the residuals are Normally & Independently Distributed about mean
= 0 and variance = σ2
4.2.4 RESIDUAL ANALYSIS Lack of Fit
This is the systematic error in estimating Y from the regression equation with a fixed value of X.
This can be caused by either:
Assuming a linear relationship when the true relationship might not be linear
Assuming a single X when there might be more than one X that influences Y

Random Error
This is the error inestimating Y caused by random variability in Y for a fixed value of X
RESIDUAL PLOTS
Histogram of Residuals
Normal Plot of Residuals Residuals
vs fitted value
Residuals vs Run Order
Collinearity:
Refers to a linear relationship between two X’s
The standard errors of coefficients are relatively high for predictors and residuals

For multi collinearity identify VIF - Variance Inflation Factor, which shows the extent to
which multi-collinearity exists among predictors
If VIF <1, X’s are not correlated, 1<VIF < 5 means X’s are moderately correlated and if
VIF>5, X’s are highly correlated.
On detection of high correlation, the Black Belt should regress one predictor to another
and keep only the original X
Multi - collinearity is an issue because it increases the variance of regression terms and
makesit unstable

Durbin Watson Statistic:


Tests for the presence of auto-correlation in residuals, i.e. is the residual value depends
on the value of the previous residue
When adjacent observations are correlated, the Ordinary Least Square 9OLS0 methodin
regression underestimates the Standard Error of the coefficients

If D<Lower bound, positive correlation exists, If D>Upper bound, no


correlation exists.If D is range bound, the test is inconclusive Press statistic:
PRESS stands for Predicted Residuals Sum of Squares

Is used as an indication of the predicting power of the model i.e. how well the model could
fit new observations
Is the sum of the squared external residuals. External residuals are calculated by
findingthe predicted value for an observation by leaving out the observation
SSE(Sum Squares of Errors) explains the quality of fit, while PRESS statistic explainsthe
predicting quality.

Best Subsets Regression:


Identifies the best fitting model that can be constructed with the specified X variables.
The goal of regression is achieved with the fewest predictors possible e.g. If we have 3
X’s, Minitab will be able to show best and second best one predictor models and then
show the full predictor model
Stepwise Regression:
In stepwise regression, the most significant variable is added or deleted from the
regression model
Three common stepwise regression procedures are Standard stepwise, forward selection
and backward elimination. Use of Mallow’s Cp statistic in stepwise regression is popular.

4.2.3 Confidence and Prediction Intervals

Point estimation involves the use of sample data to calculate a single value (known as a point
estimate or statistic) which is to serve as a "best guess" or "best estimate" of an unknown
population parameter (for example, the population mean). More formally, it is the application of
a point estimator to the data to obtain a point estimate.

When a parameter is being estimated, the estimate can be either a single number or it can be a
range of scores. When the estimate is a single number, the estimate is called a "point estimate";
when the estimate is a range of scores, the estimate is called an interval estimate. Confidence
intervals are used for interval estimates.

As an example of a point estimate, assume you wanted to estimate the mean time it takes 12-
year- olds to run 100 yards. The mean running time of a random sample of 12-year-olds would
be an estimate of the mean running time for all 12-year-olds. Thus, the sample mean, M, would
be a point estimate of the population mean, μ.
Often point estimates are used as parts of other statistical calculations. For example, a point
estimate of the standard deviation is used in the calculation of a confidence interval for μ. Point
estimates of parameters are often used in the formulas for significance testing.

Point estimates are not usually as informative as confidence intervals. Their importance lies in
the fact that many statistical formulas are based on them.
Confidence interval : A confidence interval is a range of values, derived from sample statistics,
that is likely to contain the value of an unknown population parameter. Because of their random
nature, it is unlikely that two samples from a given population will yield identical confidence
intervals. But if you repeated your sample many times, a certain percentage of the resulting
confidence intervals would contain the unknown population parameter. The percentage of these
confidence intervals that contain the parameter is the confidence level of the interval.
Confidence intervals tell you about how well you have determined the mean. Assume that the
data really are randomly sampled from a Gaussian distribution. If you do this many times, and
calculate a confidence interval of the mean from each sample, you'd expect about 95 % of those
intervals to include the true value of the population mean. The key point is that the confidence
interval tells you about the likely location of the true population parameter.

Suppose that you randomly sample light bulbs and measure the burn time. Minitab calculates
that the 95% confidence interval is 1230 – 1265
hours. The confidence interval indicates that you
can be 95% confident that the mean for the entire
population of light bulbs falls within this range.

Confidence intervals only assess sampling error in


relation to the parameter of interest. (Sampling
error is simply the error inherent when trying to
estimate the characteristic of an entire population
from a sample.) Consequently, you should be
aware of these important considerations:

• As you increase the sample size, the sampling error decreases and the intervals become
narrower. If you could increase the sample size to equal the population, there would be
no sampling error. In this case, the confidence interval would have a width of zero and be
equal to the true population parameter.

• Confidence intervals only tell you about the parameter of interest and nothing about the
distribution of individual values.
In the light bulb example, we know that the mean is likely to fall within the range, but the 95%
confidence interval does not predict that 95% of future observations will fall within the range.

SAMPLING DISTRIBUTION
A statistic is defined any function of the sample data that does not contain unknown
parameters.
The probability distribution of a statistic is called a sampling distribution.
– For example, the probability distribution of Xbar is called the sampling distribution of the mean.

CONFIDENCE INTERVAL ON THE .MEAN WITH KNOWN VARIANCE

I f X-bar is the sam pl e mean of a rando m sample of size n from a p


opulat io n with know n var iance cr2, t hen a 100(1-a ) percent rn
nfidence interv al onµ is given by
X - Z1 -a 12 Cf / n < µ < X +Z1- a120' I n

where z1 -«/2 is t ihe upper a/2 percent age point of the st andar d
norm aI distr1i b U1t i on.

CONFIDENCE INTERVAL ON THE MEAN WITH UNKNOWN


VARIANCE
If X-bar Ernd s are t he sample m ean a nd st andar d deviat io n of a random
sam ple of size n fro m a 11o rmaI populat ion, t hen a 100(1-o:} percent
oonfidence i11terval on µ is given by

where t1-a/ ,i n- i is t he upper a/ 2 percent age point of the t dist ribut ion
with n-
1 degrees o f fr eedom.
Prediction intervals tell you where you can expect to see the next data point sampled. Assume
that the data really are randomly sampled from a Gaussian distribution. Collect a sample of data
and calculate a prediction interval. Then sample one more value from the population. If you do
this many times, you'd expect that next value to lie within that prediction interval in 95% of the
samples. The key point is that the prediction interval tells you about the distribution of values,
not the uncertainty in determining the population mean.
Prediction intervals must account for both the uncertainty in knowing the value of the population
mean, plus data scatter. So a prediction interval is always wider than a confidence interval.
The difference between a prediction interval and a confidence interval is the standard error. The
standard error for a confidence interval on the mean takes into account the uncertainty due to
sampling. The key point is that the confidence interval tells you about the likely location of the
true population parameter.

4.2.5 Box Cox Transformation


Data transformation is the process of converting data from one format or structure into another
format or structure.

Normally distributed data is needed to use a number of statistical analysis tools, such as
individuals control charts, Cp/Cpk analysis, t-tests and analysis of variance (ANOVA).

When data is not normally distributed, Data transformation, and particularly the Box-Cox power
transformation help to make data normal.
Transforming data means performing the same mathematical operation on each piece of original data.

Some transformation examples from daily life are currency exchange rates (e.g., U.S. dollar into
Euros) and converting degree Celsius into degree Fahrenheit. These two transformations are
called linear transformations because the original data is simply multiplied or divided by a
coefficient or a constant is subtracted or added.

Box Cox Transformation


But these linear transformations do not change the shape of the data distribution and, therefore,
do not help to make data look more normal
George Box and David Cox developed a procedure to identify an appropriate exponent (Lambda
= l) to use to transform data into a “normal shape.”
The Lambda value indicates the power to which all data should be raised. In order to do this, the
Box- Cox power transformation searches from Lambda = -5 to Lamba = +5 until the best value is
found.
The lower and upper confidence levels (CLs) show that the best results for normality were
reached with Lambda values between -2.48 and -0.69. Although the best value is -1.54, the
process works better if this value is rounded to a whole number; this will make it easier to
transform the data back and forth. The best whole-number values here are -1 and -2 (the inverse
2
function of Y and Y , respectively). The histogram on the right shows the transformed data using
Lambda = -1, now more normally distributed.
Box Cox Transformation
Box-Cox power transformation is not a guarantee for normality.
This is because it actually does not really check for normality; the method checks for the
smallest standard deviation.
The assumption is that among all transformations with Lambda values between -5 and
+5, transformed data has the highest likelihood – but not a guarantee – to be normally
distributed when standard deviation is the smallest.
Therefore, it is absolutely necessary to always check the transformed data for normality
using a probability plot.
Additionally, the Box-Cox Power transformation only works if all the data is positive and
greater than 0.
This, however, can usually be achieved easily by adding a constant (c) to all data such that
it all becomes positive before it is transformed. The transformation equation is then:
l
Y’ = (Y+C) .

4.3 DESIGNED EXPERIMENTS


Design of experiments (DOE) is a systematic method to determine the relationship between factors affecting a process
and the output of that process. In other words, it is used to find cause-and-effect relationships. This information is
needed to manage process inputs in order to optimize the output.
The most commonly used terms in the DOE methodology include: controllable and uncontrollable input factors,
responses, hypothesis testing, blocking, replication and interaction.
Controllable input factors, or x factors, are those input parameters that can be modified in an experiment or process.
For example, in cooking rice, these factors include the quantity and quality of the rice and the quantity of water used for
boiling.
Uncontrollable input factors are those parameters that cannot be changed. In the rice-cooking example, this may be the
temperature in the kitchen. These factors need to be recognized to understand how they may affect the response.
Responses, or output measures, are the elements of the process outcome that gage the desired effect. In the cooking
example, the taste and texture of the rice are the responses.
TRADITONAL” APPROACHES TO EXPERIMENTATION
Nowadays, three traditional approaches to experimentation are still in use:
• Analysis of Unstructured Data
• The Best Educated Guess Method
• One Factor At a Time (OFAT) Experimentation
Process & Noise factors

Noise factors
Humidity
Ambient Temperature
What is Design of Experiments
• Purposeful changes of the inputs (factors) to a process in order to observe the corresponding changes in the outputs
(responses).
• Statistical DOE encompasses the planning, design, data collection, analysis, and interpretation strategy and tactics to enable
us to effectively and efficiently build a statistical model between experimental responses and controllable factor inputs.
Design of experiments is used because
• It is the most effective methods for identifying key input factors
• It is the most efficient way to understand the relationship between input factors and responses
• It allows us to build a mathematical model to characterize our process
• It can determine the optimized process settings for optimum process performance
• It is a scientific method for setting tolerances
• It helps in achieving robust performance against noise factors What is Design of Experiments

MODELS
• In our efforts to improve processes, we seek to determine cause/effect relationships, that we call Models
• Models are used to illustrate the relationship between the controllable input factors and each measurable output response,
in the presence of systematic biasing and uncontrolled variation due to both known and unknown uncontrollable factors.

EXAMPLE MODEL

yj = f(x1, x2, …, xk) + Error


The Error term is a function of the uncontrolled factors (Noise).
Statistical Model:
Mathematical relationship – relate change in factors to response:
Y=β0 + β1x1 + β2x2 + β12x1x2 + Error Signal
– systematic components
Noise – random component – accounts for uncontrolled variation
The β’s in the above equation are called coefficients, which will be estimated from experimental runs. DOE
- Terminology
Factors (Controllable): Inputs – The “knobs” you adjust (independent variables).
Can be Continuous or Categorical Examples:
 Temperature
 Pressure
 Vendor
 Location
Factors (Uncontrollable): Cannot be controlled but may affect the responses. Continuous or Categorical Examples:
 Relative humidity
 Machines
Responses: Outputs of a process – (dependent variables) Continuous
or Categorical:
Examples:
 Bond Strength
 Percent Defective
 Thickness
 Cycle Times
 Level – Specific setting for a controllable factor. The settings of a factor in an experimental run that is used
to generate experimental response values.
2 Levels 3 Levels

Pressure Temperature
60 psi 40
100 psi 50
60 deg C

• Run, Experimental Run, Trial, or Treatment Combination:


• Equivalent terms for a specific combination of the level settings for the controllable factors in an experiment. Example:
Factor Levels

RUN# TEMPERATURE PRESSURE

1 275 60

2 350 50

3 400 40

Formula : number of Runs or treatments = (Level ) Factors EXPERIMENTAL UNIT (E.U)


• The experimental unit for an experiment is the experimental material to which a particular experimental run or factor-level
combination is applied.
• Examples of Experimental Units may include different people, laboratory animals, circuit boards, semiconductor waters, etc.
• For replicated runs, different experimental units must be assigned to replicates of the same factor-level combination.
• The assignment of experimental runs to different experimental units should be done randomly to avoid any systematic biases
of unknown or uncontrolled factors.

• Main Effect: An estimate of the effect of a factor independent of any other factors

An interaction effect is the simultaneous effect of two or more Factors on a Response in which their joint effect is significantly greater
than the effect of the individual factors

Factor A: Female Rabbit

Factor B: Male Rabbit


Not in Cage In Cage

Not in Cage 0 0

In Cage 0 Baby rabbits


Cube Plot for Designs
• Cube plots display the response values at all combinations of process /design parameter settings.
• We can determine the best and worst combinations of factor levels for achieving the desired optimum response.
• A Cube plot is useful to determine the path of steepest ascent or descent.
• The figure illustrates an example of a cube plot for three factors : A,B and C
• The graph indicates that maximum response is achieved when A and C are in High level and B is in Low Level.
• The minimum response is achieved when A is low and B,C are in High level.
Main Effect Plots for factors A,B,C
Confounding: (Aliased)
• When the estimate of an effect also includes the influence of one or more other effects (usually high order interactions) the
effects are said to be confounded (aliased).
• For example, if the estimate of effect C in a three factor experiment actually estimates (C + ABC), then the main effect C is
aliased with the 3-way interaction ABC.
• Note: This causes no inaccuracy in results when the higher order interaction is either non-existent or insignificant.
RADOMIZATION OF STANDARD ORDER DESIGNS
The Standard Order should never be the order in which you run the experiment.
The order of the runs should always be randomized to reduce the possibility of systematic biases associated with the standard
order of runs.
Randomize
EXPERIMENTAL ERROR
• The model for the DOE Experiment is • Y = β0 + β1x1 + β2x2 + β3x3 + β12x1x2 + β13x1x3 + β23x2x3 + β123x1x2x3 + Error
• It is the “noise” in the experiment caused by uncontrolled and unknown nuisance factors. We will also call this “Pure Error”.
• It is the variation that occurs in a response when the same factor-level combination is independently replicated.
• It measures the failure of two “identically” treated experimental units to yield the same results.

ERROR is the residuals from the experiment due to


uncontrolled and unknown nuisance
factors.
DOE - Principles
• Randomization is an experimental design technique for allocating experimental material and/or for assigning the scheduling
sequence of experimental runs to factor-level combinations in an experimental study.
• The purpose of randomization is to eliminate or reduce the possibility of systematic effects on the responses from
uncontrollable and/or unknown nuisance factors.
• Randomization is achieved through random number tables or through computer random number generators.
• Randomization ensures that the conditions of one run neither depend on the conditions of the previous run nor predict the
conditions in the subsequent run.
• Randomization ensures that the runs are statistically independent.
• Randomization does not reduce the “noise” in the experiment, but attempts to reduce the influence of possible systematic
biases of uncontrolled nuisance factors on the response. REPLICATION
• A Replicate is an experimental run that is conducted independently of one or more other runs in the experiment that have
the same factor-level combination of the input factors.
• A Replicate is not a repeat of the measurement at the same factor levels of the current run. The replicate experimental run
must include the complete experimental setup for the run, including the use of use of a separate experimental unit.
• Independent Replicates are ensured through the use of randomization of all the runs, including all replicate runs.
We need to add replication to a designed experiment for two reasons:
 To get an estimate of the underlying variation of the process, and
 To increase the precision of the estimates of the main effects, and interaction effects. Centre Points
• Center Points are replicate runs in a design where all of the continuous factors are at their “center” level, midway between
the low and high values. A center point run has the coded levels of all continuous factors set at 0.
• Why do we want to include center points in a design?
• Estimate of the true experimental error.
• Method to determine if a drift or instability occurred during the experiment.
• Evaluation of curvature in the response.
Example: In the below example, the Low level and High level Factors are 10 and 30 and experiments are done at a factor
setting of 20 (center point) in order to detect the curvature in the effect on Y
BLOCKING
• Blocking is an experimental design technique for isolating a recognized systematic effect, and thus preventing it from masking
the effects of important input factors.
• A Block is a controllable factor or variable in the process that is nuisance factor. It is not one of the factors of interest in the
study, but cannot or should not be fixed at one level. BLOCKING AND RANDOMIZATION
• “Block what you can, and randomize what you cannot.”
• Blocking and Randomization are valuable devices for dealing with unavoidable sources of variability.
• BLOCK controllable factors of no interest. RANDOMIZE uncontrollable factors

ORTHOGONAL CODING
• The controllable input factors in an experiment may be of two types, continuous or categorical.
• The specific levels of a factor will vary depending upon the type of factor, the range of the factor (e.g., 100 to 200) and the
units of measurement (e.g., degrees centigrade).
• We will code the two levels of each factor as –1 to +1. This is called Orthogonal Coding or orthogonal scaling. BENEFITS OF
ORTHOGONAL CODING
• By coding the levels of the factors as –1 and +1, we give all of the factors equal “weight” in the model.
 No one factor will dominate simply because it has a range from 600 to 1000 as compared to a range 0.1 to 0.6.
 It is scaling all the factors to be the same “size”. Each factor becomes unit less and factor effects are directly
comparable.
 The mean (intercept) in the model will have a literal meaning – it is the mean of the response and is at the center of
the design “space”.
Balanced & Orthogonal Designs Consider a 24 full factorial with four runs

A B C D

1 -1 -1 -1

-1 -1 -1 -1
1 1 -1 -1

-1 1 -1 -1

1 -1 1 -1

-1 -1 1 -1

1 1 1 -1

-1 1 1 -1

1 -1 -1 1

-1 -1 -1 1

1 1 -1 1

-1 1 -1 1

1 -1 1 1

-1 -1 1 1

1 1 1 1

-1 1 1 1
All the runs for 24 full factorial Design

A*B A*C A*D BC BD CD


-
-1 -1 1 1 1 1

1 1 1 1 1 1
-
1 -1 1 -1 -1 1

-1 1 1 -1 -1 1
-
-1 1 1 -1 1 -1

1 -1 1 -1 1 -1
-
1 1 1 1 -1 -1
-1 -1 1 1 -1 -1

-1 -1 1 1 -1 -1
-
1 1 1 1 -1 -1

1 -1 1 -1 1 -1
-
-1 1 1 -1 1 -1

-1 1 1 -1 -1 1
-
1 -1 1 -1 -1 1

1 1 1 1 1 1
-
-1 -1 1 1 1 1

Total 0 0 0 0 0 0
Checking for orthogonal and Balance Design. The sum should be 0 • Use Minitab to create the design of experiment •
State > DOE > Factorial > Create Factorial Design
StdOrder RunOrder CenterPt Blocks A B C

14 1 1 2 1 -1 1

10 2 1 2 1 -1 -1

12 3 1 2 1 1 -1

11 4 1 2 -1 1 -1

9 5 1 2 -1 -1 -1

13 6 1 2 -1 -1 1

15 7 1 2 -1 1 1

16 8 1 2 1 1 1
2 9 1 1 1 -1 -1

5 10 1 1 -1 -1 1

6 11 1 1 1 -1 1

4 12 1 1 1 1 -1

1 13 1 1 -1 -1 -1

8 14 1 1 1 1 1

7 15 1 1 -1 1 1

3 16 1 1 -1 1 -1

Overview DOE - Methods


There are four basic objectives for experiments:
• Screening
• Optimization
• Comparison
• Robust design Screening designs
• Used early in product and process development to determine only the important variable that affect the response and their
interactions.
• We are separating the “vital few from trivial many”
Optimization designs
Used when we are trying to optimize the performance of some process of product.
• The goals can take several forms:
• We can maximize the yield
• We can minimize cost
• We can hit a target thickness
• We can have multiple goals and can achieve these subject to constraints

Comparison designs
• Used when you are comparing two or more things: vendors, processes, tools, designs, etc.
• The one-to-standard, one-to-one and multiple comparisons you study in comparative methods are single
factor experiments Robust designs
• Used when we are trying to make the performance of the product the or process insensitive to the
environment in which the product or process is used or run
Design of Experiment – Methods
STATISTICAL DESIGN OF EXPERIMENTS Four

Types of Experiments Screening Optimization

Full Factorial Med Med

M Fractional Factorial Experiments High Low

M Response Surface Methodologies Low High

Plackett-Burman High Low L

types of Statistical Experimental Designs:


• Full Factorial Experiments
• Fractional Factorial Experiments
• Plackett-Burman Designs
• Response Surface Methodologies
EXPERIMENTAL OBJECTIVE BY EXPERIMENTAL TYPE Objectives for Experiments
C
fa

H
Screening Experiments
• Objective is to determine the most important factors from a longer list of factors
• Most common first experiment
• Used when the number of factors is greater than 5
• Importance is measured by the size of the effects
• Example: Fractional Factorial experiments
• Plackett-Burman Experiments are a special case of Screening Experiments Full Factorial Experiments
• Used with small number of factors and levels (e.g., levels <=3 and factors <=5).
• Mixed up or Confused
• Used following screening experiments to resolve confounding of main effects and interactions from
screening experiment.
Response Surface Experiments
• Purpose is to obtain empirical statistical model
• Follow either full or factorial experiments
• Response Surface Designs use Response Surface Methodology (RSM)
• Model is used to:
 Find optimal levels of input factors
 Predict responses at all levels of factors (within the experimental region). CONFIRM
MODEL PREDICTIONS – CONFIMATIONS RUNS

• Confirmation Runs must be conducted to verify the statistical model that has been constructed through the factorial and
RSM experiments.
• The involves the construction of confidence intervals around predicted responses that specified levels of the controlled
input factors.
Factorial Designs in Minitab
Example: Create a Full Factorial design of experiment in Minitab for 4 Factors A,B,C and D for one Block with 3 center points
Steps:
• Open Minitab
• Go to tab Stat> DOE > Factorial > Create Factorial Design
• Select the Type of Generator as 2 level factorial (Default generators)
• Select the No of Factors as 4
• Click on Designs.
• In the Box that opens up select Full Factorial, No of center points per Block as 3, No of replicates per center point as 1 and
No of Blocks as 1 and click ok.
• Click ok in the main Box and you get the output as shown
StdOrde RunOrde CenterP Block
A B C D
r r t s
19 1 0 1 0 0 0 0

-
12 2 1 1 1 1 1
1
- -
6 3 1 1 1 1 1
1
- 1 -
7 4 1 1 1
1 1
- - -
2 5 1 1 1 1 1
1
- - -
9 6 1 1 1
1 1 1
17 7 0 1 0 0 0 0

- -
10 8 1 1 1 1 1
1
18 9 0 1 0 0 0 0

- - - -
1 10 1 1
1 1 1 1
- - 1 -
5 11 1 1
1 1 1
-
8 12 1 1 1 1 1
1
-
14 13 1 1 1 1 1
1
- - 1
13 14 1 1 1
1 1
- - -
3 15 1 1 1 1 1
1
- 1
15 16 1 1 1 1
1
- -
4 17 1 1 1 1 1
1
- -
11 18 1 1 1 1 1
1
16 19 1 1 1 1 1 1
Response surface design

Similar to that of Factorial design


4.4 FULL FACTORIAL EXPERIMENTS
Full factorial experiment is an experiment whose design consists of two or more factors, each with discrete values or "levels",
and whose experimental units take on all possible combinations of these levels across all such factors.

4.4.1 2k Full Factorial Designs

The 2k designs are a major set of building blocks for many experimental designs. These designs are usually referred to as screening
designs. The 2k refers to designs with k factors where each factor has just two levels. These designs are created to explore a large
number of factors, with each factor having the minimal number of levels, just two. By screening we are referring to the process of
screening a large number of factors that might be important in your experiment, with the goal of selecting those important for the
response that you're measuring. We will see that k can get quite large. When using screening designs k can be as large as 8, 10 or
12.

In these designs we will refer to the levels as high and low, +1 and -1, to denote the high and the low level of each factor. In most
cases the levels are quantitative, although they don't have to be. Sometimes they are qFactor Combinations

Graphical representation of the factor level settings We want to try various combinations of these settings so as to establish the
best way to run the polisher. There are eight different ways of combining high and low settings of Speed, Feed, and Depth. These
eight are shown at the corners of the following diagram.

FIGURE 3.2 A 23 Two-level, Full Factorial Design; Factors X1, X2, X3. (The arrows show the direction of increase of the factors.) 3d

cube showing the factor level settings

23 implies 8 runs Note that if we have k factors, each run at two levels, there will be 2k different combinations of the levels.
In the present case, k = 3 and 23 = 8.ualitative, such as gender, or two types of variety, brand or process. In these cases the +1 and 1
are simply used as labels.

Full Factorial Design - Example

StdOrder RunOrder CenterPt Blocks A B C Y

3 1 1 1 -1 1 -1 1.23
A R&D
7 2 1 1 -1 1 1 1.34
Engineer in
a
2 3 1 1 1 -1 -1 2.34 Pharmaceuti
cal company
1 4 1 1 -1 -1 -1 1.46 is doing an
research on
4 5 1 1 1 1 -1 1.9 developing a
new drug for
fighting
6 6 1 1 1 -1 1 2.45
cancer cells.
He is doing
8 7 1 1 1 1 1 2.13 some
experiments
5 8 1 1 -1 -1 1 1.48 to study the
impact of
chemicals A,B and C on the output factor Y. Create a Full Factorial Design with one replicate, one block and analyse the
results. Table updated with results of all experiments

StdOrder RunOrder CenterPt Blocks A B C

3 1 1 1 -1 1 -1

7 2 1 1 -1 1 1

2 3 1 1 1 -1 -1

1 4 1 1 -1 -1 -1

4 5 1 1 1 1 -1

6 6 1 1 1 -1 1

8 7 1 1 1 1 1

5 8 1 1 -1 -1 1

ANOVA Table from Minitab


Estimated Effects and Coefficients for Y (coded units)
Term Effect Coef SE Coef T P
Constant 1.7913 0.003750 477.67 0.001
A 0.8275 0.4137 0.003750 110.33 0.006
B -0.2825 -0.1412 0.003750 -37.67 0.017
C 0.1175 0.0588 0.003750 15.67 0.081
A*B -0.0975 -0.0488 0.003750 -13.00 0.089
A*C 0.0525 0.0263 0.003750 7.00 0.090
B*C 0.0525 0.0262 0.003750 7.00 0.090
S = 0.0106066 PRESS = 0.0072
R-Sq = 99.99% R-Sq(pred) = 99.55% R-Sq(adj) = 99.95%
A,B are significant as P < 0.05

Time, temperature and their interaction are significant


Residual Plot for Y with all terms

The residual is not normal

Remove
insignificant terms C , AB,BC,AC, and ABC from the input factors
Pareto Chart and Normal Plot with only significant terms

Residual Plots for


Y for significant terms

The Residual is normal now ANOVA Table


Estimated Effects and Coefficients for Y (coded units)
Term Effect Coef SE Coef T P
Constant 1.7913 0.03800 47.14 0.000 A
0.8275 0.4137 0.03800 10.89 0.000
B -0.2825 -0.1412 0.03800 -3.72 0.014
S = 0.107483 PRESS = 0.147872
R-Sq = 96.36% R-Sq(pred) = 90.68% R-Sq(adj) = 94.90%
Only significant terms are included in the model and R-sq (adj) is 94.9 % Main Effect and Interaction plots

Correlation of Y with A is +ve and B is -ve


Lines are not exactly parallel. There is some interaction between A & B

Optimum setting
of A and B for a target value of Y = 2.2

4.5 FRACTIONAL FACTORIAL EXPERIMENTS Fractional factorial design


Fractional factorial designs are experimental designs consisting of a carefully chosen subset (fraction) of the experimental runs of a
full factorial design.
The subset is chosen so as to exploit the sparsity of effects principle to expose information about the most important features of the
problem studied, while using a fraction of the effort of a full factorial design in terms of experimental runs and resources. Fractional
factorial experiments are used mainly as Screening Experiments to explore the effect of large number of factors with minimum cost
and time Why Fractional factorial design
SCREENING EXPERIMENTS
Motivation for Screening Experiments

Number of Factors Number of Factor Combinations

K 2k

2 22 = 4

3 23 = 8

4 24 = 16

5 25 = 32

6 26 = 64

7 27=128

8 28 = 256
Why Fractional factorial design

Select 8 out of 16 –
Balanced option
• A Full Factorial Design for 4 factors require 16 experiments to be done.
• How to select only 8 runs if you want to use a Half Fractional Factorial Design
• First create the Full Factorial Design containing the treatment combination of all 16 tuns
• Select the runs with Effect ABCD = +1 and run only these tests without major compromise on the accuracy of the results as
the 4 way interaction is more likely to be insignificant
M A

R E A A A B B

U A A A A B B C B B C C C

N N A B C D B C D C D D C D D D D

1 1 -1 -1 -1 -1 1 1 1 1 1 1 -1 -1 -1 -1 1

2 1 1 -1 -1 -1 -1 -1 -1 1 1 1 1 1 1 -1 -1

3 1 -1 1 -1 -1 -1 1 1 -1 -1 1 1 1 -1 1 -1
4 1 1 1 -1 -1 1 -1 -1 -1 -1 1 -1 -1 1 1 1

5 1 -1 -1 1 -1 1 1 1 -1 1 -1 -1 -1 1 1 -1

6 1 1 -1 1 -1 -1 -1 -1 -1 1 -1 1 1 -1 1 1

7 1 -1 1 1 -1 -1 1 1 1 -1 -1 1 1 1 -1 1

8 1 1 1 1 -1 1 -1 -1 1 -1 -1 -1 -1 -1 -1 -1

9 1 -1 -1 -1 1 1 1 -1 1 -1 -1 1 1 1 1 -1

10 1 1 -1 -1 1 -1 -1 1 1 -1 -1 -1 -1 -1 1 1

11 1 -1 1 -1 1 -1 1 -1 -1 1 -1 -1 -1 1 -1 1

12 1 1 1 -1 1 1 -1 1 -1 1 -1 1 1 -1 -1 -1

13 1 -1 -1 1 1 1 1 -1 -1 -1 1 1 1 -1 -1 1

14 1 1 -1 1 1 -1 -1 1 -1 -1 1 -1 -1 1 -1 -1

15 1 -1 1 1 1 -1 1 -1 1 1 1 -1 -1 -1 1 -1

16 1 1 1 1 1 1 -1 1 1 1 1 1 1 1 1 1

M A

R E A A A B B

U A A A A B B C B B C C C

N N A B C D B C D C D D C D D D D

1 1 -1 -1 -1 -1 1 1 1 1 1 1 -1 -1 -1 -1 1

2 1 1 -1 -1 -1 -1 -1 -1 1 1 1 1 1 1 -1 -1

3 1 -1 1 -1 -1 -1 1 1 -1 -1 1 1 1 -1 1 -1

4 1 1 1 -1 -1 1 -1 -1 -1 -1 1 -1 -1 1 1 1

5 1 -1 -1 1 -1 1 1 1 -1 1 -1 -1 -1 1 1 -1

6 1 1 -1 1 -1 -1 -1 -1 -1 1 -1 1 1 -1 1 1

7 1 -1 1 1 -1 -1 1 1 1 -1 -1 1 1 1 -1 1
8 1 1 1 1 -1 1 -1 -1 1 -1 -1 -1 -1 -1 -1 -1

9 1 -1 -1 -1 1 1 1 -1 1 -1 -1 1 1 1 1 -1

10 1 1 -1 -1 1 -1 -1 1 1 -1 -1 -1 -1 -1 1 1

11 1 -1 1 -1 1 -1 1 -1 -1 1 -1 -1 -1 1 -1 1

12 1 1 1 -1 1 1 -1 1 -1 1 -1 1 1 -1 -1 -1

13 1 -1 -1 1 1 1 1 -1 -1 -1 1 1 1 -1 -1 1

14 1 1 -1 1 1 -1 -1 1 -1 -1 1 -1 -1 1 -1 -1

15 1 -1 1 1 1 -1 1 -1 1 1 1 -1 -1 -1 1 -1

16 1 1 1 1 1 1 -1 1 1 1 1 1 1 1 1 1

Fractional factorial Notation


FRCTIONAL FACTORIAL DESIGNS
Notation for describing fractional factorial designs:

2kp−p
• k is the number of factors in the design.
• p is the number of higher order terms used for adding factors.
• r is the resolution of the design.

Resolution
Resolution III: (1+2) – Here the Main Factors are aliased with 2 level interactions. Since some of the 2 level interactions may
have significant impact on the response, this resolution is not quite accurate. This design is quite cost effective as we have
lesser number of runs
Resolution IV: (1+3 or 2+2) -- Here the main factors are aliased with 3 level interactions or 2 level interactions are aliased
with 2 level interactions
Resolution V: (1+4 or 2+3) -- Here the Main Effect is aliased with 4 level interaction or 2 level interactions are aliased with 2
level interactions. These experiments are more accurate when compared to Resolution III or Resolution IV Experiments.
Factorial design - in Minitab



• •
• A class of designs that allows us to create experiments with some number between these fractional factorial designs are the
Plackett-Burman designs.
• Plackett-Burman designs exist for N = 12, 20, 24, 28, 36, 40, 44, 48, ..... any number which is divisible by four.
• These designs are similar to Resolution III designs, meaning you can estimate main effects clear of other main effects.
• Main effects are clear of each other but they are confounded with other higher interactions. PLACKETT –BURMAN DESIGNS
• Consider a class of designs that are useful alternatives to the 2k-p designs for screening experiments.
• When
there is a
large number
of factors to
screen ( > 8)
and time and
resources are
limited, the
two-level
fractional
factorial
Red colour indicates high risk design in terms of design precision. experiments
Yellow à moderate risk design of the from
Green à low / no risk design 2k-p may not
Plackett - Burman Design be practical.
Fractional Factorial Designs or 2k-p designs, give us designs that have 8, 16, 32, 64, 128, etc. number of runs.
There is a big gap between 16 and 32, 32 to 64, etc. • k is the
We need other alternative designs besides these with a different number of observations. number of
factors in the
design
• P is the number of higher order terms used for adding factors.
PLACKETT –BURMAN DESIGNS Question:
Ignoring center points, what is the minimum number of runs required by 2k-p designs to estimate the main effects of 8 factors?
ANS: 16 run 28-4 IV Question:
What if we can only make 12 runs in our experiment, but still wish to screen the original 8 factors and retain some of
the desirable properties of the 2k-p designs such as balance and orthogonality? ANSWER: Placket-Burman 12 run design
The below is the Plackett - Burman Design for 8 factors with 1 Block and 1 replicate. The total number of runs required is 12
4.6 Creative Problem Solving
SCAMPER is an idea generation technique that utilizes action verbs as stimuli.
This assists the person in coming up with ideas either for modifications that can be made on an existing product or for making a new
product.
SCAMPER is an acronym with each letter standing for an action verb which in turn stands for a prompt for creative ideas. •
S – Substitute
• C – Combine
• A – Adapt
• M – Modify
• P – Put to another use
• E – Eliminate
• R – Reverse / Rearrange
Idea Generation Technique - Brain-writing
Brain - writing is a technique used to generate many ideas in a short period of time. Two key modified brainstorming
techniques used are Brain-writing 6-3-5 and Constrained Brain-writing.

Brain - writing 6-3-5 - The name brain-writing 6-3-5 comes from the process of having 6 people write 3 ideas in 5 minutes on
a pre-defined parameter.

Constrained Brain-writing: The name constrained brain - writing comes the fact that on certain occasions the team
may want to have a set of constrained ideas around a pre-determined focus, rather than ranging freely. Idea
Generation Technique - Creative Brainstorming

Nominal Group Technique:


• The nominal group technique is a structured method to narrow down & prioritize on a list of choices.
• It is called “nominal” because during the session the group doesn’t engage in the usual amount of interaction typical of a
team.
• Because of this relatively low level of interaction, nominal group technique is an effective tool when some group members
are new to each other, relatively low level of interaction is required, issues are of highly controversial nature and a team is
stuck in disagreement.
• Idea Generation Technique - Modified Brainstorming
Three key modified brainstorming techniques used are: Analogy Technique, Channeling Technique and Anti-Solution
Technique.

Analogy Technique - The ideas generated on the “analogy” then get translated to the real situation (the problem at hand).
• Channeling Technique - We begin by listing “categories” of ideas for the issue at hand. Then, as the team
brainstorms, over a period of time it can “change channels” when new ideas slow down. The objective is to
capture a broad range of ideas (several channels), as well as of quantity (as many ideas as possible in each
channel).

Anti-Solution Technique - We begin by brainstorming around the opposite of the issue at hand. For example, rather than
brainstorming on ways to ensure complete information on a personal loan form we brainstorm on how to ensure we get
no/incomplete information on the personal loan form.
Assumption Busting is a brainstorming and questioning technique that identifies and challenges
conventional assumptions and consequently eliminates them if they are obstacles to optimal solution Area:
Think of new ideas for grocery store salad dressing
Immediate assumptions, based on knowledge and experience are
• Liquid
• In a bottle
• Eaten with a fork
• Eaten from a bowl
• Refrigerated
Assumption Busting
What if that’s not true

MODULE 5.0 CONTROL PHASE


5.1 Lean Controls

Lean control is a highly refined example of nonfinancial controls in action. Lean is a


system of nonfinancial controls used to improve product and service quality and
decrease waste.
How do you sustain the improvement? Now that the process problem is fixed and improvements
are in place, the team must ensure that the process maintains the gains. In the Control Phase the
team is focused on creating a Monitoring Plan to continue measuring the success of the updated
process and developing a Response Plan in case there is a dip in performance. Once in place, the
team hands these plans off to the Process Owner for ongoing maintenance.

5.1.1 Control Methods for 5S

Few critical points for Control Methods for 5S are:

• Management commitment will determine the control and self-discipline areas for an organization.
• A 5S program can be set up and operational within 5 to 6 months, but the effort to maintain world
class conditions must be continuous.
• A well run 5S program will result in a factory that is in control.
• Management commitment will determine the control and self-discipline areas for an organization.
• A 5S program can be set up and operational within 5 to 6 months, but the effort to maintain world
class conditions must be continuous.
• A well run 5S program will result in a factory that is in control.

5.1.2 Kanban

Taichi Ohno of Toyota Motor Company was the originator of the kanban method. The approach
was inspired by a management team's visit to a Piggly Wiggly supermarket in the United States,
where Engineer Taiichi Ohno observed that store shelves were stocked with just enough product
to meet consumer demand and inventory would only be restocked when there was a visual signal
-- in this case, an empty space on the shelf.
Kanban is the Japanese word for “sign” and is a method of material control in the factory. It is
intended to provide product to the customer with the shortest possible lead times. Inventory
and lead times are reduced through Heijunka (levelling of production).
For example, if the plant production goal for day 1 is 8 units of A and 16 units of B, and on day 2
it is 20 units of A and 10 units of B, the usual method is to produce all of A, followed by all of B.
This may be the most efficient use of time for the plant machinery, but, since production will
never go according to plan, the customer may change their mind on day 2 and order less of A.
This causes a pile-up of inventory and possibly increases cycle time.
To reduce the WIP and cycle time, the goal is to be able to produce each part, every day in some
order such as, 2 As, 1 B, 2As, 1 B, etc. The factory must be capable of producing such an
arrangement. It requires control of the machinery and production schedule, plus coordination of
the employees.
A goal of the kanban system is to limit the buildup of excess inventory at any point in
production. Limits on the number of items waiting at supply points are established and then
reduced as inefficiencies are identified and removed. Whenever a limit is exceeded, this points
to an inefficiency that should be addressed.
Core Practices of the Kanban Method

1. Visualize the flow of work: This is the fundamental first step to adopting and
implementing the Kanban Method. You need to visualize – either on a physical board or
an electronic Kanban Board, the process steps that you currently use to deliver your work
or your services.
Depending on the complexity of your process and your work-mix (the different types of
work items that you work on and deliver), your Kanban board can be very simple to very
elaborate.Once you visualize your process, then you can visualize the current work that
you and your team are doing. This can be in the form of stickies or cards with different
colors to signify either different classes of service or could be simply the different type of
work items. (In SwiftKanban, the colors signify the different work item types!)
If you think it may be useful, your Kanban board can have different Swim Lanes, one for
each class of service or for each work item type. However, initially, to keep things simple,
you could also just have a single swim lane to manage all your work – and do any board
redesign later.

2. Limit WIP (Work in Progress): Limiting work-in-progress (WIP) is fundamental to


implementing Kanban – a ‘Pull-system’. By limiting WIP, you encourage your team to
complete work at hand first before taking up new work. Thus, work currently in progress
must be completed and marked done. This creates capacity in the system, so new work
can be pulled in by the team.

Initially, it may not be easy to decide what your WIP limits should be. In fact, you may start
with no WIP limits. The great Don Reinertsen suggests (he did so at one of the Lean Kanban
conferences) that you can start with no WIP limits and simply observe the initial work in
progress as your team starts to use Kanban. Once you have sufficient data, define WIP
limits for each stage of the workflow (each column of your Kanban board) as being equal
to half the average WIP. Typically, many teams start with a WIP Limit of 1 to 1.5 times the
number of people working in a specific stage.
limiting WIP and putting the WIP limits on each column of the board not only helps the team members first
finish what they are doing before taking up new stuff – but also communicates to the
customer and other stakeholders that there is limited capacity to do work for any team –
and they need to plan carefully what work they ask the team to do. “An interesting side
effect of pull systems is that they limit work-in-progress (WIP) to some agreed-upon
quantity” – David J. Anderson

3. Manage Flow: Managing and improving flow is the crux of your Kanban system after you
have implemented the first 2 practices.
A Kanban system helps you manage flow by highlighting the various stages of the
workflow and the status of work in each stage. Depending on how well the workflow is
defined and WIP Limits are set, you will observe either a smooth flow within WIP limits or
work piling up as something gets held up and starts to hold up capacity. All of this affects
how quickly work traverses from start to the end of the workflow (some people call it
value stream).
Kanban helps your team analyze the system and make adjustments to improve flow so as
to reduce the time it takes to complete each piece of work.
A key aspect of this process of observing your work and resolving/ eliminating bottlenecks
is to look at the intermediate wait stages (the intermediate Done stages) and see how
long work items stay in these “handoff stages”. As you will learn, reducing the time spent
in these wait stages is key to reducing Cycle Time.
As you improve flow, your team’s delivery of work becomes smoother and more
predictable. As it becomes more predictable, it becomes easier for you to make reliable
commitments to your customer about when you will get done with any work you are
doing for them. Improving your ability to forecast completion times reliably is a big part
of implementing a Kanban system!

4. Make Process Policies Explicit: As part of visualizing your process, it makes sense to also
define and visualize explicitly, your policies (process rules or guidelines) for how you do
the work you do.

By formulating explicit process guidelines, you create a common basis for all participants
to understand how to do any type of work in the system. The policies can be at the board
level, at a swim lane level and for each column. They can be a checklist of steps to be done
for each work item-type, entry-exit criteria for each column, or anything at all that helps
team members manage the flow of work on the board well. Examples of explicit policies
include the definition of when a task is completed, the description of individual lanes or
columns, who pulls when, etc. The policies must be defined explicitly and visualized
usually on the top of the board and on each lane and column.
5. Implement Feedback Loops: Feedback loops are an integral part of any good system. The
Kanban Method encourages and helps you implement feedback loops of various kinds –
review stages in your Kanban board workflow, metrics and reports and a range of visual
cues that provide you continuous feedback on work progress – or the lack of it – in your
system.

While the mantra of “Fail fast! Fail often!” may not be intuitively understood by many
teams, the idea of getting feedback early, especially if you are on the wrong track with
your work, is crucial to ultimately delivering the right work, the right product or service
to the customer in the shortest possible time. Feedback loops are critical for ensuring
that.

6. Improve Collaboratively, Evolve Experimentally (using the scientific method): The


Kanban Method is an evolutionary improvement process. It helps you adopt small
changes and improve gradually at a pace and size that your team can handle easily. It
encourages the use of the scientific method – you form a hypothesis, you test it and you
make changes depending on the outcome of your test.

As a team implementing Lean/ Agile principles, your key task is to evaluate your process
constantly and improve continuously as needed and as possible. The impact of each
change that you make can be observed and measured using the various signals your
Kanban system provides
you. Using these signals, you can evaluate whether a change is helping you improve or
not, and decide whether to keep it or try something else.

Kanban systems help you collect a lot of your system’s performance data – either
manually, if you use a physical board, or automatically, if you use a tool such as
SwiftKanban. Using this data, and the metrics it helps you generate, you can easily
evaluate whether your performance is improving or dropping – and tweak your system as
needed.

Poka-Yoke (Mistake Proofing)


Shigeo Shingo (1986) is widely associated with a Japanese concept called poka-yoke (pronounced
poker- yolk-eh} which means to mistake proof the process. Mr.Shingo recognized that human
error does not necessarily create resulting defects. A poka-yoke is any mechanism in a lean
manufacturing process that helps an equipment operator avoid (yokeru) mistakes (poka). Its
purpose is to eliminate product defects by preventing, correcting, or drawing attention to
human errors as they occur.
Shingo (1986) lists the following characteristics of poka-yoke devices:

They permit 100% inspection

They avoid sampling for monitoring and control

They are inexpensive

Poka-yoke devices can be combined with other inspection systems to obtain near zero
defect conditions.
Errors can occur in many ways:

Skipping an operation

Positioning parts in the wrong direction

Using wrong parts or materials

Failing to properly tighten a bolt


There are numerous adaptive approaches. Gadgets or devices can stop machines from working
if a part or operation sequence has been missed by an operator. A specialized tray or dish can
be used prior to assembly to ensure that all parts are present. In this case, the dish acts as a
visual checklist. Other service-oriented checklists can be used to assist an attendant in case of
interruption.
Numerous mechanical screening devices can be utilized. Applications can be based on length,
width, height, and weight. Cash registers at many fast food outlets have descriptions or schematics
-of the product purchased. This system, in addition to the use of bar codes at supermarkets has
eliminated data entry errors and saved time. Obviously, mistake proofing is a preventive
technique.
Mistake proofing can also be accomplished through control methods by preventing human
errors or by using a warning mechanism to indicate an error. Some of the control methods to
prevent human errors include:
Designing a part so it cannot be used by mistake

Using tools and fixtures that will not load a mis-positioned part

Having a work procedure controlled by an electric relay

A signaling mechanism can alert a worker of possible sources of error. Several applications include:
Having the parts color-coded

Having tool and fixture templates in place to only accept correct parts

Having mechanisms to detect the insertion of a wrong part


A buzzer or light signal that an error has occurred, requiring immediate action. Root
cause analysis and corrective action are required before work resumes.
Other than eliminating the opportunity for errors, mistake proofing is relatively inexpensive to
install and engages the operator in a contributing way. Work teams can often contribute by
brainstorming potential ways to thwart error-prone activities. A disadvantage is that, in many
cases, that technical or engineering assistance is required during technique development.
Design improvements to mistake proof products and processes include:

Elimination of error-prone components

Amplification of human senses

Redundancy in design (back-up systems)

Simplification by using fewer components

Consideration of functional and physical environmental factors

Providing fail safe cut-off mechanisms

Enhancing product producibility and maintainability

Selecting components and circuits that are proven

Everyday Examples of Poka-Yoke :

Gas cap attached to a car

Gas pumps with automatic shut-off nozzles

110V electrical plugs and polarized sockets

Microwave automatically stops when door is opened


Seatbelt buzzer to warn drivers and passengers

Elevator electric eye to prevent door from closing on people

Lawn mower safety shut-off when bar is released

Car keys ground symmetrical to allow two-way insertion

Product drawings on cash registers at fast food restaurants

Bar codes for product identification during distribution


Poka-yoke techniques are especially effective when:

Vigilance is required in manual operations

Mis-positioning of components can occur

Attributes not measurements are important

SPC is difficult to apply

Turnover and training costs are high

Special cause failures occur frequently

Visual Management

The Goal of LEAN is to create more value for customers by eliminating activities that are considered waste

VISUAL MANAGEMENT

Create an environment
where problems are
not tolerated

Use the scientific


Create problem solving
method of PDCA to
ability in all employees
prevent recurrence

Use visula tools to


Address problems
make abnormal
immediatly
conditions stand out

Example: Visual Management


Few important techniques are:
Andon Lights
Jidohka - automation with a human touch
Visual Standards ( Do’s and Don’ts)
Visual Management Boards
Area Information Boards
Kanban Cards / system
Transparent Machine Covers and Guards
Can be used alongwith 5S tool.

Standardized Work
Standardized work is a tool for maintaining the QUALITY, PRODUCTIVITY AND SAFETY at
a high level
Standardized Work is defined as the work in which the sequence of job elements has
been efficiently organized and is followed repeatedly by a team member
Standardized Work is a process whose goal is Kaizen. If the standardized work is not
changing, that means we are regressing
Standardized work consists of three elements:

1. Takt time, which is the rate at which products must be made in a process to meet
customer demand.

2. The precise work sequence in which an operator performs tasks within takttime.

3. The standard inventory, including units in machines, required to keep the process
operating smoothly.
5.2 Statistical Process Control (SPC)

The concepts of Statistical Process Control (SPC) were initially developed by Dr. Walter Shewhart.
Statistical process control (SPC) is a technique for applying statistical analysis to measure, monitor
and control quality during the manufacturing process. .
Quality data in the form of Product or Process measurements are obtained in real-time during
manufacturing.
The major component of SPC is the use of control charting methods.
The basic assumption made in SPC is that all processes are subject to variation. This variation may
be classified as one of the two types, chance cause variation and assignable cause variation.

Also known as “special cause”, an assignable cause is an identifiable, specific cause of variation
in a given process or measurement. A cause of variation that is not random and does not occur
by chance is “assignable”.
Benefits of statistical process control include the ability to monitor a stable process and
determine if changes occur, due to factors other than random variation. When assignable cause
variation does occur, the statistical analysis facilitates identification of the source, so that it can
be eliminated.

Statistical process control also provides the ability to determine process capability, monitor
processes and identify whether the process is operating as expected, or whether the process has
changed and corrective action is required.
5.2.1 DATA COLLECTION FOR SPC

Data drives your process improvement efforts. Setting up a good data collection process is critical.
The ten steps in effective data collection are given below.

Step 1: Write down what you want to measure.

This sounds fairly trivial, but it provides a good starting point to develop a measure. Simply put
down, in writing, what you want to measure.

Step 2: Define the purpose of the data collection.

There should always be a reason why we are collecting the data. It could be that we want to
monitor performance over time and take actions on special causes of variation. It may be that we
want to use the data in a team environment to work on process improvement. Write down the
purpose of the data collection.
Step 3: Determine if other measures are appropriate for the processes involved.

This step helps ensure that you consider the processes involved. There may be other items to
consider when setting up the measurement process. Usually, it is good to look at a process from
four dimensions:
quality, quantity, timeliness, and cost. Sometimes, with a minimum amount of extra effort, you
can collect additional good data.

Step 4: Develop the operational definitions for the measure.

An operational definition imparts a clear understanding of the measure. According to Dr. W.


Edwards Deming, an operational definition includes:

• a written statement (and/or a series of examples) of criteria or guidelines to be applied


to an object or to a group.
• a test of the object or group for conformance with the guidelines that includes specifics
such as how to sample, how to test, and how to measure.
• a decision: yes, the object or the group did meet the guidelines; no, the object or group
did not meet the guidelines; or the number of times the object or group did not meet the
guidelines.

Step 5: Determine if the measurement is currently being taken and if there are historical data available.
If the measurement is currently being taken, the process becomes easier since people are already
taking the data. In this case, you will need to check to see if the data are being taken correctly. In
some cases, there will be historical data available. These data can be used to determine how the
process has worked in the past.

Step 6: Determine who will collect the data.

A decision must be made about who should collect the data. It is usually best if the person closest
to the process collects the data. This could be anyone at any level in the organization.
For example, it could be the Controller if the data being collected involve monthly profits.

Step 7: Determine how the data will be collected and how it will be displayed.

This is a crucial step. If the process for defining how the data will be collected is not correct, a lot
of time and effort can be wasted. Questions to consider include:
• Can the data be automatically collected?
• Can the data be downloaded from the system?
• Will the data have to be manually collected?
• Can the data be collected from reports?
• Should the data be displayed as a control chart or a Pareto diagram?
• What steps do I expect the data collectors to go through to collect the data?

Part of determining how the data will be collected includes writing down the procedure, either
as a process flow diagram or a step-by-step procedure. This is definitely required for new data
collection processes. It lets the data collectors know what they need to do. A decision must be
made on how frequently to collect the data. The more frequent the data collection the better.
Daily is best, followed by weekly and then monthly. Data collection less frequent than monthly is
not very useful for process improvement. Tools that are needed, such as data collection forms,
are designed at this point. Data collection forms should always include the name of the person
collecting the data, the date taken, and a place for comments.

It is best to display the data as a time series (control chart) whenever possible. The type of control
chart to use depends on the type of data you are collecting. The two types of data are attributes
and variables. Attributes data are either yes/no or counting.

Yes/No Data: For one item, there are only two possible outcomes: either it passes or it fails some
preset specification. Each item inspected is either defective (i.e., it does not meet the
specifications) or is not defective (i.e., it meets specifications). Examples of yes/no attributes data
are:

• Mail delivery: is it on time or not ontime?


• Phone answered: is it answered or not answered?
• Invoice correct: is it correct or not correct?
• Stock item: is it in stock or not in stock?
• Flipping a coin: heads or tails?

Counting Data: With counting data, you count the number of defects. A defect occurs when
something does not meet a preset specification. It does not mean that the item itself is defective.
For example, a television set can have a scratched cabinet (a defect) but still work properly. When
looking at counting data, you end up with whole numbers such as 0, 1, 2, 3; you can't have halfof
a defect. To be considered counting data, the opportunity for defects to occur must be large; the
actual number that occurs must be small. For example, the opportunity for customer complaints
to occur is large. However, the number that actually occurs is small. Thus, the number of
customer complaints is an example of counting type data.

Variables Data: Variables data consist of observations made from a continuum (such as the
temperature today). That is, the observation can be measured to any decimal place you want if
your measurement system allows it. Some examples of variables data are contact time with a
customer, sales dollars, amount of time to make a delivery, height, weight, and costs.

The control chart to use for each type of data is given below:

• Yes/No data: p control chart


• Counting data: c control chart
• Variables data: Xbar-R or Individuals chart

Set up the measurement as a positive, for example, percent on time instead of percent late.

If you are using a p chart or c chart for attributes, you are either measuring the percentage of
defective items or the number of defects. In both cases, there are defects (e.g., errors). For both of
these, you will need to include a Pareto diagram with the control chart. The Pareto diagram examines
the reasons for the defective items or defects.

A decision must be made on whether to manually keep the charts or use software to develop the
charts. Either is acceptable. EACH DATA POINT SHOULD BE PLOTTED IMMEDIATELY AFTER IT IS
COLLECTED.
Don't wait to get five data points and then plot them all at once.
Regardless of how the charts are generated, a control strategy form should accompany each
chart. This is where you record the reasons for any out-of-control points on the chart.
Step 8: Determine how to ensure that the data collection process is carried out.

Data collection usually includes a change of behavior. You are asking associates to do something
different and new. Change is never simple. This is particularly true for new data collection
systems. Even with existing data collection processes, associates may be wondering about the
sudden interest in the data. Some things to consider in implementing the data collection
processinclude:

• Explaining the reason for the data collection to the associates doing the datacollecting.
• Letting the associates know what the goal is and what will be done with the data.
• Following up after implementation to ensure that the data collection process is taking
place.

Step 9: Determine who will review the data and how often.

Too often we collect data for the sake of collecting data. We don't review the data and no
action is taken on it. Remember, the purpose of collecting data is to take some action, to improve
a process. In this case, we are helping leadership know where it stands versus the goals it has set.
This permits action to be taken if goals are not being reached.

Step 10: Implement the process.

You have planned it. Now go do it. Often you will need to make revisions to the process once it
has been implemented to help improve the quality of the data collection.

Control chart information can be used to determine the natural range of the process and to
compare it with the specified tolerance range. If the natural range is wider, then either the
specification range should be expanded, or improvements will be necessary to narrow the natural
range.
We can expect the following key information from Shewhard control charts, which will become
the basis for our action:

• Average level of the quality characteristic

• Basic variability of the quality characteristic


• Consistency of performance
Benefits from control charting are derived from both attribute and variable charts. Once the
control chart shows that a process is in control, and within specification limits, it is often possible
to eliminate costs relating to inspection.
Control charts may be used as a predictive tool to indicate when changes are required in order
to prevent the production of out of tolerance material. As an example, in a machining operation,
tool wear can cause gradual increases or decreases in a part’s dimension. Observation of a trend
in the affected dimension allows the operator to replace the worn tool before defective parts are
manufactured.

When the manufacturing method is lot production, followed by lot inspection, if inspection finds
out of tolerance parts, very little can be done other than to scrap, rework or accept the defective
parts. Using control charts, if the process changes, the process can be stopped and only the parts
produced since the last check need to be inspected. By monitoring the process during production,
if problems do arise, the amount of defective material created is significantly less than when
using batch production and subsequent inspection methods.
An additional benefit of control charts is the ability to monitor continuous improvement efforts.
When process changes are made which reduce variation, the control chart can be used to
determine if the changes were effective.

A number of samples of component coming out of the process are taken over a period of time.
Each sample must be taken at random and the size of sample is generally kept as 5 but 10 to 15
units can be taken for sensitive control charts.

5.2.1 Data collection for SPC and Selection of Variables


Develop a sampling plan to collect data (subgroups) in a random fashion at a determined
frequency. Be sure to train the data collectors in proper measurement and charting techniques.
Establish subgroups following a rational subgrouping strategy so that process variation is
captured BETWEEN subgroups rather than WITHIN subgroups. If process variation (e.g. from two
different shifts) is captured within one subgroup, the resulting control limits will be wider, and
the chart will be insensitive to process shifts.
Sampling
Sampling is a tool that is used to indicate how much data to collect and how often it should be
collected. This tool defines the samples to take in order to quantify a system, process, issue, or
problem. To illustrate sampling, consider a loaf of bread. How good is the bread? To find out, is
it necessary to eat the whole loaf? No, of course not. To make a judgment about the entire loaf,
it is necessary only to taste a sample of the loaf, such as a slice. In this case the loaf of bread being
studied is known as the population of the study. The sample, the slice of bread, is a subset or a
part of the population.

In the bakery example, bread is made in an ongoing process. That is, bread was made yesterday,
throughout today, and will be made tomorrow. For an ongoing process, samples need to be taken
to identify how the process is changing over time. Studying how the samples are changing with
control charts will show where and how to improve the process, and allow prediction of future
performance.
For example, the bakery is interested in the weight of the loaves. The bakery does not want
to weigh every single loaf, as this would be too expensive, too time consuming, and no more
accurate than sampling some of the loaves. Sampling for improvement and monitoring is a
matter of taking small samples frequently over time. The questions now become:
• How many loaves to weigh each time a sample is taken?

• How often to collect a sample?

• These two questions, “how much?” and “how often?” are at the heart of sampling.

Sampling is used any time data is to be gathered.


Data cannot be collected until the sample size (how much) and sample frequency (how often)
have been determined.

Sampling should be periodically reviewed.


When data is being collected on a regular basis to monitor a system or process, the frequency
and size of the sample should be reviewed periodically to ensure that it is still appropriate.

What questions are being asked of the data?


Before collecting any data, it is essential to define clearly what information is required. It is
easy to waste time and resources collecting either the wrong data, or not collecting enough
information at the time of data collection. Try to anticipate questions that will be asked when
analyzing the data. What additional information would be desirable? When collecting data, it
is easy to record additional information; trying to track information down later is far more
difficult, and may not be possible.

Determine the frequency of sampling.


The frequency of sampling refers to how often a sample should be taken. A sample should be
taken at least as often as the process is expected to change. Examine all factors that are
expected to cause change, and identify the one that changes most frequently. Sampling must
occur at least as often as the most frequently changing factor in the process. For example, if
a process has exhibited the behavior shown in the diagram below, how often should sampling
occur in order to get an accurate picture of the process?

Factors to consider might be changes of personnel, equipment, or materials. The questions


identified in step 1 may give guidance to this step.

Common frequencies of sampling are hourly, daily, weekly, or monthly. Although frequency is
usually stated in time, it can also be stated in number: every tenth part, every fifth purchase
order, every other invoice, for example. If it is not clear how frequently the process changes,
collect data frequently, examine the results, and then set the frequency accordingly.

Determine the actual frequency times.


The purpose of this step is to state the actual time to take the samples. For instance, if the
frequency were determined to be daily, what time of day should the sample be taken—in the
morning at 8:00 am, around midday, or late in the day around 5:00 pm? This is important because
inconsistent timing between data gathering times will lead to data that is unreliable for further
analysis. For example, if a sample is to be taken daily, and on one day it is taken at 8:00 am, the
next day at 5:00 pm, and the following day at midday, the timing between the samples is
inconsistent and the collected data will also be inconsistent. The data will exhibit unusual
patterns and will be less meaningful. Stating the time that the sample is to be taken will reduce
this type of error. The actual time should be chosen as close to any expected changes in the
process as possible, and when taking a sample will be convenient. Avoid difficult times, such as
during a shift change or lunch break.”

Select the subgroup (sample) size.


A subgroup (or sample) is the number of items to be examined at the same time. The terms
“subgroup” and “sample” may be used interchangeably. When doing calculations, subgroup size
is denoted by the letter n. To choose the most appropriate subgroup size, determine first whether
the data being collected is “variables data” or “attributes data.”

For variables data: When measuring variables data, a subgroup size larger than one is preferable
because larger subgroups sizes yield greater possibilities for analysis. However, it may not be
possible to get a subgroup size larger than one. Some examples of this are electricity usage per
month, profit per month, sales per month, temperature of a room, and the viscosity of a fluid. In
situations such as these when a subgroup size larger than one does not make sense, the subgroup
(or sample) size is equal to one.

If a subgroup size larger than one can be chosen, the size is usually between three and eight. A
subgroup size between three and eight has been determined to be statistically efficient. The most
commonly-used subgroup size is five. When more data is desired, the frequency of taking
samples, not the subgroup size, should be increased.

When a sample is taken, it should be selected to assure that conditions within the sample are
similar. If gathering a sample size of five, for example, take all five pieces in a row as they are
produced in the process. This is known as a rational subgroup.
For attributes data: The subgroup size for attributes data depends on the process being sampled.
The general rule of thumb is to gather a large enough sample so that all possible characteristics
being investigated will appear. That is, the sample is large enough that a “0” occurrence is rare.

Begin by answering the question, “How many items does this process produce during the
frequency interval (per hour, week, etc.)?” When that number is determined, the sample size
should be at least the square root of that number. For instance, if a purchasing department
processes 100 purchase orders per week, an appropriate sample size would be 10 purchase
orders per week (the square root of 100 is 10.

Operational definition

An operational definition, when applied to data collection, is a clear, concise detailed definition of a
measure. The need for operational definitions is fundamental when collecting all types of data. It is
particularly important when a decision is being made about whether something is correct or
incorrect, or when a visual check is being made where there is room forconfusion. For example, data
collected will be erroneous if those completing the checks have different views of

what constitutes a fault at the end of a glass panel production line. Defective glass panels may be
passed and good glass panels may be rejected. Similarly, when invoices are being checked for
errors, the data collection will be meaningless if the definition of an error has not been specified.

When collecting data, it is essential that everyone in the system has the same understanding and
collects data in the same way. Operational definitions should therefore be made before the
collection of data begins.

Any time data is being collected, it is necessary to define how to collect the data. Data that is not
defined will usually be inconsistent and will give an erroneous result. It is easy to assume that
those collecting the data understand what and how to complete the task. However, people have
different opinions and views, and these will affect the data collection. The only way to ensure
consistent data collection is by means of a detailed operational definition that eliminates
ambiguity.
Example 1 Attributes
Number of black spots per radiator grill.
Characteristic of
interest:

The observation will be performed with the naked eye (or with
Measuring corrective lenses if normally worn), under the light available in the work
instrument: station(in 100% working order, i.e., no burned-out bulbs).

The number of black spots per radiator grill will be counted by taking
Method of test: samples at the work station. The sample should be studied at a distance
of 18 inches (roughly half an arm’s length) from the eye. Only the top
surface of the grill is to be examined.

Wipe the top surface of the grill with the palm of your hand and look
Decision criteria:
for any black specks embedded in the plastic. Any observed black speck
of any size counts as a black spot.

Example 2 Variables

Characteristic of
interest:
Diameter of 48 inch rod
Measuring instrument: Micrometer

The sample size is n=3. Measure 3 rods every hour. When the grinder

releases the rod, take one measurement each at 8" down, 24" down,
Method of test: and 40" down from the notched end. Tighten the micrometer as much as
possible.
Record to 4 decimal points. If the fifth number to the right of the
decimal point is 5 or higher, round the fourth number up one.

Given the benefits of control charting, one might be tempted to control chart every characteristic
or process variable. The logic is if any characteristic changes, then the process can be stopped.
This decision would also eliminate the need to determine if one characteristic is more important
than another.
The risk of charting many parameters is the operator will spend so much time and effort
completing the charts, that the actual process becomes secondary. When a change does occur,
it will most likely be overlooked. When more than a few charts are used for a process, the benefits
may decrease, as quickly as the costs increase.
Some considerations for the selection of a control chart variable include:

• Items that protect human safety

• Items that protect the environment or community

• Items that are running at a high defective rate

• Key process variables that impact the product

• Major sources of customer complaints

• Items that show adherence to applicable standards

• Items that are requested by key customers

• Variables that have caused processing difficulties

• Variables that can be measured by the person charting

• Items that can be counted by the person charting

• Items that contribute to high internal costs

• Variables that help control the process

In an ideal case; one process variable is so critical that it is indicative of the process, as a whole.
Key process input variables (KPlVs) may be analyzed to determine the degree of their effect on a
process. Key process output variables (KPOVs) are ideal for determining process capability and
for process monitoring using control charts.

Design of experiments and analysis of variance may be used to identify the variables which are
most significant to process control. Pareto analysis can be used to identify key internal and
external losses.
Control Chart Anatomy
Points to keep in mind while selecting a control chart: Variables Data • For
continuous data we can measure the average and the variation, thus X
bar &R (Range) or X bar and S (Standard deviation) can be used
Attribute Data

• Determine what are we measuring – Defects or Defective

In case we are capturing defective data then determine if we are sampling for subgroup of equal
sample size or not • In case if we are capturing data for defects then determine if the
opportunity for the defects are the same for each subgroup or not.

• Based on the above, select the appropriate chart

Control Chart Selection is based on the Data Type.


With such a powerful tool as Control Chart in our hands, one would definitely be interested to
know where and how to use it for predicting the process performance. The below flow chart
would help in determining the Control Charts to be used based on different data types,
samples/subgroups and defects/defectives.

• If the data is variable data with the sub-group size as 1, we can use I-MR (Individuals
Moving Range) Chart.

• If the data is variable data with sub-group size more than 1 but less than 8, we can use X-
Bar R (Mean and Range) Chart. • If the data is variable data with sub-group size more
than 8, we can use X-Bar S (Mean and Standard Deviation) Chart.

• If the data is attribute data, we capture Defectives and subgroup size is the same, we use
np Chart.

• If the data is attribute data, we capture Defectives and subgroup size is varying, we use p
Chart. • If the data is attribute data, we capture Defects and subgroup size is the same,
we use c Chart.If the data is attribute data, we capture Defects and subgroup size is the
varying, we use u Chart.
Dfm / Attrlb1te
ete Dat
u

E( -. :i1 n p
I

Dele ctt11e
Uri (Si11 e
M1ltiplt,ID@h/ Unt Def ct/uj,ll)
.1 cls t

Il';a 1 n= 2 lo 9 /l a.:> 1 0
Vari ble

np-cilart p<{hart
X•MR/l•MR i-R j-Sdmt
di rt chart

5.2.2 I-MR Chart


Individual Moving Range or as it’s commonly referenced term I-MR, is a type of Control Chart that
is commonly used for Continuous Data . This was developed initially by Walter Shewart and hence
the Control Charts are sometimes also referred to as Shewart Chart.

As the term indicates, in I-MR we have two charts – one with Individual data points (Individual
Chart or I Chart) and the other chart has data points that are range values derived between
consecutive individual points (Moving Range Chart or MR Chart). The two charts are useful
together and using only one may not give the complete information on the behavior of the
process.

The I-MR Chart data cannot be sub-grouped as in the case of -R Chart as individual data points
and data points of ranges are important for creating and interpreting the process information
correctly. It should be noted that the data points should be in timely order i.e., data points
captured in the time order should be maintained in the same order and not re-ordered for our
convenience.
As mentioned , there are two charts for I-MR, Individual Chart and Moving Range Chart – let’s
discuss it one-by-one and see how it compliments each other to interpret the process
information.
The formula for calculating the Lower Control Limits (LCL) and Upper Control Limits (UCL) are:

Control Limits for I Chart =


Control Limits for MR Chart

Where,

With the calculations in hand, it will be lot easier for us to start our work. As already discussed,
we have two charts in I-MR –

Individual Chart plotting the individual data points over a period of time. This is useful in detecting
the trends and shift that are present in the process. This will help us visualize the common cause
and the special cause variations, if any. It should be noted that the data generated should be
sequential i.e., in the same order in which it was captured against time axis, as the trends and
shifts are studied and best interpreted against time performance.

Moving Range Chart is as the name indicates, is a chart which is created by plotting the values
derived from the time-ordered sequential data. Each Moving Range point is calculated as Xn –
Xn- 1 and hence we will have one data point lesser than that in the Individual Chart.
Moving Range Chart
The MR Chart helps us in assessing the stability of the process caused by the variation between
consecutive individual data points. Whenever the points are out of Control Limits, it indicates
that the process is unstable. This will help us in identifying the Special/Assignable Cause that
triggered the process to become unstable. After exploring the reason for the cause of outlier, the
same should be fixed and removed from both I and MR Chart. Whenever we find the variation to
be huge in MR chart, we should refer to the I-Chart and determine the cause of the variation as
the I-Chart will have the exact individual data points against the time period which had caused
the high variation.

So, to conclude, one must know that the IMR chart can be plotted for individual data points where
n=1. The data points should always be continuous and captured in time sequential order over a
period of time. With this information in mind, it will be easier to work with the IMR data.

5.2.3 X bar- R chart

An X-bar and R (range) chart is a pair of control charts used with processes that have a subgroup
size of two or more. The standard chart for variables data, X-bar and R charts help determine if
a process is stable and predictable. The X-bar chart shows how the mean or average changes over
time and the R chart shows how the range of the subgroups changes over time. It is also used to
monitor the effects of process improvement theories. As the standard, the X-bar and R chart will
work in place of the X-bar and s or median and R chart.

The X-bar chart, on top,shows the mean or average of each subgroup. It is used to analyze central location
. The range chart, on the bottom, shows how the data is spread . It is used to study system variability .
You can use X-bar and R charts for any process with a subgroup size greater than one. Typically,
it is used when the subgroup size falls between two and ten, and X-bar and s charts are used with
subgroups of eleven or more.

Typically, twenty to twenty-five subgroups will be used in control limit calculations.

X-bar and R charts have several applications. When you begin improving a system, use them to
assess the system’s stability .

After the stability has been assessed, determine if you need to stratify the data. You may find
entirely different results between shifts, among workers, among different machines, among lots
of materials, etc. To see if variability on the X-bar and R chart is caused by these factors, collect
and enter data in a way that lets you stratify by time, location, symptom, operator, and lots.

You can also use X-bar and R charts to analyze the results of process improvements. Here you
would consider how the process is running and compare it to how it ran in the past. Do process
changes produce the desired improvement?
Finally, use X-bar and R charts for standardization. This means you should continue collecting and
analyzing data throughout the process operation. If you made changes to the system and stopped
collecting data, you would have only perception and opinion to tell you whether the changes
actually improved the system. Without a control chart, there is no way to know if the process has
changed or to identify sources of process variability.

5.2.4 U chart

In statistical quality control, the u-chart is a type of control chart used to monitor "count"-type
data where the sample size is greater than one. U-charts show how the process, measured by the
number of nonconformities per item or group of items, changes over time. Nonconformities are
defects or occurrences found in the sampled subgroup. They can be described as any
characteristic that is present but should not be, or any characteristic that is not present but
should be. For example, a scratch, dent, bubble, blemish, missing button, and a tear are all
nonconformities. U-charts are used to determine if the process is stable and predictable, as well
as to monitor the effects of process improvementtheories.
Examples of processes suitable for monitoring with a u-chart include:

• Monitoring the number of nonconformities per lot of raw material received where
the lot size varies
• Monitoring the number of new infections in a hospital per day • Monitoring
the number of accidents for delivery trucks per day
5.2.5 P chart
A p-chart is an attributes control chart used with data collected in subgroups of varying sizes.
Because the subgroup size can vary, it shows a proportion on nonconforming items rather than
the actual count. P-charts show how the process changes over time. The process attribute (or
characteristic) is always described in a yes/no, pass/fail, go/no go form. For example, use a p-
chart to plot the proportion of incomplete insurance claim forms received weekly. The
subgroup would vary, depending on the total number of claims each week. P-charts are used to
determine if the process is stable and predictable, as well as to monitor the effects of process
improvement theories

An np-chart is an attributes control chart used with data collected in subgroups that are the same
size. Np-charts show how the process, measured by the number of nonconforming items it
produces, changes over time. The process attribute (or characteristic) is always described in a
yes/no, pass/fail, go/no go form. For example, the number of incomplete accident reports in a
constant daily sample of five would be plotted on an np-chart. Np-charts are used to determine
if the process is stable and predictable, as well as to monitor the effects of process improvement
theories.
The np-chart shows the number of nonconforming units in subgroups of set sizes.

5.2.7 Xbar-S chart

Use the S chart when your subgroup sizes are 9 or greater. S charts use all the data to calculate
the subgroup process standard deviations. You should consider using S charts for processes with
a high rate of production or when data collection is quick and inexpensive.An S-chart is a type of
control chart used to monitor the process variability (as the standard deviation) when measuring
subgroups (n ≥ 5) at regular intervals from a process.

Each point on the chart represents the value of a subgroup standard deviation.

The center line for each subgroup is the expected value of the standard deviation statistic. Note
that the center line varies when the subgroup sizes are unequal.

The control limits are either:


A multiple (k) of sigma above and below the center line. Default k=3.

Probability limits, defined as the probability (alpha) of a point exceeding the limits. Default
alpha=0.27%.
If unspecified, the process sigma is the weighted average of the unbiased subgroup estimates of
sigma based on the standard deviation statistics.
The individual observations are assumed to be independent and normally distributed. Although
the chart is fairly robust and nonnormality is not a serious problem unless there is a considerable
deviation from normality (Burr, 1967).
For subgroup sizes less than 6 with k-sigma control limits, the lower limit is zero. In these cases,
probability limits

5.2.8 Cumsum chart

Cumsum charts, while not as intuitive and simple to operate as Shewhart charts, have been
shown to be more efficient in detecting small shifts in the mean of a process.In particular they
are better than Shewhart control charts when it is desired to detect shifts in the mean that are 2
sigma or less. A visual procedure known as the V-Mask, is sometimes used to determine whether
a process is out of control.A V- Mask is an overlay shape in the form of a V on its side that is
superimposed on the graph of the cumulative sums.The origin point of the V-Mask (see diagram
next slide) is placed on top of the latest cumulative sum point and past points are examined to
see if any fall above or below the sides of the V.

As long as all the previous points lie between the sides of the V, the process is in control.
Otherwise (even if one point lies outside) the process is suspected of being out ofcontrol.
In the diagram above, the V-Mask shows an out of control situation because of the point that lies
above the upper arm. By sliding the V-Mask backwards so that the origin point covers other
cumulative sum data points, we can determine the first point that signaled an out-of-control
situation. This is useful for diagnosing what might have caused the process to go out of control.

5.2.9 EWMA chart

The Exponentially Weighted Moving Average (EWMA) is a statistic for monitoring the
process that averages the data in a way that gives less and less weight to data as they are
further removed in time.
For the Shewhart chart control technique, the decision regarding the state of control of
the process at any time, t, depends solely on the most recent measurement from the
process

For the EWMA control technique, the decision depends on the EWMA statistic, which is
an exponentially weighted average of all prior data, including the most recent
measurement.
By the choice of weighting factor, λ, the EWMA control procedure can be made sensitive
to a small or gradual drift in the process, whereas the Shewhart control procedure can
only react when the last data point is outside a control limit.
As with all control procedures, the EWMA procedure depends on a database of
measurements that are truly representative of the process. Once the mean value and
standard deviation have been calculated from this database, the process can enter the
monitoring stage, provided the process was in control when the data werecollected.
The red dots are the raw data; the jagged line is the EWMA statistic over time. The chart tells us
that the process is in control because all EWMAt lie between the control limits. However, there
seems to be a trend upwards for the last 5 periods.

5.2.12 Subgroups, Impact of variation & Frequency of sampling


Statistical Process
Control Chart Subgroups Subgroups Control (SPC) –
Rational Subgrouping
Selection of subgroups
Generally, subgroups are
A control chart selected in a way that provides a
statistical makes each subgroup as
However, this
test to determine if the variation from homogeneous as selection depends
upon a knowledge of
possible and that gives the sample-to-sample is maximum
the components of
opportunity for consistent with the variation from one the total process
variation.
average variation subgroup to another. within the sample.
A control chart provides a statistical test to determine if the variation from sample-to-sample is consistent with the average variation
within the sample.
The key idea in the Shewhart control chart is the division of observations into what are called rational subgroups.
The success of charting depends a great deal on the selection of these subgroups.
Generally, subgroups are selected in a way that makes each subgroup as homogeneous as possible, and that gives the maximum
opportunity for variation from one subgroup to another.
However, this selection depends upon a knowledge of the components of the total process variation.

Statistical Process Control (SPC) – Subgroups, Impact of variation & Frequency of sampling
Where order of production is used as a basis for subgrouping, two fundamentally different approaches are possible:
• The first subgroup consists of product produced as nearly as possible at one time.
• This method follows the rule for selection of rational subgroups by permitting a minimum chance for
variation within a subgroup and a maximum chance for variation from subgroup-to-subgroup.
• Another subgroup option consists of product intended to be representative of all the production over a
given period of time.
• Product may accumulate at the point of production, with a random sample chosen from all the product
made since the last sample.
Statistical Process Control (SPC) – Center line & Control Limit calculations Statistical
Process Control (SPC) – Rational Subgrouping (Continued)
In production control charting, it is very important to maintain the order of production. A charted process which shows out of
control conditions (and resulting opportunities for correction) may be mixed to create new X - R charts which demonstrate
remarkable control. By mixing, chance causes are substituted for the original assignable causes as a basis for the differences among
subgroups.
Where order of production is used as a basis for subgrouping, two fundamentally different approaches are possible:
• The first subgroup consists of product produced as nearly as possible at one time. This method follows the rule for
selection of rational subgroups by permitting a minimum chance for variation within a subgroup and a maximum chance
for variation from subgroup-to-subgroup.
• Another subgroup option consists of product intended to be representative of all the production over a given period of
time. Product may accumulate at the point of production, with a random sample chosen from all the product made since
the last sample.
If subgrouping is by the first method, and a change in the process average takes place after one subgroup is taken and is corrected
before the next subgroup, the change will not be reflected in the control chart. For this reason, the second method is sometimes
preferred when one of the purposes of the control chart is to influence decisions on acceptance of product.
The choice of subgroup size should be influenced, in part, by the desirability of permitting a minimum chance for variation within a
subgroup. In most cases, more useful information will be obtained from, say, five subgroups of 5 rather than from one subgroup of
25. In large subgroups, such as 25, there is likely to be too much opportunity for a process change within the subgroup.

Statistical Process Control (SPC) – Rational Subgrouping (Continued)


In production control charting, it is very important to maintain the order of production. A charted process which shows out of
control conditions (and resulting opportunities for correction) may be mixed to create new X - R charts which demonstrate
remarkable control. By mixing, chance causes are substituted for the original assignable causes as a basis for the differences among
subgroups.
Where order of production is used as a basis for subgrouping, two fundamentally different approaches are possible:
• The first subgroup consists of product produced as nearly as possible at one time. This method follows the rule for selection
of rational subgroups by permitting a minimum chance for variation within a subgroup and a maximum chance for variation
from subgroup-to-subgroup.
• Another subgroup option consists of product intended to be representative of all the production over a given period of time.
Product may accumulate at the point of production, with a random sample chosen from all the product made since the last
sample.
If subgrouping is by the first method, and a change in the process average takes place after one subgroup is taken and is corrected
before the next subgroup, the change will not be reflected in the control chart. For this reason, the second method is sometimes
preferred when one of the purposes of the control chart is to influence decisions on acceptance of product.
The choice of subgroup size should be influenced, in part, by the desirability of permitting a minimum chance for variation within a
subgroup. In most cases, more useful information will be obtained from, say, five subgroups of 5 rather than from one subgroup of
25. In large subgroups, such as 25, there is likely to be too much opportunity for a process change within the subgroup. Statistical
Process Control (SPC) – Center line & Control Limit calculations
• Control Limits are not Specification Limits
• Control Limits are calculated from the process output data
• Specification Limits are provided by the Engineering Department based on the functional requirement • Control Limits are
decided based on the process behavior.

Statistical Process Control (SPC) – Rational Subgrouping (Continued)


In production control charting, it is very important to maintain the order of production. A charted process which shows out of
control conditions (and resulting opportunities for correction) may be mixed to create new X - R charts which demonstrate
remarkable control. By mixing, chance causes are substituted for the original assignable causes as a basis for the differences among
subgroups.
Where order of production is used as a basis for subgrouping, two fundamentally different approaches are possible:
• The first subgroup consists of product produced as nearly as possible at one time. This method follows the rule for
selection of rational subgroups by permitting a minimum chance for variation within a subgroup and a maximum chance
for variation from subgroup-to-subgroup.
• Another subgroup option consists of product intended to be representative of all the production over a given period of
time. Product may accumulate at the point of production, with a random sample chosen from all the product made since
the last sample.
If subgrouping is by the first method, and a change in the process average takes place after one subgroup is taken and is corrected
before the next subgroup, the change will not be reflected in the control chart. For this reason, the second method is sometimes
preferred when one of the purposes of the control chart is to influence decisions on acceptance of product.
The choice of subgroup size should be influenced, in part, by the desirability of permitting a minimum chance for variation within a
subgroup. In most cases, more useful information will be obtained from, say, five subgroups of 5 rather than from one subgroup of
25. In large subgroups, such as 25, there is likely to be too much opportunity for a process change within the subgroup.

5.3 Six Sigma Control Plan

5.3.1 Cost Benefit Analysis

Six sigma is about making money. It is about profitability, although improved quality and
efficiency are immediate by products. The financial benefits of six sigma projects are the
measurements that create a link between philosophy and action.
Financial benefits and associated risks are the factors used to evaluate, prioritize, select, and
track all six sigma projects. This Section describes the common financial measures, methods for
risk analysis, and the features of quality cost systems used for this purpose.

Project cost-benefit analysis is a comparison to determine if a project will be (or was)


worthwhile. The analysis is normally performed prior to implementation of project plans and is based
on time- weighted estimates of costs and predicted value of benefits.

The cost-benefit analysis is used as a management tool to determine if approval should be given
for the project go-ahead. The actual data is analyzed from an accounting perspective after the
project is completed to quantify the financial impact of the project.

The sequence for performing a cost-benefit analysis is:

• Identify the project benefits.

• Express the benefits in dollar amounts, timing, and duration.

• Identify the project cost factors including materials, labor and resources.

• Estimate the cost factors in terms of dollar amounts and expenditureperiod

• Calculate the net project gain (loss)

• Decide if the project should be implemented (prior to starting), or if the project was
beneficial (after completion)

• If the project is not beneficial using this analysis, but it is management’s desire to
implement the project, what changes in benefits and costs are possible to improve the
cost-benefit calculation?
Cost benefit analysis of six sigma implementation
5.3.2 Elements of Control Plan

A control plan is a document describing the critical to quality characteristics, the critical Xs or Ys,
of the part or process. Through this system of monitoring and control, customer requirements
will be met and the product or process variation will be reduced.

The intent of the control plan is to formalize and document the system of control that will be
utilized.
However, the control plan should not be a replacement for detailed operator instructions in the
form of work instructions or standard operating procedures.
Each part or process must have a control plan. A group of common parts using a common process can
be covered by a single control plan.

Types of Control Plans:


For the automotive sector, lSO/TS 16949:2002 and the Advanced Product Quality Planning APQP
(2000), identify three control plan phases:

• Prototype

• Pre-launch

• Production
A prototype control plan is used in the early development stages when the part or process is
being defined or configured. This control plan will list the controls for the necessary dimensional
measurements, types of materials, and required performance tests
A pre-launch control plan is used after the prototype phase is completed and before full
production is approved. lt lists the controls for the necessary dimensional measurements, types
of materials, and performance tests. This plan will have more frequent inspections, more in-
process and final check points, some statistical data collection and analysis, and more audits. This
stage will be discontinued once the pre-launch part or process has been validated and approved
forproduction.
A production control plan is used for the full production of a part. It contains all of the line items
for a full control
Plan: part or product characteristics, process controls, tests, measurement system analysis, and
reaction plans.
A more detailed list of input factors is provided later in this session.

The control phase is the forgotten C in DMAIC. The project control phase is necessary in order to
sustain the project gains. The control plan must be a living document for it to remain an effective
mechanism to monitor and control the process.

A responsible person must be placed in charge of the control plan. This ensures successful
monitoring and updating. A black belt may or may not be a suitable person for the role, as he/she
may be replaced or transferred to a different position. A better selection would be the process
owner.

The current process owner can be listed on the control plan, but in reality it is a functional role
that is to be passed on to the next individual in that same organizational position. If the control
plan is not maintained, the benefits of the project could slowly be lost. The frequent changing of
process owners, combined with large numbers of process projects, can easily result in neglected
or lost control plans.

Some considerations in the closing phase of the project include:

• Identify the process owner

• Involve the team in the control plan

• Create new or updated work instructions and procedures


• Notify and train the affected personnel

• Ensure that the control plan training is effective


• Place the control plan in the proper quality system document

• Attain agreement between the team members and process owner

A number of inputs or sources contribute to understanding, manufacturing and controlling the


part or process. Many of the following are included:

• Process flow diagrams

• System FMEAs, DFMEAs, and PFMEAs

• Cause-and-effect analysis

• Special customer characteristics

• Historical data

• Lessons learned

• Team process knowledge

• Design reviews

• Quality function deployment

• Designed experiments

• Statisticalapplications

• Multi-vari studies

• Regression analysis

Control plan: Provide a title for the control plan. The control plan is often placed into another
document, such as an operating instruction or six sigma database. If necessary, indicate if this is
a prototype, pre-launch, or production plan.
Control number: Provide a reference number. This number may be supplied by the responsible
department.

Team members: If a cross functional team is involved, provide the members names.
Contact person: This could be the black belt in charge of the project, however, the name and
function of the process owner are more important.
Page: Provide page numbers if the control plan exceeds one page. The examples shown in
this description are brief. Control plans may run up to 20 pages. Original date: Indicate the
original date of issue of the control plan. Revision date: Provide the latest revision date of the
control plan. Part/process: List the part number or the process flow being charted.

Key input variable (X): Note the key input variable, when appropriate. On any line item, only the X
or Y variable is filled out, not both. This is to clearly indicate which item is being monitored and
controlled.
Key output variable (Y): Note the key output variable, when appropriate.
Special characteristic note: Indicate if a special characteristic is to be monitored and controlled.

Specifications: For manufacturing applications, the engineering specifications for the part should
be monitored and controlled. For other applications, one would provide upper and lower
specification limits, as well as the target value

Measurement gage technique: The gage or measurement technique should be described. The
gage, tool, fixture, or test device used for data collection must be in conformance with the
needed measurement system analysis (MSA) requirements. When necessary, this would include:
linearity, stability, accuracy, reproducibility, repeatability, and uncertainty analysis. This can be
more difficult when attribute data is of concern. The AIAG MSA manual is a goodguide. Gage
capability: Provide the current capability of the measurement system. The AlAG MSA manual
lists: Under 10% error as acceptable

10% to 30% error may be acceptable, depending on the cost and the situation

Over 30% error is not acceptable

Measurement devices may need uncertainty determinations.


Sample Size: Provide the Sample frequency: List how often the inspection or monitoring of the
part or process is required.
Initial CPR: This provides an indication of process capability.
Person responsible for measurement: indicate who will make and record the measurement.
Control method: Note how this X or Y variable will be controlled. Examples include control charts, checklists,
visual inspections, automated measurements, etc.
Reaction plan: Describe what will happen if the variable goes out of control. How should the
responsible person respond to the situation?
Control plan construction is often led by the black belt incharge of the six sigma project. The team
is usually cross functional with individuals from different areas, including the process owner. The
team will ensure that the control plan contains the critical variables, the Xs and the Ys, of the
product or process. The control plan must show compliance and control before project closure.
A successful control plan should remain a living document to ensure that the benefits of the
project are fully realized.

5.3.3 Elements of Response Plan


An effective control system is characterized by formal documents. These documents provide
directions to the employees on how to accomplish a task, who is responsible for performing the
task, or how the company systems work. There are various names for these documents
including:

• Manuals (including the quality manual)

• Procedures

• Standard operating procedures

• Work instructions

• Records

Many companies organize the documentation into a hierarchy. The manual is the highest level
document in the system. Procedures are at the second level and describe the responsibilities of
various personnel and the administrative system is used to accomplish the tasks. The manual
details what is to be done and procedures describe who will do it.

The third level is the work instructions that describe how to do the tasks.
The work instructions detail the specific steps to accomplish the goals defined in the manual and
the procedures.
Some organizations include the standard operating procedures (SOPs) at this level, and some
include them as part of the procedures level.

Records include the data collected on the products and processes.


The basic content of any good procedure or instruction should include:

• Purpose of the document

• Basis of the document

• Scope of the document


Documentation is necessary for the continued success of a company. Formal procedures or
instructions have the following benefits and characteristics.
• Procedures or instructions are a means for management to describe, in writing, and in a
readily accessible manner, their required modes of operations.

• Procedures or instructions are not static, they must be continuously adjusted ina
controlled manner to meet changing times and conditions.

• Procedures or instructions are a reasonably simple vehicle for defining and standardizing
proven methods.
Operational procedures define how policy requirements are to be implemented in terms explicit
enough to be easily understood.

• Procedures or instructions are a means of establishing continuity of operations when


personnel changes occur and as training for new employees.

• Procedures or instructions prevent undesirable changes.

• Procedures or instructions provide a written standard to which operations may be


audited.

A good documentation system can add value to a company. Company documents must be
meaningful, and developed with specific detail so the operations can be improved.
Documented procedures allow the improvement of processes (both administrative and
technical) by first establishing a baseline.

The baseline is established by defining the steps of the process.


This definition of the process steps then can be used for subsequent improvements.
Following are the general guidelines for documentation:

• Keep the documentation simple

• Keep the documentation clear and inviting

• Include options and instructions for “emergencies”

• Keep the documentation brief

• Keep the documentation handy

• Have a process for updates and revisions

Documentation must be written to the level that is understood by the users. It should also reflect
the current processes and methods. After completion of a process improvement, the
documentation should correspond to the new methods, and the users should be trained on the
new documentation.
It may also be necessary to revise the procedures used to detect and eliminate potential
problems. The corrective action procedure itself should be revised when needed.
Effective project or process improvement activities should ultimately lead to the advancement
of company operations. However, this is not automatic. Continuous improvement takes the
concentrated and continuing efforts of everyone
In addition to changes resulting from improvements, there should be a balanced mix of
measurements to monitor overall process performance. Examples include:

• Performance results

• Quality results
• Changes in customer requirements

• Financial results

• Benchmarking results

• Process capability measurements

• Audit results

• SWOT analysis

The above results are often reported in management reports. In almost all situations, graphs and
charts are preferable to texts and columns of numbers.

The organization should have an oversight or executive committee to respond to both problems
and opportunities. Many of the approaches discussed earlier in gap, root cause, and risk analysis
are effective tools for the ongoing evaluation of the improvement process.
References

▪ http://isoconsultantpune.com/team-management-improvement-projects/
▪ https://www.greycampus.com/opencampus/lean-six-sigma-black-belt/ six-sigma-roles-

▪ https://www.linkedin.com/pulse/0064419-257374311-voice-of-the-business-
customer- process-and-employee
▪ http://www.sixsigmatrainingfree.com/the-problem-solving-strategy-y--fx-in-lean-six-
sigma.html
▪ https://www.brighthubpm.com/six-sigma/32500-most-common-six-sigma-roles/
▪ http://isoconsultantpune.com/business-process-management/
and-responsibilities

▪ https://www.isixsigma.com/implementation/change-management-
implementation/
▪ voice-business-customer-process-and-
▪ employee/
▪ https://en.wikipedia.org/wiki/Six_Sigm
a
https://demix.org/models/six-
sigma
http://www.sixsigmatrainingfree.com/basic-lean-six-sigma-metrics.html
▪ https://www.sixsigma-institute.org/
▪ http://www.whatissixsigma.net/pareto-chart-and-
▪ analysis/
https://flccoc.org/wp-content/uploads/2018/02/RFQ-for-CCOC-IT-
REVIEW.pdf
▪ http://www.cellaconsulting.com/blog/defining-process-dont-forget-the-basics/
▪ https://www.projectsmart.co.uk/pareto-analysis-step-by-step.php
▪ http://www.managementjournal.info/index.php/IJAME/article/download/271/ 260
▪ http://howardgitlow.com/sixsigmadictionary.htm
▪ ▪ https://www.slideshare.net/peterjm68/darren-foster-asb-presentation-2017
http://www.improhealth.org/fileadmin/Documents/Improvement_Tools/

Pareto_Analysis.pdf

http://www.sdgsource.com/quality/
▪ https://www.brighthubpm.com/six-sigma/52721-explaining-tollgate-reviews
▪ https://accountingexplained.com/managerial/capital-budgeting/payback-period
▪ https://www.bartleby.com/essay/Arctic-Power-Case-Study-F3CBDR2ZVJ
▪ http://www.who.int/cancer/palliative/guidetodevelopment/en/
▪ http://highperformanceteams.org/hpt_chtr.htm
▪ https://www.slideshare.net/YousufRazzaq1/ project-analysis-and-methods-of-project-
evaluation

https://www.belimo.us/mam/americas/technical_documents/pdf-web/tools_pdf/

https://www.investopedia.com/terms/n/npv.as

p
http://ejournal-balitbang.kkp.go.id/index.php/ifrj/article/download/92/ 86
▪ http://www.agadvance.com/issues/mar-2014/ calculating-npv-takes-guesswork-out-of-
farmland-purchases.aspx
▪ http://isoconsultantpune.com/project-charter/
▪ https://sixsigmastudyguide.com/history-of-lean/
▪ https://www.kaneisable.com/blog/introduction-to-six-sigma-methods

https://www.hhs.gov/idealab/2016/01/06/innovation-terms-and-methodologies-a-
starting-place/
https://www.graphicproducts.com/articles/creating-a-visual-factory-with-the-
5s-
system/

http://www.iise.org/Details.aspx?id=83
5
▪ http://traindc.com/courses/lean-and-six-sigma-training/
▪ https://www.villanovau.com/resources/six-sigma/six-sigma-vs-lean-six-sigma
▪ https://www.edrawsoft.com/6m-method.php
ttp://qualitycommunityschools.weebly.com/uploads/4/1/6/1/41611/
problem_solving_tools_5_whys.pdf

▪ http://electrical-engineering-portal.com/ functional-specification-scada-project
▪ https://www.smartdraw.com/cause-and-effect/
▪ http://www.growingscience.com/dsl/Vol3/dsl_2014_15.pdf
▪ https://www.lucidchart.com/pages/p-and-id
▪ https://www.hqsc.govt.nz/assets/Consumer-Engagement/Publications/Training-
guide/016-health-care-quality-tools.pdf
▪ http://asq.org/learn-about-quality/process-analysis-tools/overview/ flowchart.html
▪ http://www.conceptdraw.com/examples/ design-elements-cross-functional-flowcharts
▪▪

▪ http://isoconsultantpune.com/process-analysis-tools/
▪ https://camo.ici.ro/journal/vol16/v16a16.pdf
▪ http://www.dummies.com/careers/project-management/six-sigma/ six-sigma-failure-
mode-effects-analysis/
▪ http://web.ipac.caltech.edu/staff/fmasci/home/astro_refs/ SkewStatSignif.pdf
▪ https://www.statcan.gc.ca/edu/power-pouvoir/ch12/5214889-eng.htm
▪ https://www.ncbi.nlm.nih.gov/pmc/articles/PMC3198538/
▪ ttps://rashidfaridi.com/2016/09/23/ measures-of-central-tendency-an-overview/
▪ https://www.spcforexcel.com/knowledge/comparing-processes/ box-and-whisker-plots
▪ ttps://academic.oup.com/ndt/article/29/3/529/1863883

▪ https://brownmath.com/stat/shape.htm
▪ https://www.infona.pl/resource/ bwmeta1.element.baztech-2c099f16-75a3-413f-b3c2-
9e7707b999ec/content/partDownload/35d77a19-8cd2-3a95-be9b-5bfd303df8d8
▪ http://iridl.ldeo.columbia.edu/dochelp/StatTutorial/Homogeneity/index.html
https://www.slideshare.net/teratailayu7186/normal-distribution-
53857901
https://quizlet.com/16454073/quality-management-quiz-flash-
cards/


▪ https://www.researchgate.net/figure/Map-of-Georgia-counties-with-counties-in-
Atlantas-2000-metropolitan-statistical- area_fig1_277720436
http://citeseerx.ist.psu.edu/viewdoc/

download?doi=10.1.1.887.6584&rep=rep1&type=pdf

https://academic.oup.com/ndt/article/29/3/529/1863883
▪ http://blog.minitab.com/blog/real-world-quality-improvement/accuracy-vs-
precision- whats-the-difference
▪ https://www.moresteam.com/toolbox/measurement-system-analysis.cfm
▪ http://slideplayer.com/slide/6414299/
▪ https://www.sixsigma-institute.org/
▪ ▪ https://www.greycampus.com/opencampus/lean-six-

sigma-green-belt/ measurement- methods

▪ http://sixsigmastats.com/dmaic-roadmap/
▪ http://isoconsultantpune.com/measurement-systems-in-quality/
▪ https://www.moresteam.com/toolbox/measurement-system-analysis.cfm
▪ https://www.slideshare.net/hakeemrehman/10-measurement-system-analysis-msa
▪ https://www.greycampus.com/opencampus/lean-six-sigma-green-belt/ measurement-
methods
▪ https://www.sixsigma-institute.org/
▪ https://www.slideshare.net/MarkVictorManiGandan/ spc-implementation-flow-chart
▪ https://www.chartitnow.com/Chi-Square-Template.html
▪ http://stattrek.com/probability-distributions/f-distribution.aspx
▪ https://www.isixsigma.com/dictionary/chi-square-test/

▪ https://www.scribd.com/document/108580341/f-t-test
▪ http://www.six-sigma-material.com/Binomial-Distribution.html
▪ http://www.sixsigmadaily.com/tools-six-sigma-analysis-phase/
▪ https://www.isixsigma.com/dictionary/discrete-data/
https://explorable.com/f-
distribution
http://www.statisticshowto.com/probability-
distribution/


▪ https://www.usebackpack.com/resources/4290/download?1426792589
▪ https://msu.edu/course/msc/317/chi-sq.htm
▪ https://www.mathworks.com/help/stats/fcdf.html
▪ https://www.greycampus.com/opencampus/lean-six-sigma-black-belt/ classes-of-
distribution
▪ https://www.greycampus.com/opencampus/lean-six-sigma-black-belt/ inferential-
statistics
▪ https://www.sixsigma-institute.org/What_Is_Inferential_Statistics.php
▪ https://www.sixsigma-institute.org/
Six_Sigma_DMAIC_Process_Measure_Phase_Data_Collection_Strategy_Sampling.php
▪ https://onlinecourses.science.psu.edu/stat414/node/176/
▪ ttps://en.wikipedia.org/wiki/Snowball_sampling
▪ http://eccsf.ulbsibiu.ro/articole/vol63/637gulmez&kitapci&dortyol.pdf
▪ https://www.khanacademy.org/math/statistics-probability/designing-studies/ sampling-
methods-stats/a/sampling-methods-review
▪ http://shodhganga.inflibnet.ac.in/jspui/bitstream/10603/148543/10/
10_chapter%205.pdf
▪ http://qualityamerica.com/LSS-Knowledge-Center/statisticalinference/
central_limit_theorem.php
▪ http://www.six-sigma-material.com/Alpha-and-Beta-Risks.html
▪ https://www.greycampus.com/opencampus/lean-six-sigma-black-belt/ hypothesis-
testing
▪ http://www.oswego.edu/~leblanc/data/pracsig.ppt
▪ ttps://www.lean6sigmatraining.ch/example
▪ https://www.managementstudyguide.com/hypothesis-testing.htm
https://www.xlstat.com/en/solutions/features/one-sample-variance-test

▪ https://libguides.library.kent.edu/SPSS/PairedSamplestTest
▪ http://www.six-sigma-material.com/F-Test.html
▪ https://www.isixsigma.com/tools-templates/hypothesis-testing/ making-sense-
two- sample-t-test/
▪ https://www.xlstat.com/en/solutions/features/one-sample-t-and-z-tests
▪ http://static.gest.unipd.it/~livio/PDF/Solutions%20Manual.pdf
▪ http://home.ku.edu.tr/~mmuradoglu/ENGR201/
StatInferenceForTwoSampleChap10.ppt
https://statistics.laerd.com/spss-tutorials/ one-way-anova-using-spss-statistics.php
https://www.researchgate.net/topic/Pedology
http://www.whatissixsigma.net/regression-analysis/
▪ http://stattrek.com/regression/residual-analysis.aspx?Tutorial=reg
▪ https://www.thedataschool.co.uk/elnisa-marques/ residual-plots-tableau-important/
▪ https://pdxscholar.library.pdx.edu/cgi/
viewcontent.cgi?article=1156&context=honorstheses
▪ http://www.six-sigma-material.com/Correlation.html
▪ http://www.theijes.com/papers/vol6-issue6/Version-2/D0606022231.pdf
▪ https://mcalglobal.com/2018/02/22/ machine-learning-hello-world-using-python/
▪ https://onlinecourses.science.psu.edu/stat501/node/251/
ttp://stattrek.com/regression/residual-analysis.aspx
https://www.mathsisfun.com/data/correlation.html
http://www.scikit-
yb.org/en/latest/api/regressor/residuals.html
▪ tp://eprints.ugd.edu.mk/19293/1/IJMTT-V52P549.pdf
▪ https://www.sciencedirect.com/science/article/pii/S1996681416000079
▪ https://www.moresteam.com/toolbox/regression-analysis.cfm
http://blog.minitab.com/blog/adventures-in-statistics-2/ when-should-i-use-confidence-
intervals-prediction-intervals-and-tolerance-
intervals
https://www.isixsigma.com/tools-templates/normality/ making-data-normal-using-box-
cox-power-
transformation/
▪ https://www.scribd.com/document/368400902/ Multiple-Regression-Analysis-in-SPSS-
Statistics-Laerd-Statistics
▪ ttps://statistics.laerd.com/spss-tutorials/ multiple-regression-using-spss-statistics.php





▪ https://www.researchgate.net/post/
How_to_determine_accuracy_of_predictors_in_ordinary_regression_model_using_SPS
S
▪ http://davidmlane.com/hyperstat/B12386.html
▪ https://www.slideshare.net/aakashaw/mkrh-correlation-and-regression
https://stats.stackexchange.com/questions/214616/ confidence-interval-for-regression-
line-simple-linear-
regression
https://support.minitab.com/en-us/minitab-express/1/help-and-how-
to/basic-
statistics/inference/supporting-topics/basics/ what-is-a-confidence-interval/
http://www.dummies.com/careers/project-management/six-sigma/ how-to-calculate-
multiple-linear-regression-for-six-
sigma/

▪ https://www.digite.com/kanban/what-is-kanban/

▪ https://www.greycampus.com/opencampus/lean-six-sigma-green-belt/ poka-yoke-and-
kanban
▪ http://whatis.techtarget.com/definition/kanban
▪ https://www.slideshare.net/mosaruf/productivity-concepts
▪ http://theresonance.net/resonvate/
▪ https://goleansixsigma.com/control-phase-5-of-5-of-lean-six-sigma/
▪ http://open.lib.umn.edu/principlesmanagement/chapter/15-7-lean-control/
▪ https://www.beaconquality.com/news-and-press/2013/03/06/ ease-inc-integrates-
standardized-work-into-software-system/
▪ https://www.honsha.org/ standardized-work-hangs-ten-with-san-diegos-surfing-
culture-meeting-the-challeng es-of-leadership-culture-and-resistance/
▪ https://www.greycampus.com/opencampus/lean-six-sigma-green-belt/statistical-
process-control
▪ http://www.pqsystems.com/qualityadvisor/DataCollectionTools/sampling.php
▪ http://www.whatissixsigma.net/imr/
▪ https://www.itl.nist.gov/div898/handbook/pmc/section3/pmc323.htm
▪ https://www.spcforexcel.com/knowledge/process-improvement/data-collection
▪ https://www.spcforexcel.com/files/Types_of_Data.ppt
▪ http://isoconsultantpune.com/statistical-process-control-using-control-chart
https://www.sixsigma-institute.org/

Six_Sigma_DMAIC_Process_Control_Phase_Control_Chart_Selection.php

▪ https://www.spcforexcel.com/knowledge/attribute-control-charts/ np-control-charts
https://bmcmedresmethodol.biomedcentral.com/articles/10.1186/1471-2288-6-
8
https://www.moresteam.com/toolbox/statistical-process-control-
spc.cfm
http://learningspc.blogspot.com/
▪ https://www.spcforexcel.com/files/trainingpreviews/Process%20Capability%20Preview.
pdf
▪ https://www.greycampus.com/opencampus/lean-six-sigma-green-belt/elements-of-
control-plan
tps://peer.asee.org/ lean-six-sigma-case-study-within-a-public-school-district.pdf
http://isoconsultantpune.com/business-process-management/

You might also like